LONDON

- CHICAGO -

IQILI-1915 '. COLUMBUS + sai FRANCISCO

THIRD SERIES NEW YORK DALLAS.

~ -“ GINN AND COMPANY

BOSTON - ATLANTA -

“y

a rm a rm 0

2) B = iS : * a Ae 7. © < 3 Z = 4 2 %

3 Sy

g2 Entrance Examination Board

5B

* ¢ as « = 4 ; rie on . 4 am ] . 5. wh, : ey Pos oa y : : ' | Malevan | : : rt hen oh Yeh a 4 as le es ++ raed pees vy Fi - s ; 7 P : : 1 7 é - : Rie 5 a) 7 - + “3 :

Cotte

a - : - = ray poets Se, ee 25, Siete SES Speier Beenie Pato Saerame Se Sahate hy fol piers =: = 3 * : see | : ee See: Ss ee, 3 erereess : s are. owe ete ao. oe AS Sige eer: Seni sth: + = etree: parepers Tele te ixe? > od rerdee. Smog so. =, E $s = SF Msc orete Hy tet peat se =< meta a re pera ee ere 5 Saint Tn LG Er REG ESE A Tb fii BE wees ie erasretateeed ee on = EES ESET Ais strs pirecs sh cae = So eeenpaen se i = . ss oes easwesgere page perestrasireraeh eeree: eee TS - mo Pearse s eens ia ioe egiose shone sede hor taae S = - pre tte 4 FA tene sar t ms Rd pepe mow eet mere: =. SSAoweds oe > PSE IE! <4 7. = z - aperee 4

f Sit 13

<> 8

Fae

OMAK CAD ORE

Carerane Pe rante La

CCA fi

a Pca Cy

iy ae fan bil we

College Entrance Examination Board

EXAMINATION QUESTIONS

IN

LATIN AND GREEK

THIRD SERIES

IQII-1Q15

GINN AND COMPANY

BOSTON - NEW YORK - CHICAGO - LONDON ATLANTA + DALLAS - COLUMBUS - SAN FRANCISCO

QII, 1912, 1913, 1914, 1915

- COPYRIGHT, I

BY THE

-

CoLLEGE ENTRANCE EXAMINATION BoarD

ALL RIGHTS RESERVED

217.1

PREFACE

While the annual volume of examination questions published by the College “Entrance Examination Board has met the needs of many candi- dates for examination and their teachers, the Board is constantly in receipt of communications asking for the questions set in certain subjects in succes- sive years. In order to meet this demand the Board has prepared pamphlets containing the questions in certain subjects from I9I1 to I9QI5 inclusive. These pamphlets are as follows: |

I. Examination questions in Latin and Greek, I191I-—I9QI5.

2. Examination questions in English and other modern languages, IQII-IQI5. ,

3. Examination questions in mathematics, I9I1I-I9QI5.

4. Examination questions in history, I91I-I915.

5. Examination questions in the natural sciences and in drawing, IQII-IQI5.

Besides meeting the needs of candidates for examination and their teachers, these publications ought to have more widely a beneficial influence upon teaching for the reason that they illustrate in concrete form principles agreed upon by many leading teachers of the subjects represented.

366462 3

_

uu ff WN

OSwyS Sanwa

CONTENTS

LATIN

. GRAMMAR

. ELEMENTARY PROSE COMPOSITION . SECOND YEAR LATIN

. Cicero Manilian Law and Archias, and Sight Translation of Prose . VERGIL Z£neid, Books I, II, and IV or VI, and Sight Translation of

Poetry . ADVANCED PROSE COMPOSITION . Casar Gallic War, Books I-IV . CICERO Six Orations VERGIL “xed, Books I-VI ELEMENTARY SIGHT TRANSLATION OF PROSE ADVANCED SIGHT TRANSLATION OF PROSE

. SIGHT TRANSLATION OF POETRY

GREEK

. GRAMMAR . ELEMENTARY PROSE COMPOSITION . XENOPHON Axadbasis, Books I-IV

Homer //iad, Books I-III PROSE COMPOSITION SIGHT TRANSLATION OF PROSE

. SIGHT TRANSLATION OF HOMER (IQII-I913) . . HoMER //zad, Books I-III, and Sight Translation of Homer

in

105

III 117 123 135 147 153 159 163

Digitized by the Internet Archive in 2007 with funding from - Microsoft Corporation

https://archive.org/details/examinationquest0Ocollrich

A as ae)

rs ym 2s Fi ~

& 2 “LATIN NRI-GRAMMAR Wednesday II.I5 a. M.-12.30p. m.

Do not write a translation of the following passage, but answer the questions in order.

Itaque non sum praedicaturus quantas ille res domi militiae, terra marique, quantaque felicitate gesserit, ut eius semper voluntatibus non modo cives adsenserint, socii obtemperarint, hostes oboedie- rint, sed etiam venti tempestatesque obsecundarint. Hoc bre- vissime dicam, neminem umquam tam impudentem fuisse qui ab 5 dis immortalibus tot et tantas res tacitus auderet optare quot et quantas di immortales ad Cn. Pompeium detulerunt. Quod ut illi proprium ac perpetuum sit, Quirites, cum communis salutis atque imperi tum ipsius hominis causa, sicuti facitis, velle et optare debetis.—Cicero, Pro Lege Manilia, 48. 10

1. Tell in what case each of the following words is, and why this case is used: domi (line 1), felicitate (line 2), voluntatibus (line 2), meminem (line 5), salutis (line 8).

2. Tell in what mood and what tense each of the following verbs is, and why this mood and this tense are used: gesserit (line 2), fuisse (line 5), auderet (line 6).

3. Explain the gender of quod (line 7). 4. What is the syntax of the clause quod ut . . . . sit (line 7)? ~

5. Decline ile (line 1) in the neuter singular and plural, res (line 1), cives (line 3), immortalibus (line 6) in the neuter singular and plural, hominis (line 9).

6. Write the nominative singular of neminem (line 5), dis (line 6), salutis (line 8), imperi (line 9).

7. Compare brevissime (line 4), impudentem (line 5). 8. Give the principal parts of gesserit (line 2), auderet (line 6), debetis (line 10).

9. Conjugate dicam (line 5) in the present imperative active and the perfect indicative active, detulerunt (line 7) in the present indicative passive and the imperfect subjunctive passive, facitis (line 9) in the future indicative active and the pluperfect subjunctive active.

10. Write all the infinitives of dicam (line 5), naming each. 11. Explain the derivation of voluntatibus (line 2), immortalibus (line 6).

12. Divide the following words into their syllables, and indicate the quantity of their penults and final syllables and their accent: impudentem (line 5), auderet (line 6), detulerunt (line 7), imperi (line 9), ipsius (line 9).

IQI2

LATIN NRI—GRAMMAR

Wednesday II.I5 a. M.—12.30 p. m.

respondeam. Ego multos homines excellenti animo ac virtute fuisse sine doctrina, et naturae ipsius habitu prope divino per se ipsos et moderatos et gravis exstitisse fateor; etiam illud adiungo, saepius ad laudem atque virtu- tem naturam sine doctrina quam sine natura valuisse doctrinam. Atque idem ego hoc contendo, cum ad naturam eximiam et inlustrem accesserit ratio quaedam conformatioque doctrinae, tum illud nescio quid praeclarum ac singulare solere exsistere. Ex hoc esse hunc numero, quem patres nostri viderunt, divinum hominem Africanum, ex hoc fortissimum virum et illis temporibus doctissimum, M. Catonem illum senem; qui profecto si nihil ad percipiendam colendamque virtutem litteris adiuvarentur, numquam se ad

Do not write a translation of the following passage, but answer the questions in order.

Difficile est hoc de omnibus confirmare, sed tamen est certum quid

0O ON AM PW DN H

me oH eH O

earum studium contulissent.—Cicero, Pro Archia, 15, 16. 12

I.

Decline ego (line 2), habitu (line 3), virum (line 9), senem (line 10); decline in the singular zpsius (line 3).

. Conjugate fateor (line 4) in the future indicative, adiungo (line 4) in the

perfect indicative active, nescio (line 7) in the present subjunctive active, contulissent (line 12) in the present indicative active and the present imperative active. Write all the participles of viderunt (line 9), naming each.

. Write the nominative singular masculine of excellenti (line 2), quaedam

(line 7), singulare (line 8); write the first person singular present indicative active of valuisse (line 5), accesserit (line 6).

. Compare difficile (line 1), saepius (line 4).

5. Write the principal parts of fateor (line 4), contendo (line 6), selene (line 8),

Io.

adiuvarentur (line 11).

. Tell in what case each of the following words is, and why this case is used:

animo (line 2), habitu (line 3), temporibus (line 10), virtutem (line 11).

. Tell in what mood and tense each of the following verbs is, and why this

mood and this tense are used: respondeam (line 2), accesserit (line 6), exsistere (line 8).

. What form of conditional sentence is qui... . contulissent (lines 10-12) ?

. Explain the derivation of naturae (line 3), virtutem (line 11).

Divide the following words into their syllables, mark the quantity of their penults and final syllables, and indicate the accent: respondeam (line 2), doctrina (line 3), ipsius (line 3), praeclarum (line 7), temporibus (line 10).

LATIN 1—GRAMMAR =

Wednesday II.1I5 a.m.-I p.m.

Nunc imperi vestri splendor illis gentibus lucem adferre coepit. Nunc intellegunt non sine causa maiores suos, tum cum ea temperantia magistra- tus habebamus, servire populo Romano quam imperare aliis maluisse. Iam vero ita faciles aditus ad eum privatorum, ita liberae querimoniae de aliorum iniuriis esse dicuntur ut is, qui dignitate principibus excellit, facilitate infimis par esse videatur. Iam quantum consilio, quantum dicendi gravitate et copia valeat, in quo ipso inest quaedam dignitas imperatoria, vos, Quirites, hoc ipso ex loco saepe cognovistis. Fidem vero eius quantam inter socios existimari putatis, quam hostes omnes omnium generum sanctissimam iudi- carint? Humanitate iam tanta est ut difficile dictu sit utrum hostes magis 1

Do not write a translation of the following passage, but answer the questions in order.

0MDO ON AN PW DN HS

virtutem eius pugnantes timuerint an mansuetudinem victi dilexerint. Et 11 quisquam dubitabit quin huic hoc tantum bellum transmittendum sit, qui 12 ad omnia nostrae memoriae bella conficienda divino quodam consilio natus 13 esse videatur >—Pro Lege Manilia, 41, 42. 14

I.

iS)

ff &w

To.

II.

Decline in full gentibus (line 1), aditus (line 4), aliorum (line 4); par (line 6). vos (line 7), generum (line 9).

. Conjugate adferre (line 1) in the present indicative passive, servire (line 3)

in the imperfect indicative active, maluisse (line 3) in the imperfect sub- junctive, excellit (line 5) in the present subjunctive active, inest (line 7) in the perfect subjunctive, vicii (line 11) in the present imperative active,

. Write all the infinitives of cognovistis (line 8), naming each. . Write the ablative singular of maiores (line 2), the accusative singular neuter

of faciles (line 4), the dative singular feminine of 7s (line 5), the nominative singular masculine of /iberae (line 4); write the first person singular, present indicative of matus (line 13).

. Compare faciles (line 4), infimis (line 5). . Write the principal parts of intellegunt (line 2), valeat (line 7), timuerint (line

11), dilexerint (line 11).

. Tell in what case each of the following words is, and why this case is used:

populo (line 3), dignitate (line 5), quantum (line 6), humanitate (line to), conficienda (line 13).

. Tell in what mood each of the following verbs is, and why this mood is used:

maluisse (line 3), valeat (line 7), iudicarint (line 9), transmittendum sit (line 12), videatur (line 14).

. Tell in what tense each of the following verbs is, and why this tense is used:

maluisse (line 3), valeat (line 7), tudicarint (line 9), transmitiendum sit (line 12), videatur (line 14).

Explain the derivation of iniuriis (line 5), principibus (line 5), humanitate (line ro).

Divide the following words into syllables, mark the quantity of the penults and final syllables, and indicate the accent: intellegunt (line 2), infimis (line 5), copia (line 7), timuerint (line 11), divino (line 13).

10

1914

LATIN 1—GRAMMAR

Wednesday II.I5 a.m.-I p.m.

Do not write a translation of the following passage, but answer the questions in order.

Quaeres a nobis, Grati, cur tanto opere hoc homine delectemur. Quia suppeditat nobis ubi et animus ex hoc forensi strepitu reficiatur et aures con- vicio defessae conquiescant. An tu existimas aut suppetere nobis posse quod cottidie dicamus in tanta varietate rerum, nisi animos nostros doctrina excolamus, aut ferre animos tantam posse contentionem, nisi eos doctrina eadem relaxemus? Ego vero fateor me his studiis esse deditum. Ceteros pudeat, si qui ita se litteris abdiderunt ut nihil possint ex iis neque ad com- munem adferre fructum neque in aspectum lucemque proferre; me autem quid pudeat, qui tot annos ita vivo, iudices, ut a nullius umquam me tempore aut commodo aut otium meum abstraxerit aut voluptas avocarit aut denique somnus retardarit ? Qua re quis tandem me reprehendat, aut quis mihi iure suscenseat, si, quantum ceteris ad suas res obeundas, quantum ad festos dies ludorum celebrandos, quantum ad alias voluptates et ad ipsam requiem animi et corporis conceditur temporum, quantum alii tribuunt tempestivis convi- viis, quantum denique alveolo, quantum pilae, tantum mihi egomet ad haec - gstudia recolenda sumpsero ?>—Cicero, Pro Archia, 12, 13.

Oo Ons AN PW DN H

eo e O

12 T3 14 T5 16

1. Decline in full modis (line 1), rerum (line 4), eos (line 5), litteris (line 7), fruc-

tum (line 8), ludorum (line 13).

2. Conjugate quaeres (line 1) in the pluperfect subjunctive active, reficiatur (line 2) in the imperfect subjunctive passive, conquiescant (line 3) in the present imperative active, abdiderunt (line 7) in the future indicative

active, abstraxerit (line 10) in the imperfect subjunctive active. 3. Write all the infinitives of reficiatur (line 2), naming each.

4. Write the accusative singular of opere (line 1), the nominative singular neuter of eadem (line 6), the nominative plural neuter of communem (lines 7, 8). 5. Write the principal parts of excolamus (line 5), proferre (line 8), vivo (line 9),

avocarit (line 10).

6. Tell in what case each of the following words is, and why this case is used: nobis (line 2), convicio (lines 2, 3), quod (line 3), annos (line 9), commodo

(line 10), temporum (line 14), mihi (line 15).

7. Tell in what mood each of the following verbs is, and why this mood is used: delectemur (line 1), reficiatur (line 2), excolamus (line 5), pudeat (line 7),

possint (line 7).

8. Tell in what tense each of the following verbs is, and why this tense is used: dicamus (line 4), esse deditum (line 6), possint (line 7), vivo (line 9), abstrax-

erit (line 10).

g. Explain the derivation of forensi (line 2), reficiatur (line 2), conviviis (lines

14, 15).

10. What is the gerundive? Explain its use in ad suas res obeundas (line 12). 11. Divide the following words into their syllables, mark the quantity of their penults and final syllables, and indicate the accent: forensi (line 2), ad-

diderunt (line 7), nullius (line 9), requiem (line 13), temporum (line 14). It

1915

LATIN 1—GRAMMAR

Wednesday II.15 a. M.-I p.m.

Do not write a translation of the following passage, but answer the questions in order.

Et quoniam auctoritas quoque in bellis administrandis multum atque in imperio militari valet, certe nemini dubium est quin ea re idem ille impera- tor plurimum possit. Vehementer autem pertinere ad bella administranda quid hostes, quid socii de imperatoribus nostris existiment quis ignorat, cum sciamus homines in tantis rebus ut aut contemnant aut metuant aut oderint aut ament opinione non minus et fama quam aliqua ratione certa commoveri? Quod igitur nomen umquam in orbe terrarum clarius fuit ? Cuius res gestae pares? De quo homine vos, id quod maxime facit auctori- tatem, tanta et tam praeclara iudicia fecistis? An vero ullam usquam esse oram tam desertam putatis quo non illius diei fama pervaserit, cum uni- versus populus Romanus, referto foro completisque omnibus templis ex quibus hic locus conspici potest, unum sibi ad commune omnium gentium bellum Cn. Pompeium imperatorem depoposcit >—Cicero, Pro Lege Manilia, 43, 44.

0 CONTI AN BW DN H

He He eH S HF O

H w

1. Divide the following words into their syllables, mark the quantity of their

penults and final syllables, and indicate the accent: imperio (line 2), vehementer (line 3), oderint (line 6), praeclara (line 9).

. Decline in full militari (line 2), orbe (line 7), clarius (line 7), tudicia (line 9);

decline in the feminine, singular and plural, idem (line 2).

. Conjugate valet (line 2) in the perfect subjunctive active, possit (line 3) in

the imperfect indicative, existiment (line 4) in the present subjunctive passive, sciamus (line 5) in the future indicative active.

. Write all the infinitives and participles of gestae (line 8), naming each. . Write the nominative singular masculine of nostris (line 4), aliqua (line 6).

and pares (line 8); the nominative plural neuter of quod (line 8); the genitive plural of momen (line 7). Write the third person singular, future indicative passive of commoveri (line 7).

6. Compare plurimum (line 3); compare the adverb formed from clarius (line 7).

To.

II. I2.

13.

. Write the principal parts of administranda (line 3), contemnant 5),

metuant (line 5), commoveri (line 7).

. Tell in what case each of the following words is, and why this case is used:

nemini (line 2), re (line 2), homines (line 5), opinione (line 6), sibi (line 12).

. Tell in what mood each of the following verbs is, and why this mood is used:

pertinere (line 3), existiment (line 4), oderint (line 6), pervaserit (line 10), depoposcit (line 13).

Tell in what tense each of the following verbs is, and why this tense is used: pertinere (line 3), existiment (line 4), oderint (line 6), pervaserit (line 10), depoposcit (line 13).

Explain the derivation of militari (line 2), imperator (line 2), iudicia (line 9).

Give four ways of expressing purpose in Latin. Write a Latin sentence illustrating each.

Mention three kinds of Latin relative clauses the verb of which is regularly

in the subjunctive. ss

IQII

LATIN NR2—ELEMENTARY PROSE COMPOSITION

Thursday II.15 a. M.-12.30 p, mM.

Translate into Latin:

I.

They thought that they ought to choose a man of the greatest skill to carry on this war.

. If we use all the provisions that we have, we shall be able to resist the enemy

for ten days.

. He tried to persuade them not to receive any one into the town, but they

were unwilling to listen.

While I was returning to Rome, Cato reached the city and told the senate what I had done.

Both men waited to see whether the rest of the ships would assemble before nightfall.

Cicero said that Archias was worthy of citizenship because he had glorified (illustrare) the exploits of the Roman people.

Volturcius was arrested (comprehendere) by the praetors before he could cross the bridge.

14

Igiz LATIN NR2—ELEMENTARY PROSE COMPOSITION

Thursday II.15 a. M.—12.30 p. m. Translate into Latin: ;

1. After Lucullus had waged war with Mithridates seven years, Glabrio was put in command of the army.

2. Since the new general was a man of no ability, the Roman forces were no . longer a protection to the allies.

3. The following year Cicero urged the people to intrust the war to Pompey, who was not then at Rome.

4. If you had not read Cicero’s speech, you would ask why Pompey seemed to him to be the only man who could conquer the king.

5. Do you think that he was worthy of this praise?

6. Having undertaken the war at once, he advanced with such courage that Mithridates was quickly driven from his kingdom.

7. Finally the wretched king, having been captured by his foes, ordered one of his men to kill him.

T5

1913 LATIN 2—ELEMENTARY PROSE COMPOSITION

Thursday II.15 a.m.-I p.m.

Translate into Latin:

I.

When he was a youth, Archias was so famous at home that all men wished to see him.

. It is said that he came to Rome in the consulship of Marius and Catulus. . No one can doubt that the former was more powerful than the latter at that

time.

. If you should ask what Marius did, I would tell you. . Many years after a certain Grattius tried to expel the poet from the city,

because (as Grattius said)! he was not a citizen.

. Cicero thought that he ought to speak for the friend who had helped him.

7. And so it happened that he left this speech, to which you have given so much

time.

8. When he had answered Grattius, he began to use a new style of speaking.

Io.

. This he did with the greatest zeal, for the subject was pleasing to him.

Who is there who is not moved by love for a friend ?

tIt is not necessary to translate into Latin the English words in the parenthesis.

16

1014 LATIN 2—ELEMENTARY PROSE COMPOSITION

Thursday ) II.15 a.m.-1 p.m.

Translate into Latin:

I. 2.

YO COS eA

Io.

Do you ask me, fellow-citizens, what Lucullus did in Asia?

In the year in which he was put in charge of this dangerous war, messengers had come to Rome from our allies.

. These men had been sent to ask us for help. . The knights also thought that we ought to protect them. . Although Mithridates had often been defeated by Roman generals, he was

then threatening the whole province.

. He hoped within a short time to conquer the Roman forces on land and sea. . But Lucullus arrived in Asia before the king could accomplish this.

. The latter, despairing of his fortunes, fled as swiftly as possible to Tigranes. . He left behind much gold and silver and very many most beautiful objects.

If our soldiers had not begun to collect this booty, they would have captured their enemy.

17

IQI5 LATIN 2—ELEMENTARY PROSE COMPOSITION

Thursday II.15 a. M.-I p. m.

Translate into Latin: 1. In the consulship of Lepidus and Tullus, Cicero asked the people to send Pompey to the war in Asia as commander.

2. Catulus and Hortensius, though they were men of very great influence, could not prevent this from being done.

3. The result was that the Romans, having quickly defeated Mithridates, were freed from their fear.

. We shall now speak of the other oration, which all of you enjoy.

. It is shorter than other speeches, but it ought not to be neglected by you.

There is no one of you that is not moved by desire for praise.

. It is difficult to say how much help Archias was to Cicero.

. After he had come to the city of Rome, he began to celebrate the fame of the Roman people in his verses.

9. Did anyone believe that he surpassed all poets in genius? 10. We praise Archias because he chose Cicero to defend him.

on nn +

18

IQII

LATIN NR3—SECOND YEAR LATIN

Wednesday Q-II a. m.

Translate the following passage, and answer the questions. Read the passage

through several times before beginning to write. The translation should be exact, but in clear and natural English.

His copiis coactis ad Bellovacos proficiscitur, castrisque in eorum finibus positis equitum turmas' dimittit in omnis partis ad aliquos ex- cipiendos ex quibus hostium consilia cognosceret. Equites officio functi? renuntiant paucos in aedificiis esse inventos, atque hos, non qui agrorum colendorum’ causa remansissent (namque esse undique diligenter demigratum), sed quispeculandi‘ causa essent remissi. A quibus cum quaereret Caesar, quoloco multitudo esset Bellovacorum quodve esset consilium eorum, inveniebat Bellovacos omnis, qui arma ferre possent, in unum locum convenisse, itemque Ambianos, Aulercos, Caletos, Veliocasses, Atrebates; locum castris excelsum® in silva circumdata palude delegisse, impedimenta omnia in ulteriores silvas contulisse.—De Bello Gallico, viii, 7.

tturmas, ‘squadrons.’ ? functi, participle of fungor, ‘perform.’ 3colendorum, from colo, , p p » P

‘cultivate.’ 4speculandi, from speculor, ‘spy.’ 5 quodve may be translated asif it were guodque. 6 excelsum, high.’

I.

Tell in what case each of the following words is, and why this case is used: castris (line 1), paucos (line 4), palude (line 11).

. Tell in what mood and what tense each of the following verbs is, and why

this mood and this tense are used: cognosceret (line 3), esset (line 7), convenisse (line 9).

Explain the difference in construction between agrorum colendorum causa (line 5) and speculandi causa (line 6).

Decline partis (line 2), hos (line 4) in the neuter singular and plural, arma (line 9), wnum (line 9) in the feminine singular.

Compare diligenter (line 6).

Conjugate coactis (line 1) in the perfect indicative active, positis (line 2) in the future indicative passive, possent (line 9) in the perfect subjunctive, convenisse (line g) in the imperfect subjunctive active.

Write in Latin: The Bellovaci heard that Caesar was coming into their country with three legions. When they heard this, they sent Commius to ask help from the Germans. If Caesar had not set out at that time, they would have collected larger forces.

20

| IQI2 LATIN NR3—SECOND YEAR LATIN

Wednesday Q-II a.m..

Translate the following passage, and answer the questions. Read the passage through several times before beginning to write. The translation should be exact, but in clear and natural English.

Vatinius Brundisii cum esset, cognitis rebus quae gestae erant in Illyrico, cum crebris litteris Cornifici ad auxilium provinciae ferendum evocaretur et M. Octavium audiret cum barbaris foedera percussisse? compluribusque locis nostrorum militum oppugnare praesidia partim classe per se, partim pedestir- bus copiis per barbaros, etsi gravi valetudine? adfectus vix corporis viribus animum sequebatur, tamen virtute vicit incommodum naturae difficulta- tesque et hiemis et subitae praeparationis. Nam cum ipse paucas in portu

naves longas haberet, litteras in Achaiam ad Q. Calenum misit, uti sibi -classem mitteret. Quod cum tardius fieret quam periculum nostrorum flagitabat,’ qui sustinere impetum Octavi non poterant, navibus actuarlis,‘ quarum numerus erat satis magnus, magnitudo nequaquams satis iusta® ad proeliandum, rostra? imposuit.—De Bello Alexandrino, 44. 12

t foedera percussisse, ‘had made alliances.’ ?valetudine, ‘illness.’ 3 flagitabat, equivalent to postulabat. 4actuariis, ‘swift-sailing.’ 5 nequaguam, ‘by no means.’ ‘iusta, equivalent to idonea. ‘rostra, ‘beaks.’ ,

C0 Ons Aun LW ND EH

Ss eS me O

1. Tell in what case each of the following words is, and why this case is used: Brundisii (line 1), litieris (line 2), provinciae (line 2), locis (line 3), navibus (line 10).

2. Tell in what mood and tense each of the following verbs is, and why this mood

and this tense are used: percussisse (line 3), sequebatur (line 6), mitteret (line 9), fieret (line 9).

3. Account for the choice of the pronoun s7d7 in line 8.

4. Decline crebris (line 2) in the masculine singular, gravi (line 5) in the feminine singular and the neuter plural; decline in full corporis (line 5), viribus (line 5), portu (line 7).

5. Conjugate gestae erant (line 1) in the present subjunctive active, audiret (line 3) in the future indicative active, fieret (line 9) in the pluperfect indicative, poterant (line 10) in the imperfect subjunctive.

6. Translate into Latin:

a) While these things were being done, Caesar ordered Vatinius to come to Brundisium with a fleet.

b) Caesar would have been able to conquer the enemy if he had had a larger number of ships.

c) Octavius heard that Caesar had set out to besiege the city.

21

LATIN 3—SECOND YEAR LATIN oe

Wednesday Q-II a.m.

Translate the following passage, and answer the questions. Read the passage through several times before beginning to write. The translation should be exact, but in clear and natural English.

Insequenti nocte Fabius equites praemittit sic paratos ut confligerent! atque agmen morarentur, dum consequeretur ipse. Cuius praeceptis ut res gereretur, Q. Atius Varus, praefectus equitum, singularis et animi et pru- dentiae vir, suos hortatur agmenque hostium consecutus turmas? partim idoneis locis disponit, partim equitum proelium committit. Confligit auda- cius equitatus hostium succedentibus sibi peditibus; qui toto agmine sub- sistentes? equitibus suis contra nostros ferunt auxilium. Fit proelium acri certamine. Namque nostri contemptis‘ pridie superatis hostibus, cum sub- sequi legiones meminissent, et pudores cedendi et cupiditate per se conficiendi proeli fortissime contra pedites proeliantur; hostesque nihil amplius copiarum accessurum credentes, ut pridie cognoverant, delendi® equitatus nostri nacti occasionem videbantur. Cum aliquamdiu summa contentione dimicaretur, Dumnacus instruit aciem, quae suis esset equitibus in vicem? praesidio. 13

De Bello Gallico, viii, 28, 29.

t confligerent, equivalent to dimicarent. *turmas, ‘squadrons’. 3subsistentes, equivalent to consistentes. 4contemptis, from contemnere, ‘to despise’. 5 pudore,‘shame’. °delendi, from delere, to destroy’. .7i vicem, ‘in turn’.

oO Ons Amn PWD H

Se & & S&S H O

1. Tell in what case each of the following words is, and why this case is used: animi (line 3), sibi (line 6), peditibus (line 6), certamine (line 8), equitatus (line 11), praesidio (line 13).

2. Tell in what mood each of the following verbs is, and why this mood is used: consequeretur (line 2), gereretur (line 3), nacti (line 11), esset (line 13).

3. Tell in what tense each of the following verbs is, and why this tense is used: consecutus (line 4), accessurum (line 11), instruit (line 13), esset (line 13).

4. Decline in full agmen (line 2), vir (line 4), equitatus (line 6); decline in the feminine singular foto (line 6) and acri (line 7).

5. Compare idoneis (line 5), audacius (line 5).

6. Conjugate gereretur (line 3) in the perfect indicative active, fit (line 7) in the pluperfect subjunctive, conficiendi (line g) in the present subjunctive active, nacti (line 11) in the present indicative; write the present imperative forms, active and passive, of ferunt (line 7).

7. Translate into Latin:

a) If the foot-soldiers should fight more bravely, the enemy would not be able to defeat the cavalry. $5

b) Varus believed that the legions would have to yield.

c) Iam encouraging our men not to wait until the enemy attack them.

22

LATIN 3—SECOND YEAR LATIN es

Wednesday Q-II a.m.

Translate the following passage, and answer the questions. Read the passage through several times before beginning to write. The translation should be exact, but in clear and natural English.

[A storm causes serious inconvenience to Caesar’s forces.]

Tanta enim tempestas cooritur ut numquam illis locis maiores aquas fuisse constaret. ‘Tum autem ex omnibus montibus nives' proluit? ac summas ripas fluminis superavit pontisque ambos’ quos C. Fabius fecerat uno die interrupit.4 Quae res magnas difficultates exercitui Caesaris attulit. Cas- tra enim, ut supra demonstratum est, cum essent inter flumina duo, Sicorim et Cingam, spatio milium xxx, neutrum horum transiri poterat, necessarioque omnes his angustiis continebantur. Neque civitates quae ad Caesaris amicitiam accesserant frumentum supportare neque ii qui pabulatumS longius progressi erant interclusi fluminibus reverti neque maximi commeatus qui ex Italia Galliaque veniebant in castra pervenire poterant. Tempus erat 10 autem difficillimum, quo neque frumenta in hibernis erant neque multum a _ 11 maturitate® aberant, et civitates exinanitae,7 quod Afranius paene omne 12 frumentum ante Caesaris adventum Ilerdam convexerat,® reliqui si quid 13 fuerat, Caesar superioribus diebus consumpserat.—De Bello Civili, i, 48. ' 14

tives, ‘snow.’ 7? proluit, ‘washed away.’ ambos, ‘both.’ ‘4interrupit, ‘broke down.’ Spabulatum, from pabulari, ‘to forage.’ °maturitate, ‘ripeness.’ ‘7exinanitae, supply erant, ‘were stripped.’ ®convexerat, ‘had conveyed.’

©O OnN AN hW DN HE

1. Tell in what case each of the following words is, and why this case is used:

me ve 1), fluminibus (line 9), quo (line 11), [lerdam (line 13), reliqui ne 13).

2. Tell in what mood each of the following verbs is, and why this mood is used: constaret (line 2), essent (line 5).

3. Tell in what tense each of the following verbs is, and why this tense is used: cooritur (line 1), fuisse (line 2), constaret (line 2), fecerat (line 3).

4. What form of the verb is pabulatum (line 8)? What idea does this form express ?

5. Write out the Latin words for which C. (line 3) and xxx (line 6) stand.

6. Decline in full locis (line 1), maiores (line 1), montibus (line 2), milium (line 6) ; decline duo (line 5) in the feminine plural and quid (line 13) in the mascu- line singular.

7. Compare longius (line 8), dificstlinaen (line 11).

8. Conjugate attulit (line 4) in the present indicative active, transiré (line 6) in the imperfect subjunctive active, veniebant (line 10) in the perfect indicative active, consumpserat (line 14) in the future indicative passive; write all the infinitives of progressz (line 9), and name each.

g. Translate into Latin:

a) It was reported to the enemy that the supplies that were being brought to Caesar were approaching the river.

b) When they received that news, they sent horsemen, that the river might not be crossed.

c) The danger was so great that our men did not dare to resist them.

23

1915 LATIN 3—SECOND YEAR LATIN

Wednesday Q-II a.m.

Translate the following passage, and answer the questions. Read the passage through several times before beginning to write. The translation should be exact, but in clear and natural English.

(Caesar, in quest of information, finds himself exposed to an unexpected attack.|

Caesar, ut per se consilium caperet quid faciendum videretur, navem con- scendit atque omnem classem se sequi iussit nullis nostris militibus impositis, quod, cum longius paulo discederet, munitiones nudare nolebat. Cumque ad eum locum accessisset, qui appellatur Chersonensus, aquandique’ causa remiges? in terram exposuisset, non nulli ex eo numero, cum longius a navibus praedatum processissent, ab equitibus hostium sunt excepti. Ex his cogno- verunt Caesarem ipsum in classe venisse nec ullos milites in navibus habere. Qua re comperta magnam sibi facultatem fortunam obtulisse bene gerendae rei crediderunt. Itaque navis omnis quas paratas habuerant ad navi- gandum propugnatoribus instruxerunt Caesarique redeunti cum classe occurrerunt. Qui duabus de causis eo die dimicare nolebat, quod et nullos milites in navibus habebat et post horam x diei res agebatur, nox autem adlatura videbatur maiorem fiduciamS illis, qui locorum notitia‘ confidebant. 13

De Bello Alexandrino, to.

*aquandi, from aguari, ‘to get water.’ *remiges, ‘oarsmen.’ 3 fiduciam, ‘confidence.’ 4 notitia, ‘knowledge.’

oOo wont Aun LhW DN

eH eR eH SOS fF O

1. Tell in what case each of the following words is, and why this case is used: quid (line 1), sibi (line 8), rez (line g), die (line 11). Why is the preposition necessary in ab equitibus (line 6)?

2. Tell in what mood each of the following verbs is, and why this mood is used: caperet (line 1), videretur (line 1), venisse (line 7).

3. Tell in what tense each of the following verbs is, and why this tense is used: caperet (line 1), venisse (line 7), habuerant (line 9), redeunti (line 10).

4. What form of the verb is faciendum (line 1)? Explain fully its use in this place.

5. Decline in full numero (line 5), redeunti (line 10), nox (line 12); decline mullis (line 2) in the masculine singular, and zpswm (line 7) in the feminine, sin- gular and plural.

6. Compare bene (line 8).

7. Conjugate sequi (line 2) in the present subjunctive, iussi¢ (line 2) in the future indicative active, nolebat (line 3) in the imperfect subjunctive, instruxerunt (line 10) in the imperfect indicative active.

24

8. Write the principal parts of impositis (line 2), confidebant (line 13), naming each part. 9. Translate into Latin:

a) While he was forming this plan, Caesar learned that the leaders of a neighboring state wished to attack him on the following night.

b) But he had left his own soldiers on land, that the larger camp might be defended for a few days from great danger.

c) And so those whom he had with him begged him not to fight with this multitude of the enemy.

25

LATIN 4

-

CICERO— MANILIAN LAW AND ARCHTAS, AND SIGHT TRANS- LATION OF PROSE

27

IQII LATIN NR4—CICERO (MANILIAN LAW AND ARCHIAS) AND SIGHT TRANSLATION OF PROSE

Thursday : . Q-II a. Mm.

Translate the following passages, and answer the questions. The translation should be exact, but in clear and natural English. It is important that enough time be given to II to deal with it satisfactorily.

I

1. Quare videant ne sit periniquum et non ferendum illorum auctoritatem de Cn. Pompei dignitate a vobis comprobatam semper esse, vestrum ab illis de eodem homine iudicium populique Romani auctoritatem improbari, praesertim cum iam suo iure populus Romanus in hoc homine suam auctoritatem vel contra omnis qui dissentiunt possit 5 defendere, propterea quod isdem istis reclamantibus vos unum illum ex omnibus delegistis quem bello praedonum praeponeretis. Hoc si vos temere fecistis et rei publicae parum consuluistis, recte isti studia vestra suis consiliis regere conantur.—Pro Lege Manilia, 63, 64.

a) Explain briefly Cicero’s argument in this passage.

b) What is the force of dis in dissentiunt (line 5)? What meaning is there in istt (line 9) which ili would not have?

2. Cum hic domicilium Romae multos iam annos haberet, professus est apud praetorem Q. Metellum familiarissimum suum. Si nihil aliud nisi de civitate ac lege dicimus, nihil dico amplius; causa dicta est. Quid enim horum infirmari, Grati, potest? Hera- cliaene esse eum ascriptum negabis? Adest vir summa auctoritate 5 et religione et fide, M. Lucullus, qui se non opinari sed scire, non audivisse sed vidisse, non interfuisse sed egisse dicit. Adsunt Heraclienses legati, nobilissimi homines; huius iudici causa cum mandatis et cum publico testimonio venerunt, qui hunc ascriptum Heracliensem dicunt.—Pro Archia, 7, 3. Io

a) What was the charge against Archias? What penalty would have been imposed upon him if the case had gone against him? What was his legal defense ?

b) Where was Heraclia ? c) Explain the tense of haberet (line 1).

28

II Read the passage through several times before beginning to write the translation. [Laelius speaks of his friendship for Scipio]

Mihi quidem Scipio, quamquam est subito ereptus, vivit tamen semper- que vivet; virtutem enim amavi illius viri, quae exstincta non est. Nec mihi soli versatur ante oculos, qui illam semper in manibus habui, sed etiam posteris erit clara et insignis. Nemo umquam animo aut spe maiora suscipiet qui sibi non illius memoriam atque imaginem proponendam putet. Equidem* ex omnibus rebus, quas mihi aut fortuna aut natura tribuit, nihil habeo quod cum amicitia Scipionis possim comparare. . . . . Numquam illum ne minima quidem re offendi, quod quidem senserim; nihil audivi ex eo ipse quod nollem; una domus erat, idem victus,? isque communis, neque solum militia sed etiam peregrinationes rusti- cationesque communes. Nam quid ego de studiis dicam cogno- scendi semper aliquid atque discendi, in quibus remoti ab oculis populi omne otiosum tempus contrivimus?? Quarum rerum recordatio et memoria si una cum illo occidisset,4 desiderium coniunctissimi atque amantissimi viri ferre nullo modo possem.

Cicero, De Amicitia, 102-104.

t Equidem, ‘for my part.’ *victus, ‘style of living.’ 3contrivimus, ‘passed.’ 4 occidisset from cado.

29

IQI2 LATIN NR4—CICERO (MANILIAN LAW AND ARCHIAS) AND SIGHT TRANSLATION OF PROSE

Thursday Q-II a.m.

Translate the following passages, and answer the questions. The translation should be exact, but in clear and natural English. It is important that enough time be given to II to deal with it satisfactorily.

j

1. Et quoniam auctoritas quoque in bellis administrandis multum atque in imperio militari valet, certe nemini dubium est quin ea re idem ille imperator plurimum possit. Vehementer autem pertinere ad bella administranda quid hostes, quid socii de imperatoribus nostris existi- ment, quis ignorat, cum sciamus homines in tantis rebus ut aut con- 5 temnant aut metuant aut oderint aut ament opinione non minus et fama quam aliqua ratione certa commoveri? Quod igitur nomen umquam in orbe terrarum clarius fuit? Cuius res gestae pares? De quo homine vos, id quod maxime facit auctoritatem, tanta et tam praeclara iudicia fecistis >—Pro Lege Manilia, 43. . 10 a) Explain briefly Cicero’s argument in this passage.

b) Of what events in Pompey’s career was Cicero thinking when he asked Cuius res gestae pares (line 8)? Mention the most important of the iudicia (line 9). |

c) What is the syntax of the clauses introduced by quid (line 4)? To what does id (line 9) refer ?

2. Carus fuit Africano superiori noster Ennius, itaque etiam in sepulcro

Scipionum putatur is esse constitutus ex marmore; cuius laudibus certe

non solum ipse qui laudatur,sed etiam populi Romani nomen ornatur.

In caelum huius proavus Cato tollitur; magnus honos populi Romani

rebus adiungitur. Omnes denique illi Maximi, Marcelli, Fulviinonsine 5

communi omnium nostrum laude decorantur. Ergo illum qui haec

fecerat, Rudinum hominem, maiores nostri in civitatem receperunt; nos

hunc Heracliensem multis civitatibus expetitum, in hac autem legibus

constitutum de nostra civitate eiciemus ?>—Pro Archia, 22.

a) What is the importance of Ennius in the history of Latin literature? What work of Ennius did Cicero have in mind when he said cuius laudi- bus ... . decorantur (lines 2-6)?

6) Tell who the following men were, and for what they were famous: Africano (line 1), Cato (line 4).

_¢) What did Cicero gain for his argument by contrasting Rudinum hominem (line 7) with Heracliensem (line 8)? What is the syntax of civitatibus

(line 8) ?

30

IT Read the passage through several times before beginning to write the translation.

Homines Graeci, quos antea nominavi, inique a suis civibus damnati atque expulsi, tamen, quia bene sunt de suis civitatibus meriti, tanta hodie gloria sunt, non in Graecia solum sed etiam apud nos atque in ceteris terris, ut eos a quibus: illi oppressi sint nemo nominet, horum calamitatem dominationi illorum omnes anteponant, Quis Karthaginiensium pluris' fuit Hannibale consilio, virtute, rebus gestis, qui unus cum tot imperatoribus nostris per tot annos de imperio et de gloria decertavit? Hunc sui cives e civitate eiecerunt; nos etiam hostem litteris nostris et memoria videmus esse celebratum. . . . . Cogitemus denique corpus virorum fortium magnorumque hominum esse mortale, animi vero motus et virtutis gloriam sempiternam; neque, hanc opinionem si in illo sanctissimo Hercule consecratam videmus cuius corpore ambusto? vitam eius et virtutem immortalitas excepisse dicatur, minus existimemus eos qui hanc tantam rem publicam suis consiliis aut laboribus aut auxerint aut defenderint aut servarint esse immortalem gloriam consecutos.—Cicero, Pro Sestio, 142, 143.

t pluris, genitive of indefinite value. 2 ambusto, from amburo, ‘burn.’

~s

31

1913 LATIN 4—CICERO (MANILIAN LAW AND ARCHIAS)

AND SIGHT TRANSLATION OF PROSE Thursday Q-II a.m.

Translate the following passages, and answer the questions. The translation should be exact, but in clear and natural English. It is important that enough time be given to- II to deal with it satisfactorily.

I

1. Age vero, illa res quantam declarat eiusdem hominis apud hostis populi Romani auctoritatem, quod ex locis tam longinquis tamque diversis tam brevi tempore omnes huic se uni dediderunt; quod Cretensium: legati, cum in eorum insula noster imperator exercitusque esset, ad Cn. Pompeium in ultimas prope terras venerunt, eique se omnis Cre- 5 tensium civitates dedere velle dixerunt? Quid? Idem iste Mithri- dates nonne ad eundem Cn. Pompeium legatum usque in Hispaniam misit, eum quem Pompeius legatum semper iudicavit, ii quibus erat molestum ad eum potissimum-esse missum speculatorem quam lega- tum iudicari maluerunt ?—Pro Lege Manilia, 46. 10

a) Account for the number of age (line 1). b) What was ila res (line 1) ?

c) What was Pompey doing in Spain at the time of the incident mentioned in the last sentence of this passage?

2. Quae cum ita sint, quid est quod de eius civitate dubitetis, praesertim cum aliis quoque in civitatibus fuerit ascriptus? Etenim cum medio- cribus multis et aut nulla aut humili aliqua arte praeditis gratuito civitatem in Graecia homines impertiebant, Reginos credo aut Locrensis aut Neapolitanos aut Tarentinos, quod scaenicis artificibus 5 largiri solebant, id huic summa ingeni praedito gloria noluisse. Quid? Ceteri non modo post civitatem datam sed etiam post legem Papiam aliquo modo in eorum municipiorum tabulas inrepserunt; hic qui ne utitur quidem illis in quibus est scriptus, quod semper se Heracliensem esse voluit, reicietur?—Pro Archia, to. 10

a) What is meant by Graecia (line 4)? Name five of its chief cities. b) Why did Cicero single out scaenici artifices (line 5) for special mention ?

c) To what did Cicero refer in civitatem datam (line 7)? What was the lex Papia?

32

II Read the passage through several times before beginning to write the translation.

[A dream and its fulfilment.]

Singulari vir ingenio Aristoteles et paene divino ipsene errat an alios vult etrare, cum scribit Eudemum Cyprium,? familiarem suum, iter in Macedoniam facientem Pheras venisse, quae erat urbs in Thessalia tum admodum? nobilis, ab Alexandro autem tyranno crudeli dominatu’ tenebatur; in eo igitur oppido ita graviter aegrum* Eudemum fuisse ut omnes medici diffiderent; ei visum in quiete’ egregia facie’ iuvenem? dicere fore ut perbrevi® convalesceret, paucis- que diebus interiturum Alexandrum tyrannum, ipsum autem quinquennio? post domum esse rediturum? Atque ita quidem prima statim scribit Aristoteles consecuta, et convaluisse Eudemum et ab uxoris fratribus interfectum tyrannum; quinto autem anno exeunte, cum esset spes ex illo somnio” in Cyprum illum ex Sicilia esse rediturum, proeliantem eum ad Syracusas occidisse;* ex quo ita illud somnium esse interpretatum, ut cum animus Eudemi e corpore excesserit, tum domum revertisse videatur.—Cicero, De Divinatione, i, 53.

tCyprium, ‘of Cyprus.’ *admodum, ‘very.’ 3dominatu, ‘rule.’ 4aegrum, ‘sick.’ 5 quiete. ‘sleep.’ ‘facie, ‘beauty.’ 7iuvenem, ‘youth.’ 8 perbrevi, equivalent to perbrevi tempore. 9% quin- quennio, used for guingue annis. *somnio, ‘dream.’ * occidisse, from cado.

33

1914 LATIN 4—CICERO (MANILIAN LAW AND ARCHIAS) AND SIGHT TRANSLATION OF PROSE

Thursday Q-II a.m.

Translate the following passages, and answer the questions. The translation should be exact, but in clear and natural English. It is important that enough time be given to II to deal with it satisfactorily.

I

Cnidum aut Colophonem aut Samum, nobilissimas urbis, innumerabilis- que alias captas esse commemorem, cum vestros portus, atque eos portus quibus vitam ac spiritum ducitis, in praedonum fuisse potestate sciatis ? An vero ignoratis portum Caietae celeberrimum ac plenissimum navium inspectante praetore a praedonibus esse direptum; ex Miseno autem eius 5 ipsius liberos qui cum praedonibus antea ibi bellum gesserat a praedonibus esse sublatos? Nam quid ego Ostiense incommodum atque illam labem atque ignominiam rei publicae querar, cum prope inspectantibus vobis classis ea cui consul populi Romani praepositus esset a praedonibus capta atque oppressa est? Pro di immortales, tantamne unius hominis incredibilis ac 10 divina virtus tam brevi tempore lucem adferre rei publicae potuit ut vos qui modo ante ostium Tiberinum classem hostium videbatis, ii nunc nullam intra Oceani ostium praedonum navem esse audiatis p—Pro Lege Manilia, 33.

1. What four things does Cicero enumerate in this speech as the chief qualifica- tions of an imperator? Which of these qualifications is he illustrating in the passage above?

2. By what law was Pompey put in command of the war with the pirates? In what year? Describe his previous military achievements.

3. Where were the places referred to in Samum (line 1), Miseno (line 5), Ostiense (line 7)? What is meant by Oceani ostium (line 13)?

4. What is the point of the contrast between Cnidum aut Colophonem aut Samum (line 1) and vestros portus (line 2)? Explain the meaning of guibus vitam ac spiritum ducitis (line 3). Of what facts is Cicero thinking when he says tam brevi tempore (line 11) ?

5. What force does am vero (line 4) give to a ne What part of Speen is pro (line 10) ? ae the use of 7 (line 12).

34

II Read the passage through several times before beginning to write the translation.

[Metellus takes steps to entrap Jugurtha.]

Metellus postquam videt neque oppidum capi, neque Iugurtham nisi ex in- sidiis aut suo loco pugnam facere, et iam aestatem exactam esse, ab Zama discedit et in iis urbibus quae ad se defecerant’ satisque munitae loco aut moenibus erant praesidia imponit; ceterum exercitum in provinciam quae proxima est Numidiae hiemandi gratia? conlocat. Neque id tempus ex aliorum more quieti aut luxuriae concedit, sed quoniam armis bellum parum procedebat, insidias regi per amicos tendere’ et eorum perfidia pro armis uti parat. Igitur Bomilcarem, qui Romae cum Jugurtha fuerat et inde vadibus‘ datis de Massivae neceS iudicium fugerat, quod ei per maximam amicitiam maxima copia fallendi erat, multis pollicitatio- nibus adgreditur, ac primo efficit ut ad se conloquendi gratia occultus® veniat; deinde fide data, si Iugurtham vivum aut necatum sibi tradidisset, fore ut illi - senatus impunitatem et sua omnia concederet, facile Numidae’ persuadet, cum® ingenio® infido tum metuenti ne, si pax cum Romanis fieret, ipse per condiciones ad supplicium traderetur.—Sallust, De Bello Iugurthino, 61.

tdefecerant, ‘had gone over.’ *hiemandi gratia, equivalent to hiemandi causa. 3tendere, ay.’ ‘4vadibus, ‘bail.’ Smnece, ‘murder.’ ‘occultus, ‘secretly.’ 7Numidae, ‘the Numidian’; Bomilcar is meant. *cum... tum, ‘both... and.’ %ingenio infido, ‘of a faithless disposi- tion.’

35

IQI5 LATIN 4—CICERO (MANILIAN LAW AND ARCHIAS) AND SIGHT TRANSLATION OF PROSE

Thursday Q-II a.m.

Translate the following passages, and answer the questions. The translation should be exact, but in clear and natural English. It is important that enough time be given to II to deal with it satisfactorily. ;

I

Qua re conservate, iudices, hominem pudore eo quem amicorum videtis comprobari cum dignitate tum etiam vetustate; ingenio autem tanto quantum id convenit existimari quod summorum hominum ingeniis expetitum esse videatis; causa vero eius modi quae beneficio legis, auctoritate municipi, testimonio Luculli, tabulis Metelli comprobetur. Quaecumitasint,petimus 5 a vobis, iudices, si qua non modo humana verum etiam divina in tantis ingeniis commendatio debet esse, ut eum qui vos, qui vestros imperatores, qui populi Romani res gestas semper ornavit, qui etiam his recentibus nostris vestrisque domesticis periculis aeternum se testimonium laudis daturum esse profitetur, estque ex eo numero qui semper apud omnis sancti sunt 10 habiti itaque dicti, sic in vestram accipiatis fidem ut humanitate vestra levatus potius quam acerbitate violatus esse videatur.—Pro Archia, 31.

1. In what year was this speech delivered? Who was the praetor before whom the case was tried? What was the charge against Archias? What was the real purpose of the prosecution ?

2. Explain the reference in /egis (line 4). State the provisions of this legis. Give the name of the municipi (line 4). Of what is Cicero thinking when he says his .. . . periculis (lines 8, 9)?

3. Where did Archias receive his early education? What advantage does Cicero find in the fact that the poems of Archias were written in Greek ? What poetical writings of Archias does Cicero refer to in this speech ?

4. For what does Cicero say that he was personally indebted to Archias? Show precisely how Cicero uses his praise of literature as an argument for the acquittal of his client.

5. What is gained by the repetition of qui (lines 7, 8)? After what other words would qua (line 6) have the same meaning that it has in this sentence ?

36

II 3 Read the passage through several times before beginning to write the translation.

[Cicero tells why he refused to act as counsel for Autronius, charged with complicity in the conspiracy of Catiline.|

_ _Veniebat enim ad me et saepe veniebat Autronius multis cum lacrimis* supplex ut se defenderem, et se meum condiscipulum? in pueritia, familiarem in adulescentia commemorabat fuisse, multa mea in se, non nulla etiam sua in me proferebat officia. Quibus ego rebus, iudices, ita flectebar animo atque frangebar ut iam ex memoria quas mihi ipse fecerat insidias deponerem, ut iam immissum esse ab eo C. Cornelium, qui me in meis sedibus, in conspectu uxorisac ~ liberorum meorum trucidaret, obliviscerer. Quae si de uno me cogitasset, num- quam illius lacrimis ac precibus’ restitissem; sed cum mihi tela, cum caedes, cum civium cruor‘ versari ante oculos coeperat, tum denique ei resistebam, neque solum illi hosti ac parricidae sed his etiam propinquis® illius Marcellis, patri et filio, quorum alter apud me parentis gravitatem, alter fili suavitatem® obtinebat; neque me arbitrabar sine summo scelere posse, quod maleficium in aliis vindicas- sem, idem in illorum socio, cum scirem, defendere.—Cicero, Pro Sulla, 18, 19.

t lacrimis, ‘tears.’ ? condiscipulum, ‘schoolmate.’ 3 precibus, ‘entreaties.’ 4 cruor, equiva- lent to sanguis. 5 propinguis, ‘relatives.’ ° suavitatem, ‘lovableness.’

37

De ide aes 5

LATIN 5

VERGIL— 4NEJ/D, BOOKS I, Il, AND IV OR VI, AND SIGHT TRANSLATION OF POETRY

39

IQII

LATIN NR5—VIRGIL (AENEID, I, Il, AND IV OR VI) AND SIGHT TRANSLATION OF POETRY

LATIN DQ—VIRGIL’S AENEID, BOOKS I-VI, AND SIGHT

Thursday

TRANSLATION OF POETRY

I.30-3.30 p. m.

Translate 1 and either 2 or 3, and answer the questions on the passages translated.

Translate also II. It is important that enough time be given to II to deal with it satis- factorily. The translation should be exact, but in clear and natural English.

I.

I

“Sanguine placastis ventos et virgine caesa,

cum primum Iliacas, Danai, venistis ad oras;

sanguine quaerendi reditus, animaque litandum

Argolica.” Volgi quae vox ut venit ad auris,

obstipuere animi, gelidusque per ima cucurrit 120 ossa tremor, cui fata parent, quem poscat Apollo.

Hic Ithacus vatem magno Calchanta tumultu

protrahit in medios; quae sint ea numina divom

flagitat. Et mihi iam multi crudele canebant

artificis scelus et taciti ventura videbant.—ii, 116-125. 125

a) From whose tale is this passage taken? Whose words does he pretend to quote in verses 116-119? Tell the story to which reference is made in virgine caesa (verse 116). Where were the J/liacas oras (verse 117)? Who is meant by Jthacus (verse 122), and why was he so called ?

b) What other form might have been used for obstipuere (verse 120), had not the metre prevented? Explain the form of divom (verse 123).

c) Copy verse 117, and indicate the division into feet and the principal caesura.

“Vixi, et quem dederat cursum fortuna peregi, et nunc magna mei sub terras ibit imago. Urbem praeclaram statui, mea moenia vidi; 655 ulta virum poenas inimico a fratre recepi; felix, heu nimium felix, si litora tantum numquam Dardaniae tetigissent nostra carinae.”’ Dixit, et os impressa toro, ‘‘ Moriemur inultae, sed moriamur,” ait. “Sic, sic iuvat ire sub umbras. 660 Hauriat hunc oculis ignem crudelis ab alto Dardanus, et nostrae secum ferat omina mortis.” —iv, 653-662.

a) Tell more fully the story touched upon in verses 655 and 656. Why is Aeneas called Dardanus (verse 662) ? b) What would regularly be used instead of mez (verse 654) ?

40

4

3. “Quisquis es, armatus qui nostra ad flumina tendis, fare age, quid venias, iam istinc, et comprime gressum. Umbrarum hic locus est, somni noctisque soporae; 390 corpora viva nefas Stygia vectare carina. Nec vero Alciden me sum laetatus euntem accepisse lacu nec Thesea Pirithoumque, dis quamquam geniti atque invicti viribus essent. Tartareum ille manu custodem in vincla petivit 395 ipsius a solio regis traxitque trementem; hi dominam Ditis thalamo deducere adorti.”—vi, 388-397.

a) Who were Alciden (verse 392), dominam Ditis (verse 397), Ditis (verse - 397)? What was the Tartareum custodem (verse 395) ? b) Explain the spelling of vincla (verse 395). How does the pronunciation of ipsius in verse 396 differ from prose usage ?

II Read the passage through several times before beginning to write the translation.

[The tranquil life of the philosopher is contrasted with the vain ambitions of other men} Suave,’ mari magno turbantibus aequora ventis, e terra magnum alterius spectare laborem; non quia vexari quemquam est iucunda voluptas, sed quibus ipse malis careas quia cernere suave est. Suave etiam belli certamina magna tueri per campos instructa tua sine parte pericli. Sed nil dulcius est, bene quam munita tenere edita? doctrina sapientum templa serena, despicere unde queas alios passimque videre errare atque viam palantis‘ quaerere vitae, certare ingenio, contendere nobilitate, noctes atque dies niti praestante labore ad summas emergere’ opes rerumque potiri. O miseras hominum mentis, O pectora caeca! Qualibus in tenebris vitae quantisque periclis degitur® hoc aevi, quodcumque est! Nonne videre? nil aliud sibi naturam latrare,’ nisi utqui? corpore seiunctus dolor absit, mensque fruatur iucundo sensu, cura semota metuque ?

Lucretius, De Rerum Natura, ii, 1-19.

*Suave, ‘sweet’; supply est. *edita, ‘lofty.’ 3gueas, from queo, ‘be able.’ 4palantis, from palor, ‘wander about.’ Semergere, ‘to raise themselves.’ ‘degitur, ‘is passed.’ tyidere, exclamatory infinitive. ‘*atrare, ‘demands.’ %wtgui, translate as if it were ut alone.

41

IgI2 LATIN NR5—VERGIL (AENEID, I, Il, AND IV OR VI) AND SIGHT TRANSLATION OF POETRY

Thursday I. 30-3.30 p. m.

Translate 1 and either 2 or 3, and answer the questions on the passages translated. Translate also II. It is important that enough time be given to II to deal with it satis- factorily. The translation should be exact, but in clear and natural English.

I

z *‘ Aspice (namque omnem, quae nunc obducta tuenti mortalis hebetat visus tibi et umida circum 605 caligat, nubem eripiam; tu ne qua parentis iussa time neu praeceptis parere recusa): hic, ubi disiectas moles avolsaque saxis saxa vides mixtoque undantem pulvere fumum, Neptunus muros magnoque emota tridenti 610 fundamenta quatit totamque a sedibus urbem eruit: hic Iuno Scaeas saevissima portas prima tenet sociumque furens a navibus agmen ferro accincta vocat. Iam summas arces Tritonia, respice, Pallas 615 insedit, nimbo effulgens et Gorgone saeva.”’—ii, 604-616

a) Under what circumstances were these words spoken? What effect was the vision here described intended to produce upon the one who was to see it ? Why did the speaker think it necessary to say tu... . recusa (verses 606-607) ?

b) Why was Juno saevissima (verse 612) to Troy? What precisely is meant by Gorgone saeva (verse 616) ?

c) Copy verses 605 and 606, and indicate the quantity of each syllable, the division into feet, and the principal caesuras.

d) Explain how it happens that so many verses in the Aeneid (as in this passage, verse 614) are not complete hexameters.

2. “Tliacas igitur classis atque ultima Teucrum

iussa sequar? Quiane auxilio iuvat ante levatos,

et bene apud memores veteris stat gratia facti?

Quis me autem, fac velle, sinet, ratibusve superbis 540

invisam accipiet? Nescis heu, perdita, necdum

Laomedonteae sentis periuria gentis ?

Quid tum? Sola fuga nautas comitabor ovantis ?

An Tyriis omnique manu stipata meorum

inferar, et quos Sidonia vix urbe revelli

rursus agam pelago et ventis dare vela iubebo ? 545

Quin morere, ut merita es, ferroque averte dolorem.”

IV, $37-547: a) Rewrite Quiane ... . levatos (verse 538), supplying in Latin the words

needed to complete the meaning. In what way does the scansion help you to determine the meaning in verse 543 ?

b) Why is Laomedonteae (verse 542) a term of reproach? Explain the refer- ence in nautas ovantis (verse 543), Sidonia urbe (verse 545).

42

< Tum pater Anchises lacrimis ingressus obortis: ““O gnate, ingentem luctum ne quaere tuorum. Ostendent terris hunc tantum fata, neque ultra esse sinent. Nimium vobis Romana propago 870 visa potens, Superi, propria haec si dona fuissent. Quantos ille virum magnam Mavortis ad urbem Campus aget gemitus, vel quae, Tiberine, videbis funera, cum tumulum praeterlabere recentem! Nec puer Iliaca quisquam de gente Latinos 875 in tantum spe tollet avos, nec Romula quondam ullo se tantum tellus iactabit alumno.”—vi, 867-877.

a) Of what recital does this passage form a part? Who is meant by hunc (verse 869) ? |

b) Explain the reference in Mavortis (verse 872), Campus (verse 873). c) Rewrite ne quaere (verse 868) in the form which is most common in prose.

II Read the passage through several times before beginning to write the translation

O genus attonitum gelidae formidine mortis! Quid Styga, quid tenebras et nomina vana timetis, materiem! vatum, falsique pericula mundi? ? Corpora sive rogus flamma seu tabe vetustas abstulerit, mala posse pati non ulla putetis. Morte carent animae, semperque priore relicta sede novis domibus vivunt habitantque receptae. Omnia mutantur, nihil interit.4 Errat et illinc huc venit, hinc illuc, et quoslibets occupat artus spiritus; eque feris humana in corpora transit, inque feras noster, nec tempore deperit® ullo. Ipsa quoque adsiduo labuntur tempora motu non secus ac flumen. Neque enim consistere flumen, nec levis hora potest; sed ut unda impellitur unda, urgeturque eadem veniente urgetque priorem, tempora sic sis eae pariterque sequuntur, et nova sunt semp Nam quod fuit ante relictum est, fitque quod haud fuerat, momentaque cuncta novantur. Ovid, Metamorphoses, xv, 153-185. t materiem, ‘theme.’ ? mundi, ‘world.’ 3iabe, ‘decay.’ 4 inierit, from intereo, synonymous with pereo. § quoslibet, indefinite pronoun, ‘whateverit will.’ °deperit, synonymous with inierit.

43

Thursday

TOT3

LATIN 5—VERGIL (AENEID, I, Il, AND IV OR VI) AND SIGHT TRANSLATION OF POETRY

2-4 p..m.

Translate 1 and either 2 or 3, and answer the questions on the passages translated.

Translate also II. It is important that enough time be given to II to deal with it satis- factorily. The translation should be exact, but in clear and natural English.

I

-Haec dum Dardanio Aeneae miranda videntur, dum stupet obtutuque haeret defixus in uno, 495 regina ad templum, forma pulcherrima Dido, incessit magna iuvenum stipante caterva. Qualis in Eurotae ripis aut per iuga Cynthi exercet Diana choros, quam mille secutae hinc atque hinc glomerantur Oreades (illa pharetram 500 fert umero, gradiensque deas supereminet omnis; Latonae tacitum pertemptant gaudia pectus), cat talis erat Dido, talem se laeta ferebat

_ per medios, instans operi regnisque futuris.

Tum foribus divae, media testudine templi, 505 saepta armis solioque alte subnixa, resedit.

Iura dabat legesque viris, operumque laborem

partibus aequabat iustis aut sorte trahebat.—i, 494-508.

a) What had caused the feeling described in the first two verses of this passage ?

b) What are the two important elements in this description of Dido? Why was Aeneas already disposed to sympathize with her? c) Where were the iuga Cynthi (verse 498) ?

d) Who were the Oreades (verse 500) ?

e) Copy verses 494 and 495, and indicate the quantity of each syllable, the division into feet, and the principal caesuras. f) What place in the temple is indicated by the words of verse 505 ?

“Tuppiter omnipotens, cui nunc Maurusia pictis gens epulata toris Lenaeum libat honorem, aspicis haec? An te, genitor, cum fulmina torques, nequiquam horremus, caecique in nubibus ignes terrificant animos et inania murmura miscent ? 210 Femina, quae nostris errans in finibus urbem exiguam pretio posuit, cui litus arandum cuique loci leges dedimus, conubia nostra reppulit ac dominum Aenean in regna recepit.’’ iv, 206-214.

a) By whom were these words spoken, and under what circumstances? What effect did they have ?

b) Explain the allusion in Lenaeum (verse 207).

c) In verses 209-210 what three words carry the weight of the argument?

d) Tell the story alluded to in pretio (verse 212).

3. - “Phoebe, gravis Troiae semper miserate labores, Dardana qui Paridis direxti tela manusque corpus in Aeacidae, magnas obeuntia terras tot maria intravi duce te penitusque repostas Massylum gentis praetentaque Syrtibus arva; 60 iam tandem Italiae fugientis prendimus oras. Hac Troiana tenus fuerit Fortuna secuta. Vos quoque Pergameae iam fas est parcere genti, dique deaeque omnes quibus obstitit Ilium et ingens gloria Dardaniae.”—vi, 56-65. a) Why did Vergil write Dardana (verse 57) agreeing with éela, rather than Dardani agreeing with Paridis? b) Write the usual form of direxti (verse 57). c) Of what is Aeneas thinking when he uses the words tandem and fugientis (verse 61) ? 3 d)- Explain the allusion in Aeacidae (verse 58).

II Read the passage through several times before beginning to write the translation.

[Anna tells Aeneas the details of Dido’s death.]

Tum rapido praeceps cursu resolutaque crinem _ evasit propere’ in celsam, quam struxerat ante magna mole, pyram; cuius de sede dabatur cernere cuncta freta et totam Carthaginis urbem. “Di longae noctis, quorum iam numina nobis mors instans maiora facit, precor,’’ inquit, “adeste et placidi victos ardore admittite Manis. Aeneae coniunx, Veneris nurus, ulta maritum, vidi constructas nostrae Carthaginis arces. Nunc ad vos magni descendet corporis umbra.” Haec dicens ensem media in praecordia? adegit, ensem Dardanii quaesitum in pignus amoris. Viderunt comites tristique per atria planctu concurrunt; magnis resonant ululatibus aedes. Accepi infelix dirisque exterrita fatis, ora manu lacerans, lymphato‘ regia cursu tecta peto celsosque gradus evadere nitor. Ter diro fueram conata incumbere ferro, ter cecidi exanimae membris revoluta sororis. , Silius Italicus, Punica, viii, 130-156.

* propere, ‘inhaste.’ * praecordia, equivalent to pectus. 3 planctu, ‘lamentation.’ 4lymphato, -frantic.’ i 45

1914 LATIN 5—VERGIL (AENEID, I, Il, AND IV OR VI) AND SIGHT TRANSLATION OF POETRY

Thursday 2-4 p.m.

Under I, translate either 1 or 2, and answer the questions on the passage translated. Translate also II. It is important that enough time be given to II to deal with it satis- factorily. The translation should be exact, but in clear and natural English.

I

x “‘Quae quibus anteferam? Jam iam nec maxuma Iuno, nec Saturnius haec oculis pater aspicit aequis. Nusquam tuta fides. Eiectum litore, egentem excepi et regni demens in parte locavi; amissam classem, socios a morte reduxi. 375 Heu furiis incensa feror! Nunc augur Apollo, nunc Lyciae sortes, nunc et Iove missus ab ipso interpres divom fert horrida iussa per auras. Scilicet is superis labor est, ea cura quietos sollicitat. Neque te teneo, neque dicta refello; 380 i, sequere Italiam ventis, pete regna per undas. Spero equidem mediis, si quid pia numina possunt, supplicia hausurum scopulis, et nomine Dido saepe vocaturum. Sequar atris ignibus absens, et, cum frigida mors anima seduxerit artus, 385 omnibus umbra locis adero. Dabis, improbe, poenas. Audiam, et haec Manis veniet mihi fama sub imos.”’

Iv, 371-387.

a) Under what circumstances were these words spoken ?

b) State two reasons why it is natural for Dido to think first of Juno (verse 371). How does the goddess show her pity at the close of Book IV ?

c) Explain the reference in Saturnius (verse.372), Lyciae sortes (verse 377), interpres divom (verse 378). Where was Lycia? d) Describe the scene in Book I to which the words socios a morte reduxi (verse 375) refer. To what does amissam refer? e) What part does the guidance of Apollo (verse 376) play in the life of Aeneas ? What is meant by is (verse 379) ? f) Explain the meaning of pia in connection with numina (verse 382)? Givea literal translation of hausurum (verse 383) and explain the metaphor. g) Copy verse 373 and 383, and indicate the quantity of each syllable, the division into feet, and the principal caesuras. a. “Hic vir, hic est, tibi quem promitti saepius audis, Augustus Caesar, divi genus, aurea condet saecula qui rursus Latio regnata per arva Saturno quondam; super et Garamantas et Indos proferet imperium (iacet extra sidera tellus, 795 extra anni solisque vias, ubi caelifer Atlas axem umero torquet stellis ardentibus aptum). Huius in adventum iam nunc et Caspia regna responsis horrent divom et Maeotia tellus,

46

et septemgemini turbant trepida ostia Nili. 800 Nec vero Alcides tantum telluris obivit, fixerit aeripedem cervam licet, aut Erymanthi pacarit nemora, et Lernam tremefecerit arcu, nec, qui pampineis victor iuga flectit habenis, Liber, agens celso Nysae de vertice tigris. 805 Et dubitamus adhuc virtutem extendere factis, aut metus Ausonia prohibet consistere terra ?”’ vi, 791-807.

a) Under what circumstances were these words spoken ?

b) Explain the reference in divi (verse 792), Saturno (verse 794), Atlas (verse 796).

c) What and where were Latio (verse 793), Caspia regna (verse 798), Ausonia terra (verse 807) ?

d) In what respect is the order of the words aurea condet saecula qui rursus (verses 792-793) poetical? Name and explain the figure of speech used in verse 799. Explain the form divom (verse 799).

e) Account for Vergil’s attitude toward Augustus as indicated in this passage. What is the poet’s conception of the mission of Rome? What revelation of the future was granted to Aeneas in the lower world?

f) How is the thought expressed in verses 806-807 related to the verses that precede ?

g) Copy verses 796 and 797, and indicate the quantity of each syllable, the division into feet, and the principal caesuras.

II Read the passage through several times before beginning to write the translation.

[The Trojans mourn Hector. King Priam begs Achilles to give him the body of his son.]

Flent miseri amissum Phryges Hectora, totaque maesto Troia sonat planctu.t Fundit miseranda querelas? infelix Hecube? saevisque arat unguibus* ora, Andromacheque suas scindit de pectore vestes, heu tanto spoliata viro! Ruit omnis in uno Hectore causa Phrygum; ruit et defessa senectus adflicti miseranda patris, quem nec sua coniunx turbaque natorum nec magni gloria regni

oblitum tenuit vitae, quin iret inermis

et solum invicti castris se redderet hostis. Mirantur Danaum proceres, miratur et ipse Aeacides animum miseri senis. Ille trementes, adfusus® genibus, tendens ad sidera palmas

haec ait: “O Graiae gentis fortissime Achilles,

O regnis inimice meis, te Dardana solum

victa tremit pubes, te sensit nostra senectus crudelem nimium. Nunc sis mitissimus,° oro,

et patris adflicti genibus miserere precantis donaque quae porto miseri pro corpore nati accipias.”’—Jlias Latina, 1015-1034.

t planctu, ‘lamentation.’ ?querelas, ‘complaints.’ 3Hecube, equivalent to Hecuba. 4un- guibus, ‘nails.’ Sadfusus, ‘casting himself down.’ 6 mitissimus, equivalent to lenissimus.

47

IgI-

LATIN 5—VERGIL (AENEID, I, Il, AND IV OR VI‘ AND SIGHT TRANSLATION OF POETRY

Thursday 2-4 }

Translate the following passages, and answer the questions. The translation shc

be exact, but in clear and natural English. It is important that enough time be gi: II to deal with it satisfactorily.

oOo on nun +

I

“His ego nec metas rerum nec tempora pono, imperium sine fine dedi. Quin aspera Iuno, quae mare nunc terrasque metu caelumque fatigat, 280 consilia in melius referet mecumque fovebit Romanos, rerum dominos, gentemque togatam. Sic placitum. Veniet lustris labentibus aetas cum domus Assaraci Phthiam clarasque Mycenas servitio premet ac victis dominabitur Argis. 285 Nascetur pulchra Troianus origine Caesar, imperium Oceano, famam qui terminet astris, Iulius, a magno demissum nomen Iulo. : Hunc tu olim caelo, spoliis Orientis onustum, accipies secura; vocabitur hic quoque votis. 290 Aspera tum positis mitescent saecula bellis; cana Fides et Vesta, Remo cum fratre Quirinus iura dabunt; dirae ferro et compagibus artis claudentur Belli portae; Furor impius intus saeva sedens super arma et centum vinctus aenis 205 post tergum nodis fremet horridus ore cruento.”’ i, 278-206.

. By whom were these words spoken? To whom? In what connection ?

. Discuss verses 278-282 and 286-288 with reference to the purpose of the

Aeneid.

. To what historical event do verses 284, 285 refer? Explain the references

in Assaraci, Phthiam, Mycenas.

. Who were Caesar (verse 286), Iulo (verse 288), Quirinus (verse 292) ?

. What is the point of aspera (verse 279), Troianus (verse 286) ?

. What does Vergil mean by Oceano in verse 287 ?

. How was the prophecy caelo accipies (verses 289, 290) fulfilled ?

. Explain the allusion in claudentur Belli portae (verse 294).

. Copy verse 287, and indicate the quantity of each syllable, the division into

feet, and the principal caesura.

48

os

II Read the passage through several times before beginning to write the translation. [The captive Trojan women say their sad farewell to home and country.|

Ilion ardebat, neque adhuc consederat ignis; bine exiguumque? senis Priami [ovis ara cruorem Fd - combiberat. Tractata? comis antistita? Phoebi non profecturas* tendebat ad aethera palmas. Dardanidas matres patriorum signa’ deorum, dum licet, amplexas succensaque templa tenentis invidiosa trahunt victores praemia Grai. Mittitur® Astyanax illis de turribus unde pugnantem pro se proavitaque’ regna tuentem saepe videre patrem monstratum a matre solebat. Iamque viam suadet Boreas, flatuque® secundo _ carbasa? mota sonant. . Iubet uti navita™ ventis. “Troia, vale! Rapimur”’ clamant, dant oscula terrae Troades, et patriae fumantia tecta relinquunt. Ultima conscendit classem, miserabile visu, in mediis Hecuba natorum inventa sepulcris. Prensantem tumulos atque ossibus oscula dantem Dulichiae™ traxere manus. Tamen unius hausit” inque sinu cineres secum tulit Hectoris haustos. Ovid, Metamorphoses, xiii, 408-426.

exiguum, ‘scanty,’ ?tractata, frequentative, or intensive, from iraho. 3 antistita, ‘priest- ess.’ 4 profecturas, from proficio, ‘avail.’ 5 signa, equivalent to simulacra. ° mittitur, equiva- lent to deicitur. 7 proavita, ‘of his forefathers.’ 8 flatu, equivalent to vento. 9% carbasa, equivalent to vela. *™ navita, equivalent to nauta. Dulichiae, adjective, ‘of Ulysses.’ * hausit, ‘scraped up.’

?

49

Tu we AS re ot ATS f y : as

.

es

a,

PROSE COMPOSITION

IQII LATIN NR6—ADVANCED PROSE COMPOSITION 7 LATIN L—PROSE COMPOSITION

Friday II.1I5 a. M.—-12.30 p. m. Translate into Latin:

After I reached Capua, I had several conversations with Autronius. He begged me with tears to defend him, saying that we had served the state together as quaestors. He mentioned? very many kindnesses which I had done to him, some also which he had done to me. By these words I was so moved that I almost forgot that Cornelius had been sent by him to murder me in my own house in the sight of my wife and children. If he had formed these plans with reference to? myself alone, I should never have been able to resist his entreaties. But when I pictured’ to myself the wretched lot of the city if Catiline had gotten possession of it, then I did resist not only him but also his kinsmen, the Marcelli, both father and son, although I enjoyed4 the friendship of both. |

**Mention,’ proferre. 2 ‘with reference to,’ de. 3 ‘picture,’ proponere. 4 ‘enjoy,’ uti.

52

1912

LATIN NR6—ADVANCED PROSE COMPOSITION

Friday II.15 a. M.-12.30 p. m. Translate into Latin:

There are some, however, who think that Pompey ought not to be put in command of this war, who seem to fear that, if so much power be given to one citizen, the state may suffer harm. Have these men forgotten the war with the pirates? For many years we, who used to be able to go safely from Rome to any part of the world, were unable to defend our own harbors. If Pompey had then remained at Rome as a private citizen, should we now control? the whole sea? Recall how quickly he finished that shameful war. He did not even wait _ until the sea was fit for navigation, but at once sent ships in all directions to help the allies of the Roman people. Do not doubt that Pompey will so wage this war also that we shall all enjoy the advantages? of his victory,

t ‘control,’ obtinere. ? ‘advantage,’ commodum.

53

1913 LATIN 6—ADVANCED PROSE COMPOSITION

Friday II.I5 a.M.-I p. m.

Translate into Latin:

If any one should ask you, fellow-citizens, why this war is so serious, would you not tell him that it must be waged because the reputation of the Roman people is at stake? Our ancestors, when it was reported to them that their en- voys had been discourteously* treated? at Corinth, determined to destroy that city, although it was then the most beautiful city in all Greece. We, however, for many years have suffered a king to go unpunished who.on a single day put to death eighty thousand Roman citizens. Our allies are at length beginning to fear that we cannot protect them. Do not think that you can wait until they openly ask you to appoint Pompey commander. Let us rather send to them without delay the only general who can conquer Mithridates, in order that no one hereafter? may dare to injure the friends of the Roman people.

*‘discourteously,’ superbe. ? ‘treat,’ éractare. 3 ‘hereafter,’ posthac.

54

1914 LATIN 6—ADVANCED PROSE COMPOSITION Friday II.15 a.M.-I p.m. Translate into Latin:

Do you remember what Cicero said in the last part of the speech which he _ delivered? for the Manilian Law when he was praetor? He promised Manilius that he would use all the ability that he had to defend the honor of the state and the safety of its allies. There were some to whom Cicero seemed to be putting? his own interests above? the welfare of his country. These men thought that he was praising Pompey because the latter had so much influence with the people. Cicero himself, however, was afraid that, in urging the Romans to put all their hopes in Pompey alone, he had incurred the hatred of many powerful citizens. But although this was so, he did not hesitate to say that no one could prevent him from advocating? the plan which he believed to be the best for the republic. If he had not done this, would he now be worthy of our praise ?

t ‘deliver,’ habere. ‘put above,’ praeferre. 4 ‘advocate,’ suadere.

55

1915 LATIN 6—ADVANCED PROSE COMPOSITION

Thursday II.I5 a.m.-1I p.m.

Translate into Latin:

I see, fellow-citizens, that all of you understand how much praise Lucullus ought always to receive from the Roman people. Within a short time after he had set out from Rome for Asia, he not only prevented Mithridates from getting possession of Cyzicus, a city most friendly to us, but also, although the forces of the enemy were very large, so defeated the king that the latter was com- pelled to retreat with the greatest speed. _ In fact, Lucullus would have captured him if our soldiers had not been so eager for gold and silver. For Mithridates, fearing that he would not be able to escape from their hands, ordered his men to leave behind enough booty to delay the pursuers for a few days. He thus reached Armenia before anyone could overtake him. But he was not long without assistance. For Tigranes thought that he ought to help his father-in-law,' because, since he was himself a king, he pitied the misfortunes of other kings.

* ‘father-in-law,’ socer, socert.

56

LATIN B

CASAR— GALLIC WAR, BOOKS I-IV

57

IQII LATIN B—CAESAR Wednesday Q-II a.m. Translate four of the following passages, and answer the questions on the passages chosen. The translation should be exact, but in clear and natural English.

1. Neque homines inimico animo data facultate per provinciam itineris faci- undi temperaturos ab iniuria et maleficio existimabat. Tamen, ut spatium intercedere posset dum milites quos imperaverat convenirent, legatis respondit diem se ad deliberandum sumpturum: si quid vellent, ad Id. April. reverterentur. I, 7.

a) Tell in what case animo (line 1) is, and why this case is used.

b) Write in Latin an alternative construction for itineris faciundi (line 1).

c) Tellin what mood and tense convenirent (line 3) is, and why this mood and this tense are used.

d) Change to the direct form vellent (line 4) and reverterentur (line 5).

e) Conjugate reverterentur (line 5) in the perfect indicative.

2. Ubi vineis actis aggere exstructo turrim procul constitui viderunt, primum irridere ex muro atque increpitare vocibus, quod tanta machinatio a tanto spatio institueretur: Quibusnam manibus aut quibus viribus praesertim homines tantulae staturae tanti oneris turrim in muro sese collocare posse confiderent? IIT, 30.

a) Tellin what mood institueretur (line 3) is, and why this mood is used. b) Decline oneris (line 4) in the singular.

c) Tell in what case oneris (line 4) is, and why this case is used.

d) Restore confiderent (line 5) to the direct form.

e) Conjugate confiderent (line 5) in the future perfect indicative.

3. Sabinus idoneo omnibus rebus loco castris sese tenebat, cum Viridovix contra eum duorum milium spatio consedisset cotidieque productis copiis pugnandi potestatem faceret, ut iam non solum hostibus in contemptionem Sabinus veniret, sed etiam nostrorum militum vocibus non nihil carperetur; tantam- que opinionem timoris praebuit, ut iam ad vallum castrorum hostes accedere auderent. ITI, 17.

a) Compare idoneo (line 1).

b) Decline duorum (line 2) in all genders.

c) Tellin what case spatio (line 2) is, and why this case is used. d) Explain the derivation of potestatem (line 3).

e) Write the principal parts of auderent (line 6).

58

4. Cum paulo longius a castris processisset, suos ab hostibus premi atque aegre sustinere et conferta legione ex omnibus partibus tela coici animadvertit. Nam quod omni ex reliquis partibus demesso frumento pars una erat reli- qua, suspicati hostes huc nostros esse venturos noctu in silvis delituerant. IV, 32.

a) If cum (line 1) were changed to postquam, what would processisset (line 1) become ?

b) Compare aegre (line 1).

c) Write the first person singular of the present indicative of the following verbs: coici (line 2); suspicati (line 4).

d) Write the genitive and dative singular of pars wna (line 3).

e) What adverb might be substituted for ex omnibus partibus (line 2) ?

5. Ex his omnibus longe sunt humanissimi qui Cantium incolunt, quae regio est maritima omnis, neque multum a Gallica differunt consuetudine. Inte- riores plerique frumenta non serunt, sed lacte et carne vivunt pellibusque sunt vestiti. Omnes vero se Britanni vitro inficiunt, quod caeruleum efficit colorem. V, 14.

a) Substitute in Latin another possible construction for ex his omnibus (line 1).

b) Explain-the derivation of consuetudine (line 2).

c) Where was Cantium (line 1) ?

d) Conjugate differunt (line 2) in the pluperfect indicative active.

e) Under what nominatives in a vocabulary would one find Jacte (line 3) and carne (line 3) ?

6. Tum praemissis paulum impedimentis atque in tumulo quodam conlocatis, ‘habetis,’ inquit ‘milites, quam petistis facultatem: hostem impedito atque iniquo loco tenetis; praestate eandem nobis ducibus virtutem, quam saepe numero imperatori praestitistis, atque illum adesse et haec coram cernere existimate.’ VI, 8.

a) Write in indirect discourse after dixit, “‘Habetis, milites, quam petistis faculiatem” (line 2).

b) Conjugate petistis (line 2) in the future indicative active.

c) Account for the omission of the preposition im in the phrase impedito

. . loco (line 2).

d) Explain the composition of iniquo (line 3).

¢) Conjugate existimate (line 5) in the present tense, active and passive, of the mood in which the form is here found.

59

IQI2 7 LATIN B—CAESAR Wednesday Q-II a.m.

Translate three of the following passages, and answer the questions on the passages chosen. The translation should be exact, but in clear and natural English.

1. Bello Helvetiorum confecto totius fere Galliae legati, principes civitatum, ad Caesarem gratulatum convenerunt: intellegere sese, tametsi pro veteribus iniuriis populi Romani ab his poenas bello repetisset, tamen eam rem non minus ex usu terrae Galliae quam populi Romani accidisse, propterea quod eo consilio florentissimis rebus domos suas Helvetii 5 reliquissent, uti toti Galliae bellum inferrent imperioque potirentur locumque domicilio ex magna copia deligerent, quem ex omni Gallia opportunissimum ac fructuosissimum iudicassent, reliquasque civi- tates stipendiarias haberent.—i, 30, 1-3. a) Write in Latin an alternative construction for gratulatum (line 2).

b) Tell in what case each of the following words is, and why this case is used: bello (line 3), rebus (line 5), ¢mperio (line 6), domicilio (line 7).

c) Conjugate potirentur (line 6) in the present subjunctive.

d) Change to the direct form intellegere sese (line 2).

e) Tell in what mood and tense each of the following verbs is, and why this mood and this tense are used: repetisset (line 3), accidisse (line 4).

2. Hi novissimos adorti et multa milia passuum prosecuti magnam multi- tudinem eorum fugientium conciderunt, cum ab extremo agmine, ad quos ventum erat, consisterent fortiterque impetum nostrorum sus- tinerent, priores, quod abesse a periculo viderentur neque ulla necessi- tate neque imperio continerentur, exaudito clamore perturbatis ordini- 5 bus omnes in fuga sibi praesidium ponerent. Ita sine ullo periculo tantam eorum multitudinem nostri interfecerunt quantum fuit diei spatium; sub occasum solis sequi destiterunt seque in castra, ut erat imperatum, receperunt.—ii, 11, 4-6.

a) Tellin what case each of the following words is, and why this case is used: milia (line 1), sibi (line 6).

b) Write the first person singular present indicative active of conciderunt (line 2), marking the quantity of the penult and indicating the accent. Write the first person singular present indicative active of destiterunt (line 8).

c) Account for the number and the gender of quos (line 3).

d) Tell in what mood and tense each of the following verbs is, and why this mood and this tense are used: viderentur (line 4), ponerent (line 6).

e) What is the prefix in perturbatis (line 5), and what does it denote?

3. His de rebus Caesar certior factus et infirmitatem Gallorum veritus, quod sunt in consiliis capiendis mobiles et novis plerumque rebus student, nihil his committendum existimavit. Est enim hoc Gallicae consuetudinis, uti et viatores etiam invitos consistere cogant, et quid quisque eorum de quaque re audierit aut cognoverit quaerant, et mercatores in oppidis_ 5 vulgus circumsistat quibusque ex regionibus venient quasqua ibi res cognoverint pronuntiare cogat.—iv, 5, 1-2. a) Explain the derivation of mobiles (line 2), viatores (line 4), and tell in

each case what the suffix denotes.

60

b) Decline certior (line 1) in the neuter singular.

c) Tellin what case each of the following words is, and why this case is used: capiendis (line 2), rebus (line 2), consuetudinis (line 3).

d) Write the principal parts of student (line 2), cogant (line 4).

e) Tell in what mood and tense each of the following verbs is, and why this | mood and this tense are used: sunt (line 2), cogant (line 4), cognoverint

(line 7).

4. Dumnorix omnibus primo precibus petere contendit ut in Gallia relin- queretur, partim quod insuetus navigandi mare timeret, partim quod religionibus impediri sese diceret. Posteaquam id obstinate sibi negari vidit, omni spe impetrandi adempta principes Galliae sollicitare, sevo- care singulos hortarique coepit uti in continenti remanerent; metu' 5 territare: non sine causa fieri ut Gallia omni nobilitate spoliaretur; id esse concilium Caesaris, ut quos in conspectu Galliae interficere veretur, hos omnes in Britanniam traductos necaret.—v, 6, 3-5.

a) Change to the direct form non sine causa fiert ut Gallia omni nobilitate spoliaretur (line 6). |

b) Tell in what case each of the following words is, and why this case is used: navigandi (line 2), religionibus (line 3), nobilitate (line 6).

c) If primus were substituted for primo (line 1), what change would occur in the meaning of the sentence ?

d) Tell in what mood and tense each of the following verbs is, and why this mood and this tense are used: timeret (line 2), éerritare (line 6), spoliaretur (line 6).

e) What motives caused Caesar to invade Britain?

5. In omni Gallia eorum hominum qui aliquo sunt numero atque honore genera sunt duo; nam plebes paene servorum habetur loco, quae nihil audet per se, nulli adhibetur consilio. Plerique, cum aut aere alieno aut magnitudine tributorum aut iniuria potentiorum premuntur, sese in servitutem dicant nobilibus; quibus in hos eadem omnia sunt iura 5 quae dominis in servos. Sed de his duobus generibus alterum est druidum, alterum equitum. Illi rebus divinis intersunt, sacrificia publica ac privata procurant.—vi, 13, 1-4.

a) Decline duo (line 2).

b) Tell in what case each of the following words is, and why this case is used: numero (line 1), loco (line 2), dominis (line 6), druidum (line 7).

c) Write the nominative singular of genera (line 2), aere (line 3); the nomina- tive singular masculine of potentiorum (line 4).

d) Decline plebes (line 2) in the singular. Write the principal parts of dicant (line 5). |

e) Why is ili (line 7) used rather than hi ?

61

LATIN B—CAESAR

TQI3

Wednesday Q-II a.m.

Translate three of the following passages, and answer the questions on the passages

chosen. The translation should be exact, but in clear and natural English.

1. Horum adventu tanta rerum commutatio est facta ut nostri, etiam qui

vulneribus confecti procubuissent, scutis innixi proelium redintegrarent, calones perterritos hostes conspicati etiam inermes armatis occurrerent, equites vero, ut turpitudinem fugae virtute delerent, omnibus in locis pugnando se legionariis militibus praeferrent. At hostes etiam in 5 extrema spe salutis tantam virtutem praestiterunt ut, cum primi eorum cecidissent, proximi iacentibus insisterent atque ex eorum corporibus pugnarent; his deiectis et coacervatis cadaveribus, qui superessent, ut ex tumulo, tela in nostros conicerent et pila intercepta remitterent. —ii, 27, 1-4. a) Write the first person singular, present indicative of innixi (line 2), praestiterunt (line 6), cecidissent (line 7). b) Tell in what case each of the following words is, and why this case is used: vulneribus (line 2), scutis (line 2), pugnando (line 5). c) Explain the derivation of inermes (line 3), turpitudinem (line 4). d) Tellin what mood and tense each of the following verbs is, and why this mood and this tense are used: procubuissent (line 2), delerent (line 4). e) Conjugate procubuissent (line 2) in the future indicative active.

2. Prima luce productis omnibus copiis, duplici acie instituta, auxiliis in

mediam aciem coniectis quid hostes consili caperent exspectabat. TIili

etsi propter multitudinem et veterem belli gloriam paucitatemque nos-

trorum se tuto dimicaturos existimabant, tamen tutius esse arbitraban-

tur obsessis viis, commeatu intercluso sine vulnere victoria potiri, et si

propter inopiam rei frumentariae Romani sese recipere coepissent, impe-

ditos in agmine et sub sarcinis infirmiores animo adoriri cogitabant.

Hoc consilio probato ab ducibus, productis Romanorum copiis sese

castris tenebant.— iii, 24, 1-4.

a) Write the nominative singular neuter of duplici (line 1), the nominative singular of sarcinis (line 7); write the first person singular, present indicative active of obsessis (line 5).

b) Tell in what case each of the following words is, and why this case is used: consili (line 2), tutius (line 4), victoria (line 5), animo (line 7).

c) Distinguish in meaning acie (line 1) and agmine (line 7).

d) Tell in what mood and tense each of the following verbs is, and why this mood and this tense are used: caperent (line 2), coepissent (line 6).

e) Conjugate potiri (line 5) in the imperfect subjunctive.

3. Ad haec Caesar quae visum est respondit; sed exitus fuit orationis: sibi

nullam cum iis amicitiam esse posse, si in Gallia remanerent; neque

verum esse qui suos fines tueri non potuerint alienos occupare; neque 62

ullos in Gallia vacare agros qui dari, tantae praesertim multitudini, sine iniuria possint; sed licere, si velint, in Ubiorum finibus considere, quorum sint legati apud se et de Sueborum iniuriis querantur et a se auxilium petant; hoc se Ubiis imperaturum.—iv, 8.

5

a) Change to the direct form s7bt nullam cum tis amicitiam esse posse, si in

Gallia remanerent (lines 1-2).

b) Tell in what case each of the following words is, and why this case is

used: quae (line 1), sibi (line 1), hoc (line 7). _- c) Write the principal parts of tweri (line 3), considere (line 5). d) When is wllus used in Latin? Decline the word in the singular.

e) Tell in what mood and tense each of the following verbs is, and why this

mood and this tense are used: possint (line 5), considere (line 5). 4. Tum demum Titurius, qui nihil ante providisset, trepidare et concursare cohortesque disponere, haec tamen ipsa timide atque ut eum omnia deficere viderentur,—quod plerumque iis accidere consuevit qui in ipso negotio consilium capere coguntur. At Cotta, qui cogitasset haec posse in itinere accidere atque ob eam causam profectionis auctor non fuisset, nulla in re communi saluti deerat et in appellandis cohortandisque mili- tibus imperatoris et in pugna militis officia praestabat.—v, 33, 1-2. a) How does concursare (line 1) differ in meaning from concurrere ?

b) Tell in what case each of the following words is, and why this case is

used: Titurius (line 1), saluti (line 6), appellandis (line 6).

c) What is the force of the prefix in disponere (line 2)? Explain the deriva-

tion of auctor (line 5), and tell what the suffix denotes.

d) Tell in what mood and tense each of the following verbs is, and why this mood and this tense are used: providisset (line 1), viderentur (line 3). e) Conjugate capere (line 4) in the imperfect subjunctive active. Mark the

quantities of all the vowels in accidere (line 5).

5. Loquitur consulto palam, quoniam Germani appropinquare dicantur, sese suas exercitusque fortunas in dubium non devocaturum et postero die prima luce castra moturum. Celeriter haec ad hostes deferuntur, ut ex magno Gallorum equitum numero non nullos Gallicis rebus favere natura cogebat. Labienus nocte tribunis militum primisque ordinibus convocatis quid sui sit consili proponit, et quo facilius hostibus timoris det suspicionem, maiore strepitu et tumultu quam populi Romani fert consuetudo, castra moveri iubet. His rebus fugae similem profectionem efficit.—vi, 7, 6-8.

5

a) Tell in what case each of the following words is, and why this case is

used: rebus (line 4), consili (line 6), fugae (line 8).

b) Give the reason for the use of guo (line 6) to introduce the clause quo

crate + GCF. c) Who are referred to in primis ordinibus (lines 5-6) ? d) Compare celeriter (line 3), similem (line 9)

¢) Tell in what mood and tense each of the following verbs is, and why this

mood and this tense are used: dicantur (line 1), sit (line 6). 63

LATIN B—CAESAR

1914

Wednesday Q-II a.m.

Translate three of the following passages, and answer the questions on the passages

chosen. The translation should be exact, but in clear and natural English.

1. Ubi vero turrim moveri et appropinquare moenibus viderunt, nova atque inusitata specie commoti legatos ad Caesarem de pace miserunt, qui ad hunc modum locuti: non se existimare Romanos sine ope divina bellum gerere, qui tantae altitudinis machinationes tanta celeritate promovere possent; se suaque omnia eorum potestati permittere dixerunt. Unum petere ac deprecari: si forte pro sua clementia ac mansuetudine, quam ipsi ab aliis audirent, statuisset Aduatucos esse conservandos, ne se armis despoliaret. Sibi omnes fere finitimos esse inimicos ac suae virtuti invidere; a quibus se defendere traditis armis non possent.—ii, 31.

a) Change to the direct form se swaque omnia eorum potestati permittere

(line 5).

b) Tell in what case each of the following words is, and why this case is

used: altitudinis (line 4), celeritate (line 4), potestati (line 5).

c) Write the principal parts of miserunt (line 2), locuti (line 3), invidere

(line 9). d) Decline moenibus (line 1), ope (line 3).

e) Tell in what mood each of the following verbs is, and why this mood is used: possent (line 5), audirent (line 7), despoliaret (line 8). Tell in

what tense possent (line 5) is, and why this tense is used.

2. Compluribus expugnatis oppidis Caesar, ubi intellexit frustra tantum laborem sumi neque hostium fugam captis oppidis reprimi neque his noceri posse, statuit exspectandam classem. Quae ubi convenit ac primum ab hostibus visa est, circiter CCxx naves eorum paratissimae atque omni genere armorum ornatissimae profectae ex portu nostris adversae constiterunt; neque satis Bruto, qui classi praeerat, vel tribunis militum centurionibusque, quibus singulae naves erant attributae, constabat quid agerent aut quam rationem pugnae insisterent.— ili, 14.

a) Write the first person singular, present indicative of intellenit (line 1),

visa est (line 4), profectae (line 5).

b) Tell in what case each of the following words is, and why this ca used: oppidis (line 1), classi (line 6), quibus (line 7).

c) Explain the derivation of pracerat (line 6).

se is

d) Tellin what mood and tense each of the following verbs is, and why this mood and tense are used: intellexit (line 1), exspectandam (line 3),

insisterent (line 9). e) Conjugate moceri (line 3) in the present subjunctive active.

3. Germanico bello confecto, multis de causis Caesar statuit sibi Rhenum esse transeundum; quarum illa fuit iustissima, quod, cum videret Ger- manos tam facile impelli ut in Galliam venirent, suis quoque rebus eos timere voluit, cum intellegerent et posse et audere populi Romani exercitum Rhenum transire. Accessit etiam quod illa pars equitatus Usipetum et Tencterorum, quam supra commemoravi praedandi frumentandique causa Mosam transisse neque proelio interfuisse, post fugam suorum se trans Rhenum in finis Sugambrorum receperat seque cum iis coniunxerat.—iv, 16

64

a) Tell in what case each of the following words is, and why this case is used: rebus (line 3), equitatus (line 5), praedandi (line 6), proelio (line 7). b) Write the principal parts of confecto (line 1), impelli (line 3), interfuisse

(line 7).

c) Tell in what mood and tense each of the following verbs is, and why this mood and tense are used: transeundum (line 2), venirent (line 3), transisse

(line 7).

d) Conjugate confecto (line 1) in the pluperfect subjunctive active, voluit

(line 4) in the imperfect subjunctive. e) What is the force of the prefix in confecto (line 1). f) Compare facile (line 3).

4. Mittuntur ad Caesarem confestim ab Cicerone litterae, magnis propositis praemiis si pertulissent; obsessis omnibus viis missi intercipiuntur. Noctu ex materia quam munitionis causa comportaverant turres admodum cxx excitantur incredibili celeritate; quae deesse operi videbantur perficiuntur. Hostes postero die multo maioribus coactis copiis castra oppugnant, fossam complent. Ab nostris eadem ratione qua pridie resistitur. Hoc idem reliquis deinceps fit diebus. Nulla pars nocturni temporis ad laborem intermittitur; non aegris, non vulneratis facultas quietis datur. Quaecumque ad proximi diei oppugnationem opus sunt noctu comparantur.—v, 4o.

Io

a) Explain the derivation of obsessis (line 2), coactis (line 5), facultas (line 9),

and tell what the suffix denotes in the last word.

b) Tell in what case each of the following words is, and why this case is

used: praemiis (line 2), operi (line 4), die (line 5), gua (line 7).

c) Tell in what mood and tense each of the following verbs is, and why this

mood and tense are used: pertulissent (line 2), comportaverant (line 3).

d) Conjugate videbantur (line 5) in the present subjunctive active, complent

(line 6) in the future indicative active. e) Compare postero (line 5), maioribus (line 5).

5. Civitatibus maxima laus est quam latissime circum se vastatis finibus soli- tudines habere. Hoc proprium virtutis existimant, expulsos agris finitimos cedere neque quemquam prope audere consistere; simul hoc se fore tutiores arbitrantur repentinae incursionis timore sublato. Cum bellum civitas aut inlatum defendit aut infert, magistratus qui ei bello praesint, ut vitae necisque habeant potestatem deliguntur. In pace nullus est communis magistratus, sed principes regionum atque pagorum inter suos ius dicunt controversiasque minuunt. Latrocinia nullam habent infamiam, quae extra finis cuiusque Civi- tatis fiunt, atque ea iuventutis exercendae ac desidiae minuendae causa fieri praedicant.—vi, 23. .

a) Compare Jatissime (line 1), tutiores (line 4). b) Decline hoc (line 4) in the neuter, mecis (line 6), ius (line 8). ¢) Conjugate expulsos (line 2) in the pluperfect indicative active.

Io

d) Tell in what case each of the following words is, and why this case is"

used: infamiam (line 9), desidiae (line 10).

e) Tell in what mood and tense each of the following verbs is, and why

this mood and tense are used: praesint (line 6), fiert (line 11).

65

1915 LATIN B—CAESAR

Wednesday QO-II a.m.

Translate three of the following passages, and answer the questions on the passages chosen. The translation should be exact, but in clear and natural English.

1. Hac oratione habita mirum in modum conversae sunt omnium mentes, summaque alacritas et cupiditas belli gerendi innata est; princepsque decima legio per tribunos militum ei gratias egit quod de se optimum iudicium fecisset, seque esse ad bellum gerendum paratissimam con- firmavit. Deinde reliquae legiones cum tribunis militum et primorum 5 ordinum centurionibus egerunt uti Caesari satisfacerent: se nec um- quam dubitasse neque timuisse, neque de summa belli suum iudicium sed imperatoris esse existimavisse. ~i, 41.

a) Decline oratione (line 1), menies (line 1).

b) Tell in what mood and tense fecisset (line 4) is, and why this mood and tense are used.

c) Tell in what case each of the following words is, and why this case is used: belli (line 2), et (line 3), imperatoris (line 8).

d) Tell whether decima (line 3) is the cardinal, ordinal, or distributive form of the numeral, and write the two remaining forms for this same number.

e) Change to the direct form se dubitasse (lines 6, 7).

2. Legionis nonae et decimae milites, ut in sinistra parte acie constiterant, pilis emissis cursu ac lassitudine exanimatos vulneribusque confectos Atrebates (nam his ea pars obvenerat) celeriter ex loco superiore in flumen compulerunt, et transire conantis insecuti gladiis magnam partem eorum impeditam interfecerunt. Ipsi transire flumen non 5 dubitaverunt, et in locum iniquum progressi rursus resistentis hostis redintegrato proelio in fugam coniecerunt. Item alia in parte diversae duae legiones, undecima et octava, profligatis Viromanduis, quibuscum erant congressi, ex loco superiore in ipsis fluminis ripis proeliabantur.

li, 23. a) Decline in the singular in all genders simistra (line 1); decline in the plural in all genders superiore (line 3).

b) Conjugate constiterant (line 1) in the perfect subjunctive active, com- pulerunt (line 4) in the imperfect indicative passive.

c) Write all the infinitives of transire (line 4), naming each; write all the participles of confectos (line 2), naming each.

d) Compare iniquum (line 6). e) Tell in what case his (line 3) is, and why this case is used.

3. Exigua parte aestatis reliqua Caesar, etsi in his locis, quod omnis Gallia ad septentriones vergit, maturae sunt hiemes, tamen in Britanniam proficisci contendit, quod omnibus fere Gallicis bellis hostibus nostris inde subministrata auxilia intellegebat, et, si tempus anni ad bellum gerendum deficeret, tamen magno sibi usui fore arbitrabatur si modo 5 insulam adisset et genus hominum perspexisset, loca, portus, aditus cognovisset; quae omnia fere Gallis erant incognita. Neque enim temere praeter mercatores illo adit quisquam neque his ipsis quicquam praeter oram maritimam atque eas regiones quae sunt contra Gallias notum est.—iv, 20.

66

a) Tell in what mood and tense each of the following verbs is, and why this mood and tense are used: vergit (line 2), adisset (line 6). b) Write the principal parts of proficisct (line 3), perspexisset (line 6), cognovisset (line 7). c) Tell in what case usui (line 5) is, and why this case is used. d) What is the adverb formed from maturae (line 2). Compare this adverb. e) Decline tempus (line 4), portus (line 6). 4. Caesar acceptis litteris hora circiter x1 diei statim nuntium in Bellovacos ad M. Crassum quaestorem mittit, cuius hiberna aberant ab eo milia passuum xxv. lIubet media nocte legionem proficisci celeriterque ad se venire. Exit cum nuntio Crassus. Alterum ad C. Fabium legatum mittit, ut in Atrebatum finis legionem adducat, qua sibiiter faciendum 5 sciebat. Scribit Labieno, si rei publicae commodo facere posset, cum legione ad finis Nerviorum veniat. Reliquam partem ‘exercitus, quod paulo aberat longius, non putat exspectandam; equites circiter quad- ringentos ex proximis hibernis conligit.—v, 46. a) Express in Latin words the numerals in lines 1 and 3. b) Tell in what case each of the following words is, and why this case is used: hora (line 1), milia (line 2), iter (line 5). c) What is the adjective from which celeriter (line 3) is formed? Decline this adjective in the singular in all genders. d) Write the present infinitive, active and passive, of acceptis (line 1). e) Tell in what mood and tense each of the following verbs is, and why this mood and tense are used: aberant (line 2), veniat (line 7). 5. Caesar partitis copiis cum C. Fabio legato et M. Crasso quaestore celeriter- que effectis pontibus adit tripartito, aedificia vicosque incendit, magno pecoris atque hominum numero potitur. Quibus rebus coacti Menapii legatos ad eum pacis petendae causa mittunt. Ille obsidibus acceptis hostium se habiturum numero confirmat, si aut Ambiorigem aut eius’ 5 legatos finibus suis recepissent. His confirmatis rebus Commium Atrebatem cum equitatu custodis loco in Menapiis relinquit; ipse in Treveros proficiscitur.—vi, 6. a) Change to the direct form se habiturum, si recepissent (lines 5-6). b) Tell in what case each of the following words is, and why this case is used: numero (line 3), rebus (line 3), pacis (line 4), obsidibus (line 4). c) Conjugate recepissent (line 6) in the future indicative passive, relinqutt : (line 7) in the pluperfect subjunctive active. d) Tell in what tense adit (line 2) is, and why this tense is used. e) Change partitis coptis (line 1) to a cum-clause.

67

LATIN C

CICERO SIX ORATIONS

IQII

LATIN C—CICERO

Thursday Q-II a.m.

Translate any three of the following passages, and answer the questions on the pas-

sages chosen. The translation should be exact, but in clear and natural English.

I.

3-

Haec igitur tibi reliqua pars est, hic restat actus, in hoc elaborandum est, ut rem publicam constituas, eaque tu in primis cum summa tranquillitate et otio perfruare; tum te, si voles, cum et patriae quod debes solveris et naturam ipsam expleveris satietate vivendi, satis diu vixisse dicito.. Quid est enim hoc ipsum diu, in quo 5 est aliquid extremum? quod cum venit, omnis voluptas prae- terita pro nihilo est, quia postea nulla est futura. Quamquam - iste tuus animus numquam his angustiis, quas natura nobis ad vivendum dedit, contentus fuit, semper immortalitatis amore flagravit.—Pro Marcello, 27.

a) Explain the metaphor in haec pars (line 1).

b) Tell in what case each of the following words is, and why this case is used: ea (line 2), tranquillitate (line 3), angustiis (line 8).

c) Compare summa (line 2), diu (line 5).

d) What called forth this speech? How may Cicero’s tone towards Caesar be explained ?

e) Tell in what mood and what tense each of the following verbs is, and why this mood and this tense are used: perfrware (line 3), solveris (line 4).

Itaque, credo, si civis Romanus Archias legibus non esset, ut ab aliquo imperatore civitate donaretur perficere non potuit. Sulla cum Hispanos et Gallos donaret, credo, hunc petentem repudiasset: quem nos in contione vidimus, cum ei libellum malus poeta de populo subiecisset, quod epigramma in eum ae fecisset tantum modo alternis versibus longiusculis, statim ex iis rebus quas tum vendebat iubere ei praemium tribui, sed ea condicione, ne quid postea scriberet. Qui sedulitatem mali poetae duxerit aliquo tamen praemio dignam, huius ingenium et virtu- tem in scribendo et copiam non expetisset >—Pro Archia, 25.

a) Tell in what mood each of the following verbs is, and why this mood is used: esset (line 1), repudiasset (line 4), fecisset (line 6), scriberet (line 8).

6) Tell in what case each of the following words is, and why this case is used: legibus (line 1), imperatore (line 2), quem (line 4), praemio (line 9).

c) Make clear the logical force of amen (line 9) in this sentence.

d) What is the suffix in longiusculis (line 6), and what does it denote?

e) What verb is used as the passive of vendo ?

Meministine me ante diem xi Kalendas Novembris dicere in senatu fore in armis certo die, qui dies futurus esset ante diem vi Kalendas Novembris, C. Manlium, audaciae satellitem atque administrum tuae? Num me fefellit, Catilina, non modo res tanta, tam atrox tamque incredibilis, verum, id quod multo 5 magis est admirandum, dies? Dixiego idem in senatu caedem te optimatium contulisse in ante diem v Kalendas Novembris, tum cum multi principes civitatis Roma non tam sui conservandi quam tuorum consiliorum reprimendorum causa profugerunt. —In Catilinam, 1, 7.

70

_ a) Explain briefly the Roman method of indicating the days of the month.

b) Tell in what mood and what tense each of the following verbs is, and why this mood and this tense are used: dicere (line 1), futurus esset (line 2).

c) Give the principal parts of fefellit (line 4), contulisse (line 7), reprimen- dorum (line 9).

d) Change tuorum consiliorum reprimendorum causa (line 9) to the gerund construction.

e) Compare magis (line 6), multi (line 8).

Cnidum aut Colophonem aut Samum, nobilissimas urbes, innu- merabilesque alias captas esse commemorem, cum vestros portus atque eos portus quibus vitam et spiritum ducitis in praedonum fuisse potestate sciatis? An vero ignoratis portum Caietae celeberrimum ac plenissimum navium inspectante praetore a 5 praedonibus esse direptum ? ex Miseno autem eius ipsius liberos qui cum praedonibus antea ibi bellum gesserat a praedonibus esse sublatos? Nam quid ego Ostiense incommodum atque illam labem atque ignominiam rei publicae querar, cum prope

inspectantibus vobis classis ea cui consul populi Romani praepo- IO situs esset a praedonibus capta atque oppressa est >—Pro Lege Manilia, 33.

a) Give the situation of Misenum, Ostia.

b) Tell in what mood each of the following verbs is, and why this mood is used: commemorem (line 2), praepositus esset (line 10).

c) Mention and define one English derivative from any form of each of the following Latin words: plenissimum (line 5), querar (line 9), praeposi- tus (line 10). :

d) Of what is Cicero thinking when he says portus quibus vitam et spiritum ducitis (line 3)?

e) Explain the derivation of potestate (line 4).

Leguntur eadem ratione ad senatum Allobrogum populumque litterae. Si quid de his rebus dicere vellet, feci potestatem. Atque ille primo quidem negavit; post autem aliquanto, toto iam indicio exposito atque edito, surrexit, quaesivit a Gallis quid sibi esset cum eis, quam ob rem domum suam venissent, 5 itemque a Volturcio. Qui cum illi breviter constanterque respondissent per quem ad eum quotiesque venissent, quaesis- sentque ab eo nihilne secum esset de fatis Sibyllinis locutus, tum ille subito scelere demens quanta conscientiae vis esset ostendit.

Nam cum id posset infitiari, repente praeter opinionem omnium 10 confessus est.—IJn Catilinam, iii, 11

a) To whom was this speech delivered ? What led to its delivery ? b) Tell in what case domum (line 5) is, and why this case is used.

c) Tell in what mood each of the following verbs is, and why this mood is used: vellet (line 2), posset (line 10).

d) Mention and define one English derivative from any form of each of the following Latin words: Jeguntur (line 1), swrrewit (line 4), quaesivit (line 4). e) Decline vis (line 9) in the singular and the plural.

71

IgI2

LATIN C—CICERO

Thursday Q-II a.m. Translate any three of the following passages, and answer the questions on the

passages chosen. The translation should be exact, but in clear and natural English.

1. Quis enim toto mari locus per hos annos aut tam firmum habuit praesidium

ut tutus esset aut tam fuit abditus ut lateret? Quis navigavit qui non se aut mortis aut servitutis periculo committeret, cum aut hieme aut referto praedonum mari navigaret? Hoc tantum bellum, tam turpe, tam vetus, tam late divisum atque dispersum quis umquam arbitraretur 5 aut ab omnibus imperatoribus uno anno aut omnibus annis ab uno imperatore confici posse? Quam provinciam tenuistis a praedonibus liberam per hosce annos? Quod vectigal vobis tutum fuit? Quem socium defendistis? Cui praesidio classibus vestris fuistis? Quam multas existimatis insulas esse desertas? Quam multas aut metu 10 relictas aut a praedonibus captas urbes esse sociorum ? —Pro Lege Manilia, 31, 32. a) Why was Pompey’s campaign against the pirates considered so brilliant ? b) Tell in what mood each of the following verbs is, and why this mood is used: committeret (line 3), navigaret (line 4), arbitraretur (line 5). c) Tell in what case each of the following words is, and why this case is used: praesidio (line g), classibus (line 9), metu (line 10). d) Explain the derivation of servitutis (line 3). e) Decline quis locus (line 1) in the singular and plural.

2. His ego sanctissimis rei publicae vocibus et eorum hominum qui hoc idem

sentiunt mentibus pauca respondebo. Ego si hoc optimum factu iudi-

carem, patres conscripti, Catilinam morte multari, unius usuram horae

gladiatori isti ad vivendum non dedissem. Etenim si summi viri et

clarissimi cives Saturnini et Gracchorum et Flacci et superiorum com- 5

plurium sanguine non modo se non contaminarunt sed etiam honesta-

runt, certe verendum mihi non erat ne quid hoc parricida civium inter-

fecto invidiae mihi in posteritatem redundaret. Quod si ea mihi

maxime impenderet, tamen hoc animo semper fui ut invidiam virtute

partam gloriam, non invidiam putarem.—In Catilinam, i, 29. 10

a) In what way was the possibility spoken of in line 8 subsequently realized ?

b) oi 5 principal parts of sentiunt (line 2), respondebo (line 2), partam line 10). .

c) Explain the derivation of gladiatori (line 4).

d) Tell in what case each of the following words is, and why this case is used: mihi (line 7), invidiae (line 8), animo (line 9).

e) What kind of accusative is hoc (line 1) ?

3. Qua re gaude tuo isto excellenti bono, et fruere cum fortuna et gloria tum

etiam natura et moribus tuis; ex quo quidem maximus est fructus iucun- ditasque sapienti. Cetera cum tua recordabere, etsi persaepe virtuti, tamen plerumque felicitati tuae gratulabere; de nobis, quos in re publica tecum simul esse voluisti, quotiens cogitabis, totiens de maximis tuis 5 beneficilis, totiens de incredibili liberalitate, totiens de singulari sapientia tua cogitabis, quae non modo summa bona sed nimirum audebo vel sola dicere. Tantus est enim splendor in laude vera, tanta in magni- tudine animi et consili dignitas ut haec a virtute donata, cetera a fortuna commodata esse videantur.—Pro Marcello, 19. ie)

72

a) Why did Caesar’s action seem to Cicero to be so magnanimous ?

b) Tell in what case each of the following words is, and why this case is used: bono (line 1), fortuna (line 1), virtuti (line 3).

c) Give the principal parts of gaude (line 1), fruere (line 1), audebo (line 7).

d) Explain the derivation of incredibili (line 6).

e) Why is the preposition @ used with virtute and fortuna (lines g and 10) ?

4. Nunc ego, patres conscripti, mea video quid intersit. Si eritis secuti sen- tentiam C. Caesaris, quoniam hanc is in re publica viam quae popularis habetur secutus est, fortasse minus erunt hoc auctore et cognitore huiusce sententiae mihi populares impetus pertimescendi; sin illam alteram, nescio an amplius mihi negoti contrahatur. Sed tamen meo-_ 5 rum periculorum rationes utilitas rei publicae vincat. Habemus enim a Caesare, sicut ipsius dignitas et maiorum eius amplitudo postulabat, sententiam tamquam obsidem perpetuae in rem publicam voluntatis. Intellectum est quid interesset inter levitatem contionatorum et animum vere popularem, saluti populi consulentem.—In Catilinam, iv, 9. Ke)

a) State the gist of Caesar’s proposal.

b) On what legal grounds did Cicero justify the execution of the conspira- tors?

c) Tell in what mood each of the following verbs is, and why this mood is used: contrahatur (line 5), vincat (line 6).

d) Tell in what case each of the following words is, and why this case is used: mea (line 1), saluti (line 10).

e) Compare vere (line 10).

5. Si haec non gesta audiretis, sed picta videretis, tamen appareret uter esset insidiator, uter nihil cogitaret mali, cum alter veheretur in raeda paenu- latus, una sederet uxor. Quid horum non impeditissimum, vestitus an vehiculum an comes? Quid minus promptum ad pugnam, cum paenula inretitus, raeda impeditus, uxore paene constrictus esset? Videtenunc 5 illum primum egredientem e villa subito (cur ?), vesperi (quid necesse est ?) tarde (qui convenit, praesertim id temporis?). Devertit in villam Pompei. Pompeium ut videret? Sciebat in Alsiensi esse. Villam ut perspiceret? Miliens in ea fuerat. Quid ergo erat? Mora et tergi- versatio: dum hic veniret, locum relinquere noluit—Pro Milone, 54. to.

a) Explain the reference in alter (line 2), illum (line 6). b) Tell in what mood and tense each of the following verbs is, and why es rN and this tense are used: audiretis (line 1), esset (line 1), videret ine 8). c) What was tne result of the trial of Milo? d) Explain the derivation of comes (line 4). e) Write the genitive and dative singular of wéer (line 1).

73°

LATIN C—CICERO

~

-I9%3

Thursday Q-II a.m.

Translate any three of the following passages, and answer the questions on the pas-

sages chosen. The translation should be exact, but in clear and natural English.

1. Age vero, illa res quantam declarat eiusdem hominis apud hostis populi Romani auctoritatem, quod ex locis tam longinquis tamque diversis tam brevi tempore omnes huic se uni dediderunt; quod Cretensium legati, cum in eorum insula noster imperator exercitusque esset, ad Cn. Pompeium in ultimas prope terras venerunt, eique se omnis Cretensium civitates dedere velle dixerunt? Quid? Idem iste Mithridates nonne ad eundem Cn. Pompeium legatum usque in Hispaniam misit, eum quem Pompeius legatum semper iudicavit, ii quibus erat molestum ad eum potissimum esse missum speculatorem quam legatum iudicari maluerunt ?>—Pro Lege Manilia, 46.

a) Account for the number of age (line 1).

b) What was illa res (line 1) ?

Io

c) What was Pompey doing in Spain at the time of the incident mentioned

in the last sentence of this passage ?

d) Explain the construction of ewm (line 7). What is the subject of erat

(line 8)? Explain the gender of molesium (line 8).

2. At vero C. Caesar intellegit legem Semproniam esse de civibus Romanis constitutam; qui autem rei publicae sit hostis, eum civem esse nullo modo posse; denique ipsum latorem Semproniae legis iniussu populi poenas rei publicae dependisse. Idem ipsum Lentulum, largitorem et prodigum, non putat, cum de pernicie populi Romani, exitio huius urbis tam acerbe, tam crudeliter cogitarit, etiam appellari posse popularem. Itaque homo mitissimus atque lenissimus non dubitat P. Lentulum aeternis tenebris vinculisque mandare, et sancit in posterum ne quis huius supplicio levando se iactare et in perniciem populi Romani posthac popularis esse possit.—In Catilinam, iv, to.

Io

a) Why was the lex Sempronia brought into this discussion? Why was it called Sempronia? How was Cicero made to suffer later as the result

of his disregard of this law ?

b) Relate more fully the event referred to in ipsum... . dependisse

(lines 3-4). c) Explain the spelling cogitarit (line 6).

_ ad) What is the construction of the clause me. . . . possit (lines 8-10) ? e) Explain the derivation of iniussu (line 3). Give the literal meaning of

iactare (line 9), with regard to its derivation.

3. Quae cum ita sint, quid est quod de eius civitate dubitetis, praesertim cum aliis quoque in civitatibus fuerit ascriptus? Etenim cum mediocribus multis et aut nulla aut humili aliqua arte praeditis gratuito civitatem in Graecia homines impertiebant, Reginos credo aut Locrensis aut Neapolitanos aut Tarentinos, quod scaenicis artificibus largiri solebant, id huic summa ingeni praedito gloria noluisse. Quid? Ceteri non modo post civitatem datam sed etiam post legem Papiam aliquo modo in eor'um municipiorum tabulas inrepserunt; hic qui ne utitur quidem illis in quibus est scriptus, quod semper se Heracliensem esse voluit, reicietur >—Pro Archia, to.

74

Io

a) What is meant by Graecia (line 4)? Name five of its chief cities. b) Why did Cicero single out scaenici artifices (line 5) for special mention ? c) To what did Cicero refer in civitatem datam (line 7)? What was the lex Papia? d) Why was the indicative used, rather than the subjunctive, in impertiebani (line 4), solebant (line 5) ? } e) Explain the peculiarity in the pronunciation of imgeni (line 6).

4. Scio igitur Tuberonem domi manere voluisse; sed ita quidam agebat, ita

rei publicae sanctissimum nomen opponebat ut, etiam si-aliter sentiret,

verborum tamen ipsorum pondus sustinere non posset. Cessit auctori-

tati amplissimi viri, vel potius paruit; una est profectus cum lis quorum

erat una causa. ‘Tardius iter fecit itaque in Africam venit iam occu- 5

patam. Hinc in Ligarium crimen oritur, vel ira potius. Nam si crimen

est illum voluisse, non minus magnum est vos Africam, arcem omnium

provinciarum, natam ad bellum contra hanc urbem gerendum, obtinere

voluisse quam aliquem se maluisse. Atque is tamen aliquis Ligarius

non fuit. Varus imperium se habere dicebat; fascis certe habebat. _—10

—Pro Ligario, 21, 22.

a) What was the charge against Ligarius? What was the nature of Cicero’s argument P

b) In what capacity had Tubero gone to Africa ?

c) Of what was Cicero thinking when he said natam.... gerendum (line 8) ?

d) Explain the meaning of fascis habebat (line 10).

e) What is the subject of est (line 7)? Explain the construction of vos (line 7), se (line 9g).

5. Nos vero cum semel ad eum quem cupimus optamusque vestitum redieri-

mus, id agamus ut eum in perpetuum retineamus. Nam hoc quidem cum turpe est tum ne dis quidem immortalibus gratum, ab eorum aris, ad quas togati adierimus, ad saga sumenda discedere. Atque animad- verto, patres conscripti, quosdam huic favere sententiae; quorum ea 5 _ mens idque consilium est ut, cum videant gloriosissimum illum D. Bruto futurum diem, quo die propter eius salutem redierimus ad vestitum, hunc ei fructum eripere cupiant, ne memoriae posteritatique prodatur propter unius civis periculum populum Romanum ad saga isse, propter eiusdem salutem redisse ad togas.—Philippica, xiv, 2, 3. 10 a) In a chronological list of Cicero’s speeches, where does this one fall ? What was the occasion of its delivery? Why did Cicero mention aris (line 3)? b) Tell what you know of D. Brutus. c) Explain the meaning of ad saga isse, redisse ad togas (lines 9-10). What term is applied to the peculiar order of these words? d) Explain the construction of ez (line 8), and the spelling of redisse (line 10).

75

LATIN C—CICERO

1914

Thursday Q-II a.m.

Translate any three of the following passages, and answer the questions on the sages chosen. The translation should be exact, but in clear and natural English.

1. Cnidum aut Colophonem aut Samum, nobilissimas urbis, innumerabilisque alias captas esse commemorem, cum vestros portus, atque eos portus quibus vitam ac spiritum ducitis, in praedonum fuisse potestate sciatis ? An vero ignoratis portum Caietae celeberrimum ac plenissimum navium inspectante praetore a praedonibus esse direptum; ex Miseno autem eius ipsius liberos qui cum praedonibus antea ibi bellum gesserat a praedonibus esse sublatos? Nam quid ego Ostiense incommodum atque illam labem atque ignominiam rei publicae querar, cum prope inspectantibus vobis classis ea cui consul populi Romani praepositus esset a praedonibus capta atque oppressa est >—Pro Lege Manilia, 33.

pas-

Io

a) By what law was Pompey put in command of the war with the pirates ?

In what year? Describe his previous military achievements.

b) Where were the places referred to in Samum (line 1), Miseno (line 5),

Ostiense (line 7) ?

c) What is the point of the contrast between Cuidum aut Colophonem aut Samum (line 1) and vestros portus (line 2)? Explain the meaning of

quibus vitam ac spiritum ducitis (line 3). d) What force does an vero (line 4) give to a question ? 2. Introducti autem Galli ius iurandum sibi et litteras ab Lentulo, Cethego,

Statilio ad suam gentem data esse dixerunt, atque ita sibi ab his et a L.

Cassio esse praescriptum, ut equitatum in Italiam quam primum mitterent; pedestris sibi copias non defuturas. Lentulum autem sibi confirmasse ex fatis Sibyllinis haruspicumque responsis se esse tertium illum Cornelium ad quem regnum huius urbis atque imperium pervenire esset necesse; Cinnam ante se et Sullam fuisse; eundemque dixisse fatalem hunc annum esse ad interitum huius urbis atque imperi, qui esset annus decimus post virginum absolutionem, post Capitoli autem incensionem vicesimus.—In Catilinam, iii, 9.

a) Who were the Galli mentioned inline1? Give an account of their

Io con-

nection with the conspiracy. To what meeting does introducti (line 1)

refer P b) Who were Lentulo (line 1), Cinnam (line 7), Sullam (line 7) ? c) What is meant by tertium illum Cornelium (lines 5, 6) ?

d) What were the fatis Sibyllinis (line 5), the Capitoli (line 9)? Who

were the virginum (line g) ?

3- Census nostros requiris. Scilicet; est enim obscurum proximis censoribus hunc cum clarissimo imperatore L. Lucullo apud exercitum fuisse; superioribus, cum eodem quaestore fuisse in Asia; primis, Tulio et Crasso, nullam populi partem esse censam. Sed quoniam census non ius civitatis confirmat ac tantum modo indicat eum qui sit census ita se iam tum gessisse pro Cive, iis temporibus quem tu criminaris ne ipsius quidem iudicio in civium Romanorum iure esse versatum, et testamen- tum saepe fecit nostris legibus et adiit hereditates civium Romanorum et in beneficiis ad aerarium delatus est a L. Lucullo pro consule. Quaere argumenta, si quae potes; numquam enim hic neque suo neque amico- rum iudicio revincetur.—Pro Archia, 11.

76

Io

a) What was the object of the Roman census (line 1)? How often was one made? In what sense is primis (line 3) used ?

b) What force does scilicet (line 1) give to the sentence?

c) What war was being fought at the time indicated by quaestore (line 3) ? Who was the Roman commander in this war? In what capacity was Archias apud exercitum (line 2) ?

d) What is the point of Cicero’s argument in the sentence beginning sed quoniam (line 4)? Explain the meaning of im beneficiis ad aerarium delatus est (line 9).

4. Neque enim posset aut Ahala ille Servilius aut P. Nasica aut L. Opimius

aut C. Marius aut me consule senatus non nefarius haberi, si sceleratos

civis interfici nefas esset. Itaque hoc, iudices, non sine causa etiam

fictis fabulis doctissimi homines memoriae prodiderunt, eum qui patris

ulciscendi causa matrem necavisset variatis hominum sententiis non 5

solum divina sed etiam sapientissimae deae sententia liberatum. Quod

si duodecim tabulae nocturnum furem quoquo modo, diurnum autem si

se telo defenderet, interfici impune voluerunt, quis est qui, quoquo modo

quis interfectus sit, puniendum putet, cum videat aliquando gladium

nobis ad hominem occidendum ab ipsis porrigi legibus ? 10 Pro Milone, 8, 9.

a) Explain the reference to Ahala (line 1), Nasica (line 1), Opimius (line 1), Marius (line 2), senatus (line 2). b) Tell the story to which allusion is made in the sentence beginning itaque res oe 3), giving the names of eum (line 4), patris (line 4), and deae line 6). c) Put Cicero’s argument in the sentence beginning quod si (lines 6, 7) in the fewest and simplest words.

d) What were the duodecim tabulae (line 7) ?

5. Neque enim ego illa nec ulla umquam secutus sum arma civilia; semper-

que mea consilia pacis et togae socia, non belli atque armorum fuerunt.

Hominem sum secutus privato consilio, non publico; tantumque apud

me grati animi fidelis memoria valuit ut nulla non modo cupiditate sed

ne spe quidem, prudens et sciens tamquam ad interitum ruerem volun- 5

tarium. Quod quidem meum consilium minime obscurum fuit. Nam

et in hoc ordine integra re multa de pace dixi et in ipso bello eadem etiam

cum capitis mei periculo sensi. Ex quo nemo iam erit tam iniustus

existimator rerum qui dubitet quae Caesaris de bello voluntas-fuerit,

cum pacis auctores conservandos statim censuerit, ceteris fuerit iratior. 10 Pro Marcello, 14, 15.

a) What is meant by hoc ordine (line 7), ipso bello (line 7)? Who is meant by hominem (line 3) ? b) Explain the significance of togae (line 2). What was the periculo (line 8) ?

c) Explain what Cicero meant in the sentence beginning hominem sum secutus (line 3), and in the sentence beginning ex quo nemo (line 8).

d) In what year was this speech delivered? To whom was it addressed? What was the occasion ?

77

LATIN C—CICERO

IQI5

Thursday Q-II a.m.

Translate three of the following passages, and answer the questions on the passages

chosen. The translation should be exact, but in clear and natural English. _t. Inde cum se in Italiam recepisset, duabus Hispaniis et Gallia Transalpina

praesidiis ac navibus confirmata, missis item in oram Illyrici maris et in

Achaiam omnemque Graeciam navibus, Italiae duo maria maximis

classibus firmissimisque praesidiis adornavit; ipse autem ut Brundisio

profectus est, undequinquagesimo die totam ad imperium populi 5

Romani Ciliciam adiunxit. Omnes qui ubique praedones fuerunt

partim capti interfectique sunt, partim unius huius se imperio ac potes-

tati dediderunt. Idem Cretensibus, cum ad eum usque in Pamphyliam ~

legatos deprecatoresque misissent, spem deditionis non ademit obsides-

que imperavit.—Pro Lege Manilia, 35.

a) In what year was this speech delivered? What official position did Cicero hold?

b) What were the duabus Hispaniis (line 1), the duo maria (line 3)? Where were Brundisio (line 4), Ciliciam (line 6) ?

c) What arguments were urged against this bill by Hortensius and Catulus ? How does Cicero answer these arguments ?

d) Write the numeral sign for undequinquagesimo (line 5).

2. Sit Scipio clarus ille cuius consilio atque virtute Hannibal in Africam redire

atque ex Italia decedere coactus est; ornetur alter eximia laude Africanus,

qui duas urbis huic imperio infestissimas, Karthaginem Numantiam-

que, delevit; sit aeterna gloria Marius, qui bis Italiam obsidione et

metu servitutis liberavit; anteponatur omnibus Pompeius, cuius res 5

gestae atque virtutes isdem quibus solis cursus regionibus ac terminis

continentur; erit profecto inter horum laudes aliquid loci nostrae

gloriae, nisi forte maius est patefacere nobis provincias quo exire

possimus quam curare ut etiam illi qui absunt habeant quo victores

revertantur.—In Catilinam, iv, 21. . 10

a) To whom was this speech delivered? What led to its delivery?

b) What course of action was urged by Silanus? What by Caesar? Which of the two courses did Cicero favor ?

c) Where were Karthaginem (line 3), Numantiam (line 3)? To what victories does bis (line 4) refer ?

d) What force does nisi forte (line 8) give to the dias which it introduces ?

3. Qua re conservate, iudices, hominem pudore eo quem amicorum videtis

comprobari cum dignitate tum etiam vetustate; ingenio autem tanto quantum id convenit existimari quod summorum hominum ingeniis expetitum esse videatis; causa vero eius modi qua beneficio legis, auctoritate municipi, testimonio Luculli, tabulis Metelli comprobetur. 5 Quae de causa pro mea consuetudine breviter simpliciterque dixi, iudices, ea confido probata esse omnibus. Quae a forensi aliena iudici-

78

alique consuetudine et de hominis ingenio et communiter de ipso ~

studio locutus sum, ea, iudices, a vobis spero esse in bonam partem

accepta; ab eo qui iudicium exercet certo scio.—Pro Archia, 31, 32. 10

a) In what year was this speech delivered? Who is meant by eo (line 10) ? What was the charge against Archias ? 3

b) Explain the reference in /egis (line 4). State the provisions of this legis. Give the name of the municipi (line 5).

c) What advantage does Cicero find in the fact that the poems of Archias were written in Greek? What poetical writings of Archias does Cicero refer to in this speech?

d) Show precisely how Cicero uses his praise of literature as an argument for the acquittal of his client.

4. Interim cum sciret Clodius—neque enim erat difficile scire—iter sollemne,

legitimum, necessarium ante diem xm Kalendas Februarias Miloni

esse Lanuvium ad flaminem prodendum, Roma subito ipse profectus

pridie est, ut anté suum fundum, quod re intellectum est, Miloni insidias

conlocaret. Atque ita profectus est ut contionem turbulentam, in qua 5

eius furor desideratus est, relinqueret, quam, nisi obire facinoris locum

tempusque voluisset, numquam reliquisset.. Milo autem cum in senatu

fuisset eo die, quoad senatus est dimissus, domum venit, calceos et

_ vestimenta mutavit, dein profectus id temporis cum iam Clodius, si

-quidem eo die Romam venturus erat, redire potuisset.—Pro Milone, 10

27, 28.

a) Write the Latin words for which xm (line 2) stands. What is the construction of the clause quod . . . . est (line 4)?

b) What were the calceos et vestimenta (lines 8, 9) ?

c) Why were Clodius and Milo enemies? What was the result of the trial ?

d) Show the bearing of the words profectus . . . . potwisset (lines 9, 10) upon the question of Milo’s innocence.

5. Sed hoc quaero: Quis putat esse crimen fuisse in Africa? Nempe is

qui et ipse in eadem Africa esse voluit et prohibitum se a Ligario queritur, et certe contra ipsum Caesarem est congressus armatus. Quid enim, Tubero, tuus ille destrictus in acie Pharsalica gladius agebat ? Cuius latus ille mucro petebat? Qui sensus erat armorum tuorum? 5 Quae tua mens, oculi, manus, ardor animi? Quid cupiebas? Quid optabas? Nimis urgeo; commoveri videtur adulescens. Ad me revertar. Isdem in armis fui. Quid autem aliud egimus, Tubero, nisi ut quod hic potest nos possemus? Quorum igitur impunitas, Caesar, tuae clementiae laus est, eorum ipsorum ad crudelitatem te 10 acuet oratio ?>—Pro Ligario, g, to. a) Explain fully the events referred to in gui . . . . queritur (lines 2, 3)?

Who is meant by hic (line g)? b) Where was Pharsalus? What happened to Pompey after the battle? c) Why did Cicero side with Pompey ? d) How did Ligarius show his gratitude for Caesar’s pardon ?

79

LATIN D

VERGIL— 4£NEZJD, BOOKS I-VI

81

IQII

LATIN D—VIRGIL’S AENEID, BOOKS I-VI

Thursday I.30-3.30 p. m.°

I.

2.

Translate the following passages, and answer the questions on them. The transla- tion should be exact, but in clear and natural English.

a) b)

c) d) é) f) g)

“Nec tacui demens, et me, fors si qua tulisset, si patrios umquam remeassem victor ad Argos, 95 promisi ultorem, et verbis odia aspera movi. Hinc mihi prima mali labes, hinc semper Ulixes criminibus terrere novis, hinc spargere voces in volgum ambiguas, et quaerere conscius arma. Nec requievit enim, donec Calchante ministro— 100 Sed quid ego haec autem nequiquam ingrata revolvo quidve moror, si omnis uno ordine habetis Achivos, idque audire sat est? Jamdudum sumite poenas; hoc Ithacus velit, et magno mercentur Atridae.”—ii, 94-104.

Rewrite remeassem (verse 95) in the direct form.

In verses 94-96, what trait in his own character is Sinon seeking to sug- gest to his hearers ?

Tell in what mood terrere (verse 98) is, and why this mood is used.

Who is meant by Ithacus (verse 104)? Where was Ithaca?

Why is verse 104 a clever close of this part of Sinon’s tale?

To what kind of clause is conscius (verse 99) equivalent ?

Copy verses 101 and 102, and indicate the division into feet and the principal caesura.

Bellum etiam pro caede boum stratisque iuvencis, Laomedontiadae, bellumne inferre paratis et patrio Harpyias insontis pellere regno ? Accipite ergo animis atque haec mea ss dicta, 250 quae Phoebo pater omnipotens, mihi Phoebus Apollo praedixit, vobis Furiarum ego maxuma pando. Italiam cursu petitis, ventisque vocatis ibitis Italiam, portusque intrare licebit; sed non ante datam cingetis moenibus urbem, 255 quam vos dira fames nostraeque iniuria caedis ambesas subigat malis absumere mensas.”’—iii, 247-257.

Where were the Trojans when these words were spoken to them ? »

Explain the taunt in Laomedontiadae (verse 248).

What part does Apollo play in Book IIT?

Give the principal parts of pando (verse 252).

What poetical construction is used in verse 254? Change it to the cor- responding prose construction.

When and how is the prophecy made in verses 255-257 fulfilled ?

Write the nominative of malis (verse 257).

82

g)

*Tilae autem, paribus quas fulgere cernis in armis, concordes animae nunc et dum nocte premuntur, heu quantum inter se bellum, si lumina vitae attigerint, quantas acies stragemque ciebunt, aggeribus socer Alpinis atque arce Monoeci 830 descendens, gener adversis instructus Eois! Ne, pueri, ne tanta animis adsuescite bella, neu patriae validas in viscera vertite viris; tuque prior, tu parce, genus qui ducis Olympo, proice tela manu, sanguis meus! ’’—vi, 826-835. 835

Explain the reference in socer (verse 830) and gener (verse 831).

Explain the figure in aggeribus Alpinis (verse 830).

Show how the description of the opposing forces in verses 830-831 was true.

Give the principal parts of ciebunt (verse 829).

Rewrite ne adsuescite (verse 832) in the form which is most common in prose.

Of what facts is Anchises thinking when he says genus qui ducis Olympo (verse 834) ?

Point out and name the noteworthy feature of sound illustrated in verse 833.

83

LATIN D—VERGIL’S AENEID, BOOKS LVI” Thursday I.30-3.30 p.m.

Translate the following passages, and answer the questions on them. The translation should be exact, but in clear and natural English.

I. Tum breviter Dido vultum demissa profatur: “Solvite corde metum, Teucri, secludite curas.

Res dura et regni novitas me talia cogunt moliri et late finis custode tueri. Quis genus Aeneadum, quis Troiae nesciat urbem, 565 virtutesque virosque aut tanti incendia belli? Non obtunsa adeo gestamus pectora Poeni, nec tam aversus equos Tyria Sol iungit ab urbe. Seu vos Hesperiam magnam Saturniaque arva sive Erycis finis regemque optatis Acesten, 570 auxilio tutos dimittam opibusque iuvabo.”—i, 561-571.

a) Under what circumstances does Dido speak these words ?

6) Explain the syntax of vultum (verse 561). c) Of what facts is Dido thinking when she says res dura et regni novitas (verse 563) ?

d) Write the longer form of Aeneadum bee 565).

e) State in simple English the thought which is pa hs i figuratively in verse 568.

f) Why was Italy called Hesperiam (verse 569)? Where were the Erycis finis (verse 570) ?

g) Copy verses 568 and 569, and indicate the quantity of each syllable, the division into feet, and the principal caesuras.

2. Creta Iovis magni medio iacet insula ponto,

mons Idaeus ubi et gentis cunabula nostrae. 105 Centum urbes habitant magnas, uberrima regna, maxumus unde pater, si rite audita recordor, Teucrus Rhoeteas primum est advectus ad oras optavitque locum regno. Nondum Ilium et arces Pergameae steterant; habitabant vallibus imis. 110 Hinc mater cultrix Cybelae Corybantiaque aera Idaeumque nemus, hinc fida silentia sacris et iuncti currum dominae subiere leones. Ergo agite, et divom ducunt qua iussa sequamur.—iii, 104-114.

a) Where were the Trojans when these words were spoken to them ?

6) Give the location of the places referred to in Creta (verse 104), Rhoeteas

oras (verse 108), Idaeum nemus (verse 112).

84

c) Explain the allusion in mater (verse 111), Corybantia aera (verse 111).

d) What poetical construction is used in verse 110? Change it to the corre- sponding prose construction.

e) Explain the derivation of culirix (verse 111), giving the force of the suffix.

f) In what respect is the order of words in verse 114 an order that neither Caesar nor Cicero would have used ?

Tum Saturnius haec domitor maris edidit alti: “Fas omne est, Cytherea, meis te fidere regnis, 800 unde genus ducis. Merui quoque; saepe furores compressi et rabiem tantam caelique marisque. Nec minor in terris (Xanthum Simoentaque testor) Aeneae mihi cura tui. Cum Troia Achilles exanimata sequens impingeret agmina muris, 805 milia multa daret leto gemerentque repleti amnes nec reperire viam atque evolvere posset in mare se Xanthus, Pelidae tunc ego forti congressum Aenean nec dis nec viribus aequis nube cava rapui, cuperem cum vertere ab imo , 810 structa meis manibus periurae moenia Troiae.”—v, 799-811

a) Under what circumstances were these words spoken ? b) Who is meant by domitor maris (verse 799), Cytherea (verse 800) ?

c) Explain the allusion in Saturnius (verse 799), unde genus ducis (verse 801), Simoenta (verse 803).

d) Write the nominative case of Simoenta (verse 803). e) Explain the derivation of Pelidae (verse 808), giving the force of the suffix. f) Account for the syntax of Pelidae (verse 808), dis (verse 809).

35

LATIN D—VERGIL’S AENEID, BOOKS LVI

Thursday 2-4 p. m.

Translate the following passages, and answer the questions on them. The translation

should be exact, but in clear and natural English.

I.

2.

Sic Venus, et Veneris contra sic filius orsus: 325 “Nulla tuarum audita mihi neque visa sororum, O—quam te memorem, virgo ? Namque haud tibi voltus mortalis, nec vox hominem sonat.. O dea certe! An Phoebi soror ? An Nympharum sanguinis una? Sis felix nostrumque leves, quaecumque, laborem 330 et quo sub caelo tandem, quibus orbis in oris iactemur doceas; ignari hominumque locorumque erramus, vento huc vastis et fluctibus acti. Multa tibi ante aras nostra cadet ‘hostia dextra.”—i, 325-334.

a) Under what circumstances were these words spoken ?

b) Tell in what case mihi (verse 326) is, and why this case is used. What kind of accusative is hominem (verse 328) ?

c) Tell in what mood memorem (verse 327) is, and why this mood is used.

d) Explain the allusion in Phoebi soror (verse 329).

e) Write the principal parts of orsus (verse 325).

f) In what way does the scansion help you to determine the meaning in verse 334°

g) Copy verses 326 and 327, and indicate the quantity of each syllable, the division into feet, and the principal caesuras.

Tum pius Aeneas: “‘ Equidem sic poscere ventos iamdudum et frustra cerno te tendere contra. Flecte viam velis. An sit mihi gratior ulla, quove magis fessas optem demittere navis, quam quae Dardanium tellus mihi servat Acesten "30 et patris Anchisae gremio complectitur ossa ? ”’ Haec ubi dicta, petunt portus, et vela secundi intendunt Zephyri; fertur cita gurgite classis, et tandem laeti notae advertuntur harenae.—v, 26-34. a) To whom was Aeneas speaking, and under what circumstances ? b) In what various senses is the adjective pius (verse 26) applicable to Aeneas ? c) Rewrite verse 29 as the words would stand in prose.

d) ‘To what tellus does Aeneas refer in verse 30?

86

-e) What part does Acestes play in the story ? f) Why did Vergil write portus (verse 32) in the plural, rather than in the

singular ? g) How is the fact stated in verse 31 connected with the general subject of Book V ? | “Quin, ut te supplex peterem et tua limina adirem, 115

idem orans mandata dabat. Gnatique patrisque, alma, precor, miserere (potes namque omnia, nec te nequiquam lucis Hecate praefecit Avernis). Si potuit Manis arcessere coniugis Orpheus, Threicia fretus cithara fidibusque canoris, 120 si fratrem Pollux alterna morte redemit itque reditque viam totiens—quid Thesea magnum, quid memorem Alciden ?—et mi genus ab Iove summo.”’ vl, 115-123.

a) To whom were these words spoken, and under what circumstances ?

b) Tell in what case each of the following words is, and why this case is used: patris (verse 116), mz (verse 123).

c) Relate briefly the story referred to in verses 119-120.

d) Explain the allusion in fratrem .. . . redemit (verse 121), Alciden (verse 123). |

e) How did Aeneas learn that he had permission to descend alive into the lower world ?

f) Of what was the speaker thinking when he said ef . . . . summo (verse 123)? .

g) In what way. does Vergil use Book VI to glorify Rome?

87

LATIN D—VERGIL’S AENEID, BOOKS LVI

Thursday 2-4 p. m.

Translate the following passages, and answer the questions on them. The translation

should be exact, but in clear and natural English.

I.

“Hanc pro Palladio moniti, pro numine laeso effigiem statuere, nefas quae triste piaret. - Hanc tamen immensam Calchas attollere molem 185 roboribus textis caeloque educere iussit, ne recipi portis aut duci in moenia possit, neu populum antiqua sub religione tueri. Nam si vestra manus violasset dona Minervae, tum magnum exitium (quod di prius omen in ipsum 190 convertant!) Priami imperio Phrygibusque futurum; sin manibus vestris vestram ascendisset in urbem, ultro Asiam magno Pelopea ad moenia bello venturam, et nostros ea fata manere nepotes.”’—ii, 183-194.

a) Under what circumstances were these words spoken ?

b) Explain fully the reference in Palladio (verse 183).

c) What poetical construction is used in verse 186? Change it to the corresponding prose construction.

d) Make clear the meaning of verse 188. Explain the allusion in Pelopea (verse 193).

e) To whom does ipsum (verse 190) refer? What reason had the speaker given for hating this man?

f) How is Book II of the Aeneid related to Books I and III?

g) Copy verses 190 and 191, and indicate the quantity of each syllable, the division into feet, and the principal caesuras.

Agnovit prolem ambiguam geminosque parentes, 180 seque novo veterum deceptum errore locorum.

Tum memorat: “N ate, Iliacis exercite fatis,

sola mihi talis casus Cassandra canebat.

Nunc repeto haec generi portendere debita nostro,

et saepe Hesperiam, saepe Itala regna vocare. 185 Sed quis ad Hesperiae venturos litora Teucros

crederet ? Aut quem tum vates Cassandra moveret ?

Cedamus Phoebo et moniti meliora sequamur.”—iii, 180-188.

a) Under what circumstances were the words in verses 182-188 spoken ?

b) Write the principal parts of canebat (verse 183).

c) Why was Italy called Hesperiam (verse 185) ?

d) What is the literal meaning of camnebat (verse 183)? Why is it an appropriate word in this place?

e) Who was Cassandra? To what misfortune of hers ‘ies verse 187 refer ?

f) Name in order the places mentioned in Book III at which Aeneas landed.

g) What part did Phoebus (verse 188) play in the wanderings of Aeneas ?

88

““Huius in adventum iam nunc et Caspia regna responsis horrent divom et Maeotia tellus, et septemgemini turbant trepida ostia Nili. 800 Nec vero Alcides tantum telluris obivit, fixerit aeripedem cervam licet, aut Erymanthi pacarit nemora, et Lernam tremefecerit arcu, nec, qui pampineis victor iuga flectit habenis, Liber, agens celso Nysae de vertice tigris. 805 Et dubitamus adhuc virtutem extendere factis, aut metus Ausonia prohibet consistere terra ?’’—vi, 798-807.

a) Under what circumstances were these words spoken?

b) What and where were Caspia regna (verse 798), Ausonia terra (verse 807) ?

c) Explain the three allusions in verses 802 and 803.

d) Explain the form divom (verse 799). Write the longer form of pacarit (verse 803).

e) Account for Vergil’s attitude toward Augustus as indicated in this passage.

f) How is the thought expressed in verses 806 and 807 related to the verses that precede ?

g) What revelation of the future was granted to Aeneas in the lower world ?

89

I9I5 LATIN D—VERGIL’S AENEID, BOOKS I-VI

Thursday 2-4 p. m.

Translate the following passages, and answer the questions on them. The translation should be exact, but in clear and natural English.

x “His ego nec metas rerum nec tempora pono,

imperium sine fine dedi. Quin aspera Iuno,

quae mare nunc terrasque metu caelumque fatigat, 280 consilia in melius referet mecumque fovebit

Romanos, rerum dominos, gentemque togatam.

Sic placitum. Veniet lustris labentibus aetas

cum domus Assaraci Phthiam clarasque Mycenas

servitio premet ac victis dominabitur Argis. 285 Nascetur pulchra Troianus origine Caesar,

imperium Oceano, famam qui terminet astris,

Julius, a magno demissum nomen I[ulo.”—i, 278-288.

a) By whom were these words spoken? To whom? In what connection ?

b) Discuss verses 278-282 and 286-288 with reference to the purpose of the Aeneid. |

c) To what historical event do verses 284, 285 refer? Explain the references in Assaract, Phthiam, Mycenas ?

d) Who were Caesar (verse 286), Iulo (verse 288) ?

-e) What is the point of aspera (verse 279), Troianus (verse 286) ?

ft) What does Vergil mean by Oceano in verse 287?

2. “Heu nihil invitis fas quemquam fidere divis! Ecce trahebatur passis Priameia virgo crinibus a templo Cassandra adytisque Minervae, ad caelum tendens ardentia lumina frustra— 405 lumina, nam teneras arcebant vincula palmas. Non tulit hanc speciem furiata mente Coroebus, et sese medium iniecit periturus in agmen. Consequimur cuncti et densis incurrimus armis. Hic primum ex alto delubri culmine telis 410 _nostrorum obruimur, oriturque miserrima caedes armorum facie et Graiarum errore iubarum.’’—ii, 402-412. a) Explain the application of verse 4a2 to the situation. What was the errore (verse 412)? | b) Tell the story of Cassandra. c) Tell all you know about Minerva, and her part in the Trojan War. d) What word would regularly be used in prose instead of divis (verse 402) ? Change periturus (verse 408) to a common prose construction. e) Copy verses 409 and 411, and indicate the quantity of each syllable, the division into feet, and the principal caesuras. f) What is the subject of each of the first six books of the Aeneid? In what order should the books be arranged to show the chronological sequence of their contents?

go

“Nec non et Phrygii comites et laetus Iulus 140 incedunt. Ipse ante alios pulcherrimus omnis infert se socium Aeneas atque agmina iungit. Qualis ubi hibernam Lyciam Xanthique fluenta deserit ac Delum maternam invisit Apollo instauratque choros mixtique altaria circum 145 Cretesque Dryopesque fremunt pictique Agathyrsi, ipse iugis Cynthi graditur mollique fluentem fronde premit crinem fingens atque implicat auro, tela sonant umeris; haud illo segnior ibat Aeneas, tantum egregio decus enitet ore.” —iv, 140-150. 150

a) What was the occasion of the gathering described in verses 140-142?

b) Who are meant by Phrygii (verse 140), and why are they so called ?

c) What and where were Lyciam (verse 143), Delum (verse 144), Cynthi (yerse 147)?

d) What peculiarity of prosody is found in verse 146?

é) What was the fronde (verse 148)? What were the fela (verse 149) ?

gti

rt ' 7 ¥ . is - « ity ae ae $ fs , Fate ans >

*, Pa Tot a ae i E

LATIN M

ELEMENTARY SIGHT TRANSLATION OF PROSE

93

Wednesday

LATIN M—ELEMENTARY SIGHT TRANSLATION

OF PROSE

GALBA’S ACCOUNT OF THE BATTLE OF MUTINA

Cum equites Antonii apparuissent, contineri neque legio Mar- tia neque cohortes praetoriae potuerunt; quas sequi coepimus coacti, quoniam retinere eas non potueramus. Antonius ad Forum Gallorum suas copias continebat neque sciri volebat se legiones habere; tantum equitatum et levem armaturam ostendebat. Pos- teaquam vidit se invito legionem ire Pansa, sequi se duas legiones iussit tironum.' Posteaquam angustias paludis et silvarum transi- imus, acies est instructa a nobis XII cohortium; nondum venerant legiones duae: repente Antonius in aciem suas copias de vico pro- duxit et sine mora concurrit. Primo ita pugnatum est, ut acrius non posset ex utraque parte pugnari; etsi dexterius cornu, in quo ego eram cum Martiae legionis cohortibus octo, impetu primo fuga- verat legionem XXXV Antonii, ut amplius passus quingentos ultra aciem, quo loco steterat, processerit. Itaque cum equites nostrum cornu circumire vellent, recipere me coepi et levem arma- turam opponere Maurorum equitibus, ne aversos nostros adgre- derentur.—Cicero, Ad Familiares, x, 30, 2-3.

t tironum, from tiro, ‘a recruit.’

94

IQII

II.I5 a. M.—12.30 p. m.

Read the passage through two or three times before beginning to write the transla- tion. The translation should be exact, but in clear and natural English.

| Igr2 LATIN M—ELEMENTARY SIGHT TRANSLATION OF PROSE

Wednesday II.15 a. M.-12.30 p. m.

Read the passage through several times before beginning to write the translation. The translation should be exact, but in clear and natural English.

Si diligenter quid Mithridates potuerit et quid effecerit et qui vir fuerit con- sideraris, omnibus regibus quibuscum populus Romanus bellum gessit hunc regem nimirum? antepones. Qui rex sibi aliquot annis sumptis ad confirmandas rationes et copias belli tantum spe conatuque valuit ut se Oceanum cum Ponto, Sertori copias cum suis coniuncturum putaret. Ad quod bellum duobus con- sulibus ita missis ut alter Mithridatem persequeretur, alter Bithyniam tueretur, alterius? res et terra et mari calamitosae vehementer et opes regis et nomen auxerunt; L. Luculli vero res tantae exstiterunt ut neque maius bellum com- memorari possit neque maiore consilio et virtute gestum. Namcum totiusimpetus belli ad Cyzicenorum moenia constitisset? eamque urbem sibi Mithridates Asiae ianuam fore putasset, qua effracta et revulsa5 tota pateret provincia, perfecta a Lucullo haec sunt omnia ut urbs fidelissimorum sociorum defenderetur et omnes copiae regis diuturnitate obsessionis consumerentur.—Cicero, Pro Murena, 32, 33.

* nimirum, equivalent to profecto. ?alierius ... L. Luculli denote the same persons as alier . . alter. 3 constitisset, from consistere, ‘concentrate.’ 4ianuam, ‘doorway.’ 5revulsa, ‘torn away.’

95

1913 LATIN M—ELEMENTARY SIGHT TRANSLATION OF PROSE Thursday II.1I5 a M.-I p. m.

Read the passage through several times before beginning to write the translation. The translation should be exact, but in clear and natural English.

THE ROUT AT THE BATTLE OF THAPSUS

Interim Scipionis copiis prostratis passimque’ toto campo fugientibus con- festim Caesaris legiones consequi spatiumque se non dare colligendi. Qui post- quam ad ea castra quae petebant perfugerunt, ut refecti castris rursus sese de- fenderent ducemque aliquem requirerent, quem respicerent, cuius auctoritate imperioque rem gererent, et postquam animadverterunt neminem ibi esse prae- sidio, protinus? armis abiectis in regia castra fugere contendunt. Quo postquam pervenerunt, ea quoque ab Iulianis* teneri vident. Desperata salute in quodam colle consistunt atque armis demissis salutationem more militari faciunt. Quibus miseris ea res parvo praesidio fuit. Namque milites veterani ira et dolore incen- si non modo ut parcerent hosti non poterant adduci, sed etiam ex suo exercitu inlustres* urbanos complures aut vulnerarunt aut interfecerunt. In quo numero fuit Tullius Rufus quaestorius,5 qui pilo traiectus consulto® a milite interiit; item Pompeius Rufus bracchium’ gladio percussus,® nisi celeriter ad Caesarem accucurrisset, interfectus esset.—De Bello Africano, 85, 3-8.

t passim, ‘in every direction.’ ? protinus, ‘straight on.’ %Iulianis, the soldiers of Caesar.

‘inlustres, ‘distinguished.’ 5 guaesiorius, ‘an ex-quaestor.’ ®consulio, ‘deliberately.’ 7 bracchium, ‘arm.’ 8 percussus, ‘stabbed.’

96

1914 LATIN M—ELEMENTARY SIGHT TRANSLATION OF PROSE

Thursday II.15 a. M.-I p. m.

Read the passage through several times before beginning to write the translation. The translation should be exact, but in clear and natural English.

[Cicero praises Pompey in order to justify the course adopted by Deiotarus.|

Ignosce, ignosce, Caesar, si eius viri auctoritati rex Deiotarus cessit, quem nos omnes secuti sumus; ad quem cum di atque homines omnia ornamenta con- gessissent, tum tu ipse plurima et maxima. Neque enim, si tuae res gestae cete- rorum laudibus obscuritatem attulerunt, idcirco Cn. Pompei memoriam amisimus. Quantum nomen illius fuerit, quantae opes, quanta in omni genere bellorum gloria, quanti honores populi Romani, quanti senatus, quanti tui, quis ignorat? Tanto ille superiores vicerat gloria quanto tu omnibus praestitisti. Itaque Cn. Pompei bella, victorias, triumphos, consulatus admirantes numerabamus; tuos enumerare. non possumus. Ad eum igitur rex Deiotarus venit hoc misero fatalique bello, quem antea iustis hostilibusque bellis adiuverat, quocum erat non hospitio solum, verum etiam familiaritate coniunctus; et venit vel rogatus ut amicus, vel arcessi- tus ut socius, vel evocatus ut is qui senatui parere didicisset; postremo venit ut ad fugientem, non ut ad insequentem, id est ad periculi, non ad victoriae societatem.

Cicero, Pro Rege Deiotaro, 12, 13.

97

1915 LATIN M—ELEMENTARY SIGHT TRANSLATION OF PROSE

Wednesday Q-II a. m.

Read the passage through several times before beginning to write the translation. The translation should be exact, but in clear and natural English.

[Cicero puts Antony in the same class with Catiline and Clodius.]

Quonam meo fato, patres conscripti, fieri dicam ut nemo his annis viginti rei publicae fuerit hostis qui non bellum eodem tempore mihi quoque indixerit ? Nec vero necesse est quemquam a me nominari; vobiscum ipsi recordamini. Mihi poenarum illi plus quam optarem dederunt; te miror, Antoni, quorum facta imitere, eorum exitus non perhorrescere. Atque hoc in aliis minus mirabar. Nemo enim illorum inimicus mihi fuit voluntarius; omnes a me rei publicae causa lacessitii Tu ne verbo quidem violatus, ut audacior quam Catilina, furiosior quam Clodius viderere, ultro me maledictis lacessisti, tuamque a me alienationem commendationem tibi ad impios civis fore putavisti. Quid putem ? Contemptumne me? Non video nec in vita nec in gratia nec in rebus gestis nec in hac mea mediocritate ingeni quid despicere possit Antonius. An in senatu facillime de me detrahi posse credidit? Qui ordo clarissimis civibus bene gestae rei publicae testimonium multis, mihi uni conservatae dedit. Illud profecto: non existimavit sui similibus probari posse se esse hostem patriae, nisi mihi esset inimicus.—Cicero, Philippica ii, 1, 2.

98

a g ao fy te & & “a

IQII LATIN P—ADVANCED SIGHT TRANSLATION OF PROSE

Thursday Q-II a.m.

Read the passage through several times before beginning to write the translation. The translation should be exact, but in clear and natural English.

THE DEPARTURE OF HANNIBAL FROM ITALY AND THE RECEPTION OF THE NEWS AT ROME

Raro quemquam alium patriam exsilii causa relinquentem tam maestum abisse ferunt quam Hannibalem hostium terra exceden- tem. Respexisse saepe Italiae litora, et deos hominesque accusan- tem in se quoque ac suum ipsius caput exsecratum,' quod non cruen- tum ab Cannensi victoria militem Romam duxisset: Scipionem ire ad Carthaginem ausum, qui consul hostem Poenum in Italia non vidisset; se centum milibus armatorum ad Trasumennum, ad Cannas caesis circa Casilinum Cumasque et Nolam consenuisse.? Haec accusans querensque ex diutina possessione Italiae est de- tractus.

Romam per eosdem dies et Magonem et Hannibalem profectos adlatum est. Cuius duplicis gratulationis minuit laetitiam, et quod parum$ duces in retinendis iis, cum id mandatum ab senatu esset, aut animi aut virium habuisse videbantur, et quod solliciti erant, omni belli mole in unum exercitum ducemque inclinata,4 quo evasura esset res. .

| Mentio deinde ab senioribus facta est segnius homines bona quam

| mala sentire; transitu in Italiam Hannibalis quantum terroris

pavorisque esse meminisse! Quas deinde clades, quos luctus incidisse! Visa castra hostium e muris urbis; quae vota singulorum universorumque fuisse. Quotiens in conciliis voces manus.ad caelum porgentiums auditas, cn umquam ille dies futurus esset, quo vacuam hostibus Italiam bona pace florentem visuri essent!—Livy, Xxx, 20 and 21.

tExsecratum, from exsecrari, ‘to call down curses.’ *consenuisse, from consenescere, ‘to grow old.’ %parum, used here as a substantive. ‘inclinaia, from inclinare, ‘to turn.’ Sporgentium, from porgere, ‘to stretch.’

100

wat a OF

LATIN P—ADVANCED SIGHT TRANSLATION” OF PROSE

Thursday Q-II a.m.

Read the passage through several times before beginning to write the translation. The translation should be exact, but in clear and natural English.

IQI2

Tue ATTITUDE OF CICERO TOWARD THE DOMINATION OF OCTAVIANUS IN ROME

Si per tuas legiones mihi licitum fuisset, quae nomini meo populoque Romano sunt inimicissimae, venire in senatum coramque de re publica disputare, fecissem, neque tam libenter quam necessario; nulla enim remedia quae vulneribus adhibentur tam faciunt dolorem quam quae sunt salutaria. Sed ‘“quoniam cohortibus armatis circumsaeptus senatus nihil aliud vere potest decernere nisi se timere—in Capitolio signa sunt, in urbe milites vagantur, in Campo castra ponuntur, Italia tota legionibus ad libertatem nostram conscriptis, ad servitutem adductis equitatuque exterarum nationum distenetur—cedam tibi in praesentia foro, curia, et sanctissimis deorum immortalium templis, in quibus, reviviscente iam libertate, deinde rursus oppressa, senatus nihil consulitur, timet multa, adsentatur omnia. Post etiam paulo, temporibus ita postulantibus, cedam urbe, quam per me conservatam, ut esset libera, in servitute videre non potero; cedam vita, quae quamquam sollicita* est, tamen, si profutura est rei publicae, bona spe posteritatis? me consolatur, qua sublata non dubitanter occidam,3 atque ita cedam ut fortuna iudicio meo, non animus mihi defuisse videatur. Illud vero, quod et recentis doloris habet indicium et praeteritae iniuriae testimonium et absentium sensus? significationem,> non praetermittam, quin, quoniam coram id facere prohibeor, absens prosim, si quidem mea salus aut utilis rei publicae est aut coniuncta certe publicae saluti.—Pseudociceronis Epistula ad Octavianum, 1-2.

* sollicita, ‘full of trouble.’ ? posteritatis, ‘future glory.’. 3 occidam (notice the quantity of the 7). 4sensus, genitive. 5 significationem, ‘an indication.’

IoI

Roa ee 1913 “LATIN P--ADVANCED SIGHT TRANSLATION OF PROSE

Thursday Q-II a. m.

Read the passage through several times before beginning to write the translation. The translation should be exact, but in clear and natural English.

THE CONSUL COTTA ADDRESSES A CROWD OF ANGRY CITIZENS AND DECLARES THAT HE IS WHOLLY LOYAL TO THEIR INTERESTS

Quirites, multa mihi pericula domi militiaeque, multa adversa fuere, quorum alia toleravi, partim reppuli deorum auxiliis et virtute mea; in quibus omnibus numquam animus negotio defuit neque decretis labor. Malae secundaeque res opes, non ingenium mihi mutabant. At contra in his miseriis cuncta me cum fortuna deseruere. Praeterea senectus, per se gravis, curam duplicat, cui misero acta iam aetate ne mortem quidem honestam sperare licet. Nam si parricida vestri sum et deos penatis meos patriamque et summum imperium vilia* habeo, quis mihi vivo cruciatus satis est aut quae poena mortuo? Quin omnia memo- rata apud inferos supplicia scelere meo vici. A prima adulescentia in ore vestro privatus et in magistratibus egi. Qui lingua, qui consilio meo, qui pecunia voluere, usi sunt; neque ego callidam facundiam? neque ingenium ad male faci- undum exercui. Avidissimus privatae gratiae maximas inimicitias pro re pu- blica suscepi; quibus victus* cum egens alienae opis plura mala exspectarem, vos. Quirites, rursus mihi patriam deosque penatis cum ingenti dignitate dedistis. Pro quibus beneficiis -vix satis gratus videar, si singulis animam, quam nequeo,‘ concesserim.s Nam® vita et mors iura naturae sunt; ut sine dedecore? cum civibus fama et fortunis integer agas, id dono datur atque accipitur.—Sallust, Historiae, ii.

tyilia, ‘cheap.’ *callidam facundiam, ‘skilful eloquence.’ %victus. Cotta, the speaker, had been forced to go into exile. ‘4megueo, ‘I cannot.’ Sconcesserim, ‘sacrifice.’ °nam...- accipitur, explains guam nequeo. ‘dedecore, dishonor.

102

1914 LATIN P—ADVANCED SIGHT TRANSLATION

OF PROSE

Thursday Q-II a. mM.

Read the passage through several times before beginning to write the translation. The translation should be exact, but in clear and natural English.

[Hannibal’s Italian allies reproach him for his failure to come to their assistance.|

Eadem aestate Marcellus ab Nola’, quam praesidio obtinebat, crebras excur- siones in agrum Hirpinum et Samnites fecit, adeoque omnia ferro atque igni vastavit ut antiquarum cladium? memoriam renovaret. Itaque legati ad Hannibalem missi simul ex utraque gente ita Poenum adlocuti sunt: “Hostes populi Romani, Hannibal, fuimus primum per nos ipsi, quoad nostra arma, nostrae vires nos tutari poterant. Postquam iis parum fidebamus, Pyrrho regi nos adiunximus; a quo relicti pacem necessariam accepimus, fuimusque in ea per annos prope quinquaginta ad id tempus quo tu in Italiam venisti. Tua nos non magis virtus fortunaque quam unica’ comitas* ac benignitas erga cives nostfos, quos captos nobis remisisti, ita conciliavit tibi ut te salvo atque incolumi amico non modo populum Romanum sed ne deos quidem iratos, si fas’ est dici, timeremus. At hercule non solum incolumi et victore sed praesente te, cum ploratum® prope coniugum ac liberorum nostrorum exaudire et flagrantia tecta posses conspicere, ita sumus aliquotiens hac aestate devastati ut M. Marcellus, non Hannibal vicisse ad Cannas videatur. Causa autem haec est, quod neque tu defendis et nostra iuventus, quae, si domi esset, tutaretur, omnis sub signis militat tuis. Nec te nec exercitum tuum norim, nisi, a quo tot acies Romanas fusas stratasque’ esse sciam, ei facile esse ducam opprimere populatores nostros vagos® sine signis palatos?’ quo quemque trahit quamvis vana praedae spes. Numi- darum paucorum illi quidem praeda erunt, praesidiumque miseris simul nobis et Nolae ademeris, si modo quos, ut socios haberes, dignos duxisti, haud?® indignos iudicas quos in fidem receptos tuearis.’”’—Livy, xxiii, 41, 42.

*Nola, a city in Campania. ?cladium, ‘disasters.’ 3 unica, ‘singular.’ 4comitas, ‘courtesy.’

5 fas, ‘right.’ ° ploratum, ‘wailing.’ 7fusas stratasque, ‘routed and overthrown.’ 8vagos, equiva- lent to vagantes. 9 palatos, from palari, ‘to wander.’ * haud, ‘not.’

103

IQI5 LATIN P—ADVANCED SIGHT TRANSLATION OF PROSE

Thursday Q-II a. m.

Read the passage through several'times before beginning to write the translation. The translation should be exact, but in clear and natural English.

[Cicero describes the wisdom with which Aratus of Sicyon adjusted a difficult ques- tion of property rights.|

At vero Aratus Sicyonius iure laudatur, qui, cum eius civitas quinquaginta annos a tyrannis teneretur, profectus Argis' Sicyonem clandestino introitu urbe est potitus, cumque tyrannum Nicoclem improviso oppressisset, sescentos exsules, qui locupletissimi fuerant eius civitatis, restituit remque publicam adventu suo liberavit. Sed cum magnam animadverteret in bonis et possessioni- bus difficultatem, quod et eos quos ipse restituerat, quorum bona alii possederant, egere iniquissimum esse arbitrabatur et quinquaginta annorum possessiones moveri non nimis aequum putabat, propterea quod tam longo spatio multa hereditatibus,? multa emptionibus,3 multa dotibus+ tenebantur sine iniuria, iudicavit neque illis adimi nec iis non satis fieri quorum illa fuerant oportere. Cum igitur statuisset opus esse ad eam rem constituendam pecunia, Alexandriam se proficisci velle dixit remque integram ad reditum suum iussit esse, isque celeriter ad Ptolemaeum suum hospitem venit, qui tum regnabat alter post Alexandriam conditam. Cui cum exposuisset patriam se liberare velle causam- que docuisset, a rege opulento vir summus facile impetravit ut grandis pecunia adiuvaretur. Quam cum Sicyonem attulisset, adhibuit sibi in consilium quin- decim principes, cum quibus causas cognovit et eorum qui aliena tenebant et eorum qui sua amiserant, perfecitque aestimandis possessionibus ut persuaderet aliis ut pecuniam accipere mallent, possessionibus cederent, aliis ut commodius putarent numerari’ sibi quod tanti? esset quam suum recuperare. Ita perfectum est ut omnes concordia constituta sine querella® discederent. O virum magnum dignumque qui in re publica nostra natus esset! Sic par est agere cum civibus, non hastam? in foro ponere et bona civium voci subicere praeconis.” At ille Graecus, id quod fuit sapientis et praestantis viri, omnibus consulendum putavit.

Cicero, De Officiis, ii, 81-83.

* Argis, from Argi (pl.), ‘Argos.’ 2 hereditatibus, from hereditas, ‘inheritance.’ 3 emptionibus, from emptio, ‘purchase.’ 4dotibus, from dos, ‘dowry.’ 5 grandi, ‘large.’ © numerari, ‘to be paid.’ 7tanti, supply quanti possessiones amissae. * querella, ‘complaint.’ 9% hastam, ‘spear.’ 10 praeconis, ‘auctioneer.’

104

IQII LATIN Q—SIGHT TRANSLATION OF POETRY

Thursday I.30-3.30p. m.

Read the passage through several times before beginning to write the translation. The translation should be exact, but in clear and natural English.

Translate into English: THE SCENES REPRESENTED ON HANNIBAL’S SHIELD

Condebat primae Dido Carthaginis arces, instabatque operi subducta classe iuventus. Molibus hi claudunt portus, his tecta domosque partiris,‘ iustae Bitia venerande senectae. Ostentant caput effossa tellure repertum bellatoris equi atque omen clamore salutant. Has inter species orbatum? classe suisque Aenean pulsum pelago dextraque precantem cernere erat. Fronte hunc avide regina serena infelix ac iam vultu spectabat amico.

Hinc et speluncam furtivaque foedera amantum Callaicae‘ fecere manus; it clamor ad auras latratusque canum, subitoque exterrita nimbo occultant alae venantum corpora silvis.

Nec procul Aeneadum vacuo iam litore classis aequora nequiquam revocante petebat Elissa. Ipsa, pyram super ingentem stans, saucia Dido mandabat Tyriis ultricia bella futuris; ardentemque rogum media spectabat ab unda Dardanus et magnis pandebat carbasaé fatis. Parte alia supplex infernis Hannibal aris arcanum® Stygia libat cum vate cruorem

et primo bella Aeneadum iurabat ab.aevo.

At senior Siculis exultat Hamilcar in arvis— Spirantem credas certamina anhela movere, ardor inest oculis, torvumque minatur imago. Silius Italicus, Punica, II, 406-431. * Partiris =dividis. 2 Orbatum: bereft.

3 Erat: it was possible. 4Callaicae: Spanish.

5 Carbasa =vela.

6 Arcanum: secret (in reference to the rite).

106

IgI2

LATIN Q—SIGHT TRANSLATION OF POETRY Thursday I.30-3.30 p.m.

Read the passage through several times before beginning to write the translation. The translation should be exact, but in clear and natural English.

Kinc ANrus DESCRIBES THE WONDERFUL PowER Wuaicyu BaAccuus GAvE To His DAUGHTERS, AND TELLS How THE SAME DEITY SAVED THEM FROM THE ANGER OF ATRIDES

Tum pius Anchises: ‘‘O Phoebi lecte sacerdos, fallor, an et natum, cum primum haec moenia vidi, bisque duas natas, quantum reminiscor, habebas ?” Huic Anius niveis circumdata tempora vittis concutiens et tristis ait: ‘“Non falleris, heros maxime; vidisti natorum quinque parentem, quem nunc—tanta homines rerum inconstantia versat— paene vides orbum.* Quod enim mihi filius absens auxilium, quem dicta suo de nomine tellus Andros? habet, pro patre locumque et regna tenentem ? Delius augurium dedit huic; dedit altera Liber femineae stirpi voto maiora fideque munera. Nam tactu natarum cuncta mearum in segetem laticemque meri bacamque Minervae’ transformabantur, divesque erat usus¢ in illis. Hoc ubi cognovit Troiae populator Atrides (ne non ex aliqua vestram sensisse procellam nos quoque parte putes), armorum viribus usus abstrahit invitas gremio genitoris, alantques imperat Argolicam caelesti minere classem. Effugiunt quo quaeque potest: Euboea duabus,

- et totidem natis Andros fraterna petita est. Miles adest et, ni dedantur, bella minatur. Victa metu pietas consortia® pectora poenae dedidit. Et timido possis ignoscere fratri. Non hic Aeneas, non, qui defenderet Andron, Hector erat, per quem decimum durastis in annum. Iamque parabantur captivis vincla lacertis. Illae tollentes etiamnum?’ libera caelo bracchia ‘Bacche pater, fer opem!’ dixere. Tulitque muneris auctor opem—si miro perdere more ferre vocatur opem. Nec qua ratione figuram perdiderint, potui scire aut nunc dicere possum. Summa mali nota est: pennas sumpsere, tuaeque coniugis in volucres niveas abiere columbas.”’

Ovid, Metamorphoses, xiii, 640-674.

t orbum, ‘childless.’ * Andros, nameof anisland. %bacam Minervae, equivalent to olivam.

4usus, ‘gain.’ 5 alant, depends upon imperai. 6 consortia pectora, equivalent to sorores. 7cliamnum, equivalent to etiam nunc.

107

IQI LATIN Q—SIGHT TRANSLATION OF POETRY > Thursday 2-4 p.m.

Read the passage through several times before beginning to write the translation. The translation should be exact, but in clear and natural English.

EVANDER, KING OF ARCADIA, AS HE SENDS FORTH HIS SON PALLAS TO WAR, ENTREATS THE GODS TO BE MERCIFUL

Fama volat parvam subito vulgata? per urbem, ocius ire equites Tyrrheni ad litora regis. 555 Vota metu duplicant matres, propiusque periclo it timor, et maior Martis iam apparet imago. Tum pater Evandrus dextram complexus euntis haeret, inexpletus? lacrimans, ac talia fatur: “O mihi praeteritos referat si Iuppiter annos, 560 qualis eram cum primam aciem Praeneste? sub ipsa stravi scutorumque incendi victor acervos, et regem hac Erulum dextra sub Tartara misi, nascenti cui tris animas* Feronia mater (horrendum dictu) dederat, terna arma movenda 565 (ter leto sternendus erat; cui tum tamen omnis abstulit haec animas dextra et totidem exuit armis), non. ego nunc dulci amplexu divellerer usquam, nate, tuo; neque finitimo Mezentius umquam huic capiti insultans tot ferro saeva dedisset 570 funera, tam multis viduasset’ civibus urbem. At vos, O superi, et divom tu maxume rector Iuppiter, Arcadii, quaeso, miserescite regis et patrias audite preces. Si numina vestra incolumem Pallanta mihi, si fata reservant, 575 si visurus eum vivo et venturus in unum, vitam oro, patior quemvis durare® laborem. Sin aliquem infandum casum, Fortuna, minaris, nunc, nunc O liceat crudelem abrumpere vitam, dum curae ambiguae, dum spes incerta futuri, 580 dum te, care puer, mea sola et sera voluptas, complexu teneo; gravior neu nuntius auris vulneret.” Haec genitor digressu dicta supremo fundebat; famuli conlapsum in tecta ferebant. Aeneid, viii, 554-584. yulgata, ‘spread abroad.’ ?inexpletus, ‘unsatisfied.’ 3Praenesie, nameofatown. ‘4animas, ‘lives.’ Sviduasset, from viduare, ‘to bereave.’ °durare, ‘endure.’

108

LATIN Q—SIGHT TRANSLATION OF POETRY

Thursday

1914

2-4 p.m,

Read the passage through several times before beginning to write the translation. The translation should be exact, but in clear and natural English.

[The Trojans mourn Hector.

his son.

King Priam begs Achilles to give him the body of The funeral is described.]

Flent miseri amissum Phryges Hectora, totaque maeste Troia sonat planctu’. Fundit miseranda querelas? infelix Hecube3 saevisque arat unguibus? ora, Andromacheque suas scindit de pectore vestes, heu tanto spoliata viro! Ruit omnis in uno Hectore causa Phrygum; ruit et defessa senectus adflicti miseranda patris, quem nec sua coniunx turbaque natorum nec magni gloria regni

oblitum tenuit vitae, quin iret inermis

et solum invicti castris se redderet hostis. Mirantur Danaum proceres, miratur et ipse Aeacides animum miseri senis. Ille trementes,

_ adfusus’ genibus, tendens ad sidera palmas

haec ait: ‘‘O Graiae gentis fortissime Achilles, O regnis inimice meis, te Dardana solum victa tremit pubes, te sensit nostra senectus crudelem nimium. Nunc sis mitissimus,® oro, et patris adflicti genibus miserere precantis donaque quae porto miseri pro corpore nati accipias; si nec precibus nec flecteris auro,

in senis extremis

tua dextera saeviat annis;

saltem scaeva’ pater comitabor funera nati. Non vitam mihi nec magnos concede favores, sed funus crudele mei. Miserere parentis

et pater esse meo mitis de vulnere disce. Hectoris interitu vicisti Dardana regna,

vicisti Priamum;

sortis reminiscere® victor

humanae variosque ducum tu respice casus.”’

His tandem precibus grandaevum motus Achilles adlevat a terra corpusque exsangue parenti reddidit Hectoreum, post haec sua dona reportat. It patriam Priamus tristisque ex more suorum comparat exsequias? supremaque funera ducit. Tum pyra construitur, quo bis sex corpora Graium quadrupedesque adduntur equi currusque tubaeque et clipei galeaeque ocreaeque” Argivaque tela. Haec super ingenti gemitu componitur Hector.

Ilias Latina, 1015-1051.

t planctu, ‘lamentation.’ *guerelas, ‘complaints.’ 3 Hecube, equivalent to Hecuba. 4ungui-

bus, ‘nails.’

5 adfusus, ‘casting: himself down.’ 6 mitissimus, equivalent to lenissimus. 7scaeva,

‘mournful.’ %reminiscere, ‘remember.’ 9% exsequias, ‘obsequies.’ ocreae, ‘greaves.’

Trt

1915 LATIN Q—SIGHT TRANSLATION OF POETRY

Thursday 2-4 p.m.

Read each passage through several times before beginning to write the translation of it. The translation should be exact, but in clear and natural English.

[Aristaeus in distress reproaches his mother.|

Pastor Aristaeus fugiens Peneia Tempe," amissis, ut fama, apibus morboque? fameque, tristis ad extremi sacrum caput adstitit amnis, multa querens, atque hac adfatus voce parentem: “Mater, Cyrene mater, quae gurgitis huius ima tenes, quid me praeclara stirpe deorum (si modo, quem perhibes,3 pater est Thymbraeus Apollo) invisum fatis genuisti? Aut quo tibi nostri pulsus amor? Quid me caelum sperare iubebas ? En etiam hunc ipsum vitae mortalis honorem, quem mihi vix frugum et pecudum custodia sollers* omnia temptanti extuderat,5 te matre relinquo. Quin age et ipsa manu felicis erue silvas, fer stabulis inimicum ignem atque interfice messis,® ure sata, et validam in vitis’ molire bipennem, tanta meae si te ceperunt taedia laudis.” Vergil, Georgics, iv, 317-332. tT, empe, plur. neut., name of the valley through which the Peneus flows. ? morbo, ‘disease.’

erhibes, sollers, ‘skilful.’ 5 extuderat, from extundo, ‘work out. SSI. k 3 hib ‘assert.’ 4 sollers, ‘skilful.’ 5 extuderat, f tundo, ‘work out.’ © messis, ‘harvests.’ 7 vitis, ‘vines.’

[The captive Trojan women say their sad farewell to home and country.|

Ilion ardebat, neque adhuc consederat ignis;

exiguumque’ senis Priami Iovis ara cruorem

combiberat. Tractata? comis antistita’ Phoebi

non profecturas‘ tendebat ad aethera palmas.

Dardanidas matres patriorum signa’ deorum,

dum licet, amplexas succensaque templa tenentis

invidiosa trahunt victores praemia Grai.

Mittitur® Astyanax illis de turribus unde

pugnantem pro se proavitaque’ regna tuentem

saepe videre patrem monstratum a matre solebat.

Iamque viam suadet Boreas, flatuque® secundo

carbasa? mota sonant. Iubet uti navita™ ventis.

“Troia, vale! Rapimur” clamant, dant oscula terrae

Troades, et patriae fumantia tecta relinquunt.

Ultima conscendit classem, miserabile visu,

in mediis Hecuba natorum inventa sepulcris.

Prensantem tumiulos atque ossibus oscula dantem

Dulichiae™ traxere manus. Tamen unius hausit”

inque sinu cineres secum tulit Hectoris haustos.

Ovid, Metamorphoses, xiii, 408-426. t exiguum, ‘scanty.’ ? tractata, frequentative, or intensive, from traho. 3 antistita, ‘priest-

ess.’ 4 profecturas, from proficio, ‘avail.’ 5 sig gna, equivalent to simulacra. ° mittitur, equivalent to deicitur. 7 proavita, ‘of his forefathers.’ * flatu, equivalent to vento. ° carbasa,

equivalent to vela. ™navita, equivalent to mnauta. Dulichiae, adjective, ‘of Ulysses.’

12 hausit, ‘scraped up.’ a

ae aed a FO

“e . J poe

IQII GREEK A/—-GRAMMAR

Saturday II.15 a M.-12 m.

Do not translate the following passage, but answer the questions in order.

‘“"Av8pes, Edy pou mee Ofjre, ovTE KLVOUVEVoOuYTES OUTE TOVHTAVTES TOV GAXwY Tréov

mpoTinjrerOe otparwrav id Kipov. th ody Kedetw rovgoar; viv dSetrar Kipos emer Oar Tous “EAAnvas éri Baciréa eyw otv dye buds xphvar SiaBAvar tov Eddpd- THY ToTapov, mpiv dpArov elvan 6 TL ot GAAOL "EAAnVes GroKpwwotvrar Kipw. jv pev yap Uyndiowvras ererOar, duets Sdgere alrior evar aptavres rod diaBaiverv, Kal ds mpoOvmordrois ovow tpiv xapw eioerar Kipos kal drodace: ériotarat 8 et Tis Kal GdAos: Hv & arolndpiowvras of GAO, Amipev pev dravres eis ToUmradw, piv 8’ ds povois TeBouevors mitToTaTous XpHTETaL Kal eis Ppovpia Kal eis Aoxaylas, Kal BAov obrivos dv dénade oid’ Ste ds piror redgeobe Kvpov.”—Xenophon, Anabasis, I, 4, 14-15. a. Give the dative singular and plural of dvdpes (line 1); the genitive and voca- tive singular of orparwrév (line 2); the genitive singular of xépw (line 6); the dative plural, in all genders, of dpgavres (line 5); decline 7r/ (line 2) in all genders, in the plural only.

Oo On AM PW ND HF

b. Compare zAé<ov (line 1), airio (line 5), and zpoOvpordros (line 6).

c. Give the second person singular, in all moods, of rewo67re (line 1) in this voice and tense only, also the infinitive and the nominative plural of the parti- ciple, in all genders; give the second person plural, in all moods, of the present of dmev (line 7), together with the infinitive and the nominative singular of the participle, in all genders.

d. Inflect the plural of the subjunctive aorist active of xeAevw (line 2); also the singular of the optative present active of mpotiujoeoGe (line 2), giving contracted forms only. :

e. Give the principal parts of érecOa (line 3); droxpwodvra (line 4); TevéerGe (line 9).

f. Account for the case of orpatwrdyv (line 2); tiv (line 7); aAAov (line 8).

g. Account for the mood of weo@re (line 1); SuaPpvar (line 3); «tvac (line 4). Explain the force of #s (line 5).

h. Explain the use of the supplementary participle, and illustrate by short examples in Greek with English translations.

I12

IgI2

GREEK A/—-GRAMMAR

Saturday II.1I5 a. M.—12 m.

SS ON AM PW WD H

Do not translate the following passage, but answer the questions in order.

kat év tovtw Kipos mapedatvev airos ctv Iiypyte To éppnvet Kai dAdo Tpioiv } Tértapor TO KXcapyw Boa Gye 7d orparevpa Kara peor Tb TOV TOAELiwV, OTL Exel Bacrreds cin. Kav TovT’, en, vikdpev, wav’ hytv werointa. sSpav 6 KA€apxos 7d pécov aridos Kal dxotwv Kipov gf dvra rod EAAnviKod etwvipuov BactAéa—rooodrov yap wAHVa repinv Bacireds hore pécov TOV EavTOD Exwv TOD Kipov ciwvipov ew Rv—éAN’ Spws 6 Kreapyxos ovk HOeXev droordca ard TOD woTapod 7d SeEvdv Képas, oPotpevos 7) KvKAwVe(n Exarcpwhev, TO SE Kvpw daexplvaro OTe aiT@ péAo drws

kas €xot.— Xenophon, Anabasis, I, 8, 12-13.

. Give the genitive singular and dative plural of €pyyvet (line 1); the nomina-

tive plural of oridos (line 4); the genitive singular of «épas (line 6); the genitive plural, in all genders, of zav6" (line 3); the dative singular, in all genders, of dxovwv (line 4).

. Compare the adjective of xadd@s (line 8), giving the nominative singular, in

all genders.

. Give the second person singular of the second aorist active of dyew (line 2)

in the several moods, adding the infinitive and the nominative singular of the participle in all the genders; also the third person singular of the future of dzexpivaro (line 7) in the moods where it is found.

. Inflect the optative present active of €8da (line 2), and the imperative present

middle of ¢oBovpevos (line 7), giving contracted forms only.

. Give the principal parts of zapeAatvev (line 1); of Spay (line 3); of eyo

(line 8).

Give the third person plural present indicative, the second person plural present optative, and the present infinitive of é¢7 (line 3).

. Account for the case of jpiv (line 3); Kvpov (line 4); 7A7Oe (line 5).

. Change the words xav vxGpev (line 3) to the less vivid (ideal) form; account

for the construction of dvra (line 4); for the mood of kvxAwecy (line 7).

. What are the different ways of expressing a result clause in Greek, and what is

the difference in meaning between them? [Illustrate by short Greek sen- tences with English translation.

113

GREEK AL-GRAMMAR a

Saturday II.I5 a. M.—12.15 p. m. Do not translate the following passage, but answer the questions in order.

"QO Gvbpes “EdAnvess, ove GvOpdtav aropay BapSaper cupydyous tpas Gye, adAa vopiley apeivous xai xpeitrovs wohhGv BapBapev ipas etvai, dia tTovTo wpocdéAafov. Gras civ weobe dvips Gio Hs Aa Gepias Hs wéxtynote xai Fs ipas eyo eddaipovite. ef yap tote Sn TH Devieplar Daisy to tol Sc bugs ata aa <a Gras 32 nal cdijre cis olov tpyeaBe GyGva, eye ipas cides Sida—w. 7d péy yap whijBes woAv, xal cpavyy wokAy eriacw- Gy Taiz ava- 75 XSpq Svras GxOpinrovs—Xenophon, Anabasis, I, 7, 3-4.

a. Give the genitive plural of dvdpes (line 1); the dative plural of ayava (line 6); the pzttive cingular ond eonaeciiee es or oe the accusative singular in all genders of wodAwp (line 2); the genitive singular in all genders of ravrwy (line 5); the dative singular of ipas (line 2); the nominative plural of #pé (line 8).

5. Compare wokAey (line 2); a£coz (line 3).

c. Give the third person singular, in all moods, of €Ao/pnp (line 5), in this tense only, in the active voice; also the infmitive and the nominative plural of the participle im all genders. Give the second person singular, in all moods, of iove (line 4), in this tense; also the infmitive and the nominative plural of the participle im all genders.

d. Infiect the aorist active of &:daf@ (line 6) in the subjunctive plural; the indicative singular of the present of «éxtyo@e (line 4), giving contracted forms only.

e. Give the principal parts of dye (line 2); €Aoiunp (line 5); épyeoGe (line 6); yweceste (line 8).

f. Account for the case of av@pa@rer (line 1); BapBdpwr (line 2); Hs (line 4, after €hevBepias); jpiv (line 8).

g- Account for the seed of eet Ot 3 Se eee avacxnoGe (line 7)-

kk. What constructions are used with xpi? Give two illustrative examples in

oOo won AU & WwW WN

ri4

IQ14 GREEK A/GRAMMAR

Saturday II.I5 a M.-12.15 p. m. Do not translate the following passage, but answer the questions in order.

“Tlapexadeoa buds, dvdpes Piro, drrws ody viv Bovrevdpevos 6 Tt Sikadév ot Kal mpds Oe@v Kal mpds avOpworrwv, TodTO TPdEw rept ’OpdvTa TouTovi: TOUTOY yap TpaTov Mev 6 Euos TATHP dwxev ITrHKOOov eEivar epol- émel d€ TAXOEls, ws ehn avdTds, bd TOD euod adeXhod odTos éroddunce éuol xv tiv év Sdpdcow axpdtodw Kal éyo atv mpootorcuav érroinoa ate S6—at ToUT@ TOU pos eve TrOhepov TravoacbaL, Kal SeELav €haBov cal édwxa.” peta tadta épn, “& ’Opdvta, éotiv 6 Tt ce PEikyoa;” atrekpivato, dtc ov. mad & 6 Kipos jpora: “OvdKody tatepov, ws od Opororyeis, ovS€v Um’ éuod adicovpevos, AtTroaTAS ets Mucods Kak&s ézroles

Oo om Aun PW DN H

THv éunv yopav 6 TL édtva;” &pn 6 ’Opeortas. —Xenophon, Anabasis, I, 6, 6-7.

oa .@)

a) Decline in the plural zarjp (line 3), axpdvoAw (line 5), éuoé (line 3); in the singular, in all genders, ovdér (line g), 6 7 (line 10), taxGets (line 4).

b) Compare Séxatoy (line 2) in the nominative singular, m all genders; compare kaxas (line Q).

c) Give the second singular of éAaGov (line 7), in this tense and voice, in the several moods, adding the infinitive, and the nominative singular of the par- ticiple in all genders; give the second plural, present tense, of éoré (line 2) in the several moods, adding the infinitive, and the participle in the nominative masculine only; inflect dwoords (line g) in this tense, in the indicative and optative moods.

d) Inflect the plural of the optative aorist active of Sdfa: (line 6); the singular imperfect middle of #3¢kyoa (line 7), giving contracted forms only.

e) Give the principal parts of wapexdAcoa (line 1); of taxis (line 4); of exov (line 5); Of daexpivaro (line 8).

f) Account for the case of irijxoov (line 3); of ddeAgod (line 4); of euoé (line 5); of 7roA€uov (line 6).

g) Account for the mood of zpdéw (line 2); of dda (line 6); of ravcacOau (line 6).

hk) How is a conditional sentence of the so-called more vivid future type expressed in Greek? State the chief constructions in which the participle may occur.

115-

IQI5 GREEK A/—GRAMMAR

Saturday II.I5 a. M.-12.15 p. m.

© osr Amn fh W ND EH

eH FF SO FE O

a)

b)

af

d)

f)

g)

h)

Do not translate the following passage, but answer the questions in order.

Kal 75n te Hv wept rAnOoveav ayopay Kal épyovtar Tapa Baciréws kal Ticoadépvous kypuxes, of ev adXror BapBapa, jv 8 av’tav Parivos els “EXAnv, ds érvyyave trapa Ticcadépva av Kal évtivws éywv: kal yap mpocerroeito éricTHpov eivar TOV audi Tafes Te Kal oTOmayiar. oto. O€ mpocedOdvtes Kal KarécavTes Tors TOV “EXAHnVaV apyovTas Aéyouow St Bacireds Kerdever TOs “EAAnvas, érel WKaV TYyYaveE Kal Kipov aréxtove, wmapaddvtas Ta Stra idvtas él tas Bacidéws Ovpas

e "3 A £ b , lal \ 3 e / evploxecOar av tt Stvwvta. ayabov. Ttadta peéy eitrov of Bacidéws KnpuKes+ of 6€"EXAnves Bapéws pev jKovoav bus Kré€apyos tocod- Tov elev Ott OV TOV WK@VTaV ein TA SAA TrapadiddvaL: “adN’,” Sdn, acs, a / > » 5 / / > / ral 4 U Xx /

vpeis pev, @ avdpes oTpatnyol, TovTOWS amoKpivacbe 6 TL KAANTOV TE Kal dpiotov éyete* éyw avtixa HEw.”’—Anabasis, II, 1, 7-9.

Decline in the singular Baovdéws (line 1), ra£ers (line 4); in the singular, all genders, kadéoavres (line 5), odrou (line 5); in the plural, in all genders, ruxdv (line 6); in all genders, eis (line 3).

Give, in the nominative singular masculine, two comparatives and two superlatives of dyadév (line 8); the comparative of xa\\oror (line 11); give the adverbs for the three degrees of comparison of xa\\oTor.

Give the third person plural, in all the moods, of the aorist active of ériyxave (line 3); also the third person plural, in all the moods, in this tense, of rapa- dovras (line 7). In both instances write also the corresponding infinitive, and the nominative singular, in all genders, of the participle.

Give the principal parts of dméxrove (line 7); #Kovoay (line 9); éxere (line 12).

Inflect the present middle subjunctive plural of xadéoavres (line 5), giving contracted forms only; inflect the first aorist active indicative and optative of Néyovorr (line 6) in the singular; the subjunctive and optative plural of mpogeNOovres (line 5) in the same tense as the participle.

Account for the case of air@y (line 2); rév (line 4); “EAAnvas (line 6); rovrots (line 11).

Account for the mood of mapaduddvar (line 10); divwvra (line 8); ety (line ro).

Write short Greek sentences, with an English translation, to illustrate three

different ways of expressing purpose. 116

GREEK A 2

ELEMENTARY PROSE COMPOSITION

117

IQII

GREEK A/JI—ELEMENTARY PROSE COMPOSITION

Saturday ‘12 M.-12.30 p. m.

Translate into Greek:

I.

2.

All the chariots were made of the same wood.

He saw that in the beginning the soldiers were not willing to fight against th horsemen. : 3

. The arrows with which the enemy hit them were larger than those of the bar-

barians. Obey the general, but do not trust the satrap. You must cross a river which is three stadia in width. Clearchus evidently benefited the king, so that he gave him valuable presents.

118

1912 GREEK AJI-ELEMENTARY PROSE COMPOSITION

Saturday I2 M.-1I2.30 p. m.

Translate into Greek: 1. The soldiers said they would not obey the captain. 2. A river ten feet in depth flows through the middle of the village.

3. Cyrus promised to lead the Greeks back by the quickest road and to give them whatever they wished.

4. If you had done this, you would have died within three days. 5. Although Xenophon was an Athenian, they elected him general. 6. Do not proceed to the city, but let us remain on this very large island.

119

1913 GREEK A2—ELEMENTARY PROSE COMPOSITION

Saturday. I2.15-I p.m,

Translate into Greek:

. The Greeks secretly crossed the river to fight with the barbarians.

. Whenever we were in that city, the citizens admired our beautiful shields. . When they had heard these things, they returr~* to the camp.

. If you stop fighting before the enemy are defeated, you will deserve death.

. Why should we be willing to die for the king? Is it not evident that he is hostile to us?

6. They were on a large plain, so that those on the hill could easily see them.

mn & WwW ND we

120

1914 GREEK A2—ELEMENTARY PROSE COMPOSITION Saturday I2.15-I p.m.

Translate into Greek:

1. The general himself led the hoplites to the top of the mountain.

2. These villages were not far from a river, Kydnos by name.

3. They marched forward quickly on this day that they might arrive at a city and get provisions.

4. When the captain had come, he blamed (airidouar) the soldiers because they were going to flee.

5. A messenger told him that the ery were near and that the Greeks ought to attack them.

6. If you would give more money to the men, they would follow you.

7. The Greek soldiers are better than those whom the great king is sending against them.

121

IQI5 GREEK AJ/—ELEMENTARY PROSE COMPOSITION Saturday 3 . *F2. 15=3 p. mm,

Translate into Greek:

1. All those present said the same things about the army of the Greeks. 2. Because the country is hostile, we must have our arms that we may proceed through it in safety.

3. If the general brings us the ships, we shall use them.

4. Xenophon, who was leading the hoplites, commanded them to cross the river by means of this bridge.

. The soldiers said that they did not wish to make war against the great king.

. Fearing that they would not have provisions, they remained only two days in that place.

7. Whenever he got money, he gave it to his men.

On vi

122

GREEK B

XENOPHON ANABASIS, BOOKS I-IV

123

IQII

GREEK B—ANABASIS, I-IV

Saturday Q-II a. m.

The translation should be exact, but clear and idiomatic English is required. Number your answers.

1. Translate into English:

Otro éXeyov dtu Kipos pév réOvyxev, ‘Apiaios redevyws ev TH orabua ety peta TOV GAdwv BapBdpwv bev TH mpoTEepaiga Opynvro, Kal Aéyou Stu Tavrnv pev THV Huepav mepypevorev adtovs ei pwéeAAovev Hew, TH SE GAAH amrévar haiy er "Iwvias, 6Oevrep HADe. adr’ axotoavres of oTparyyol Kal of dAAo “EAAnves muvOavépevor Bapéws epepov. Kréapyos rade elev: “’AAX’ Shere ptv Kdpos tiv’ eel dt rereXedryxev, dmayyeAXAere “Apuaip ote qucis vixGpev te Bacrréa, Kal ds Spare ovdeis Ere Hpiv pdxerar, kal ef wy tues HAOere, exropevdpel? Gv ext Baoiréa, émayyeAdopucBa St “Apiain, éav évOdde Oy, cis tov Opdvov tov Baci-

Aevov Kabteiy abrov. Tov yap mdaxy ViKovTwV Kal Td dpxev éori.”—IT, 1, 3-4.

Oo Ons Aun HhW DN H

a) Account for the mood of «iy (line 1); of 7Aere (line 7). Account for the case of iv (line 7).

b) What would émi “Iwviav have meant (see lines 3 and 4)? Explain the difference in tense between dxovcavres (line 4) and vvGavopevor (line 5). Write in Greek another way of expressing the thought given by the words dere Lv (lines 5 and 6). .

c) Who was Ariaeus? How did he treat the Greeks afterward ?

2. Translate into English:

"EXOav 8 6 Hevodav éryjpero tov ArddAw rin av Gedy Ovwv Kal edydpevos Kadduora Kat dpurta €XOo. tHv 68dv Hv emivoel Kal KarA@s mpdgas owbe’y. Kat dveitev aitd 6 “ArddAwv Deois ols eer Ovew. ewel SE war HAGE, Ayer TH pavreiav TO Swxpdre. 6 8 dxovoas yTato aitov Stu ov TovTO mp@TOv ypwra morepov Adov cin abtS wopeverOar } péverv, GAN’ adrds kpivas iréov elvar Todr’ érvvOdvero bras Gv KdAdiora wopevdetn. “erred pevTo OVTWS pov, Tadr’,” Edn, “yon moveiv doa 6 Beds éxéXevoev.”—ITI, 1, 6-7.

Iam kh W DD HF

a) Account for the case of 65dv (line 2); of air@ (line 5). Explain the use of av in dzws av mopevOe‘n (line 6). What form is jpov (line 6) ?

b) What does this incident show in regard to Xenophon’s religious attitude ? Cite another instance which shows this attitude.

124

3. Translate into English:

I 2 3 4 5 6 7 8 9

*EvOa 8) mpocepxera Revopavre tv weAtacrav avnp AOjvynct pacKxwr Se5ov- Aevkévar, A€ywv Ste yryvwoKo. THY hwvyv Tov avOperwv. “Kal olpaz,”’ Edy, “eunv ratirny marpioa elvare Kal ef py Te KwAver Dew adrois SiadrexOAva.” "ANN obdev KwAve,” Edy, GAAG Suareyou Kai wae mp@rov tives ciciv.” of 8 elrov épwrficavros Gre Maxpwves, “"Epwra roivy,” épy, ‘‘abrots ti avrurerd- Xaror Kal xpyCovorw Huiv modeuor elvar.” of 8 dmexpivavro, “"Ore kat ipeis éxi THv Hperepav xwpav EpxerOe.” Eye Ex€eAevov of orpatyyot dtu ‘od KaKds ye moujcovres GANG Bacrre? roreujoavres &mrepxspeda cis Tv “EAAdOa, kai ert Odrarrav Bovrcucba adikéobu.”—IV, 8, 4-6.

a) Account for the lack of the article after ravryv (line 3). What is the

construction of épwrycavros (line 5)? Give the ordinary Attic form for avrirerdxara (line 5).

b) What is the derivation of SeSovAcvxevox (line 1)? What is the force of

the preposition in dzepxduefa. (line 8) ?

125

| 1912 GREEK B—ANABASIS I-IV | Saturday Q-II a.m.

The translation should be exact, but clear and idiomatic English is required. Number your answers.

1

. Translate into English:

“"Avopes, édv por reoOnre, ovTe Kwduvedoavres obTE TOVHTAYTES TOV GAAwV TAé€ov mpotisnoerbe otpariwtdv ims Kupov, ti otv Kedciw roujoa; viv Seirar Kipos éreo Oat Tovs "EXAnvas ext Baorrtéa eyo odv Pye tas xpnvas dua Bjvae tov Eidpdryvy motapov, mplv Sfrov elvar 6 te of GAAoL “EAAnves dmoxpwodvra. Kipw. jv pev yap Undbiowvra ErecOar, tucis Sdkere alti. eivar dpSavres Tod SiaBatve, Kal ds tpoOvpordros ovow tyiv yxapw eloetar Kdpos Kal drodéca: éricrara 8 & Tis Kal GAAos. Hv 8 drofndiowvrar of GAAor, amriev Gravres Totpradw.’—I, 4, 14-15.

a) Account for the case of orpatwrév (line 2); of tod SiaBaivew (line 5).

Account for the mood of dzoyndpicwvra: (line 7). Change the clause zpiv

d7Aov eivar (line 4) to the construction required after ob dyye xpqvar.

I AN Lh W ND HF

b) What does Xenophon say in his characterization of the Greek generals about the man (Menon) who speaks these words ?

2. Translate into English:

IIpofevos 8 65 Bowitios edfis pev peipdxrov dv éreOiwe yeveoOo. dvip ra peydra mparrey ixavds: Kat dua tavryy thy ériOupiav edwxe Topyia dpyvpiov To Neovrivw. éxel St cvveyéveto éxeivw ixavds vouioas Hon elves Kat Gpxew kat iros dv Tois mpwTos py yTTAcOo evepyerov, AVE cis Tatras Tas civ Kipw mpdées* Kal wero nrhorerOar éx TovTwv Ovoua méya Kal Sivapuv peydAnv Kal xpypata mo\Ad* Tocottwv & éribvpov opddpa evdnrov ad kal Todro elxev Ste TovTw ovdev dv Oedou kraoOat per’ adixias, GAAL civ TO Sixaiw Kai KarAG gero Seiv TovTwY TvyxXd- vew, dvev rovrwov wy.—II, 6, 16-18.

On Aum kW NH

a) In which case would the subject of «rjocoOa (line 5) be, if expressed ?

b) What relations existed between Proxenus and Xenophon ?

126

3. Translate into English:

I “Grap ti ey aepl KAomns oupBdArAAoua; tpads yap eywye, & Xeupicode, dxovw Tovs Aaxedarpoviovs, door éote T&V Spoiwv, edOis ex waidwy KAErTew pere Tav, Kal ovK aioypov eivat GAAG Kaddv KAErre doa py KwAVEL VOpos* ws SE ds Kpaticta KAértyTe Kal mepacbe AavOdvev, vourpov wap’ tuiv éoti, éav AnPOnTeE Kérrovtes, pactiyotaOa. viv ovv para cou Kaipds éotw éemidei~acIat THY mul- deiav, Kai Hrrdsacbar pi) AnPOGpev KrErrovtes TOV Gpovs, Hs wy TAYyas AdBopev.”

"ANG perro,” Eby b Xeipicodos, “Kayo buds Tors “APyvaiovs dxotw Sewvors elvan KAerrey TH Sypsora, Kal para Gvros Sevvod Tod Kivdivov Te KAETTOVTL, Kal Tovs KpatioTovs péevTor pdALoTa, elrep piv of KpaTicro dpxewv Afotvrar, wore

@pa kat cot érideixvvcbae tiv mradeiav.””—IV, 6, 14-16.

a) Give the derivation of Syudou. (line 8) and of dgutvrae (line 9), showing the force of the component parts.

90D ON ANP W WN

-

b) Account for the case of iptv (line 9). c) State briefly what led up to this banter of the generals.

127

GREEK B—ANABASIS, I-IV ee

Friday 4.15-6 p. m.

The translation should be exact, but clear and idiomatic English is required. Number

your answers.

1. Translate into English:

oOo ON AM hW HN HH

6 II pdEevos—értuyxe yap tortepos tpocidv Kal Takis adT@ érropévyn TOV OTALTa@V—EVOdS odV Eis TO pécoV audhoTépwr ayav OeTO TA Orda kal édeito ToD KiXedpxov ph moveity tadta. 5 8 éyadérauver 6tt adTod driyou Sejoavtos Katadevo Ova tpdws héyou TO avTOD mabos, éxédevod 7 avtov é« Tod péoou éEictaca. év tovtTm 8 émyer kal Kipos kat érvdeto TO mpayuas evOds 8 édaBe Ta TadrTa eis TAS yeipas Kal odrv Tots Tapotot THY TicTaV hKev éavvav eis TO pécov, Kal Neyer TddE: “Krdapye cal Ipdgeve cai of adrdot of trapdvtes “EXAnves, ov« tote 6 TH movetTe. —I, 5, 14-16.

a) Account for the use of the participles rpoovdy (line 1), wapovax (line 7)»

the mood of Aéyou (line 4); the use of 7 (line 3).

b) Account for the case of KXedpyou (line 3); of avrod (line 3); of drAvyou

(line 4); for the position of avrod (line 4).

c) What sort of man was Proxenus according to the characterization given by Xenophon in the second book ?

2. Translate into English:

oon Amn ff W ND HF

A 1@)

’Evtadéa Bevodar opa tod dpous tiv xopupyy vrép avtod Tob éavtav oTpaTevpmaTos ovoaVv, Kal ard TavTns épodor eri Tov Adhov &vOa Hoav ot mon€uLol, Kal rAéyel, “Kpatictov, @ Xeipicode, Pytv tecOa ws TdyioTa él TO aKpov: jv yap TovTo AaBwpev, ov Suvyjcovtar pévery oi bTrép Ths 0000. GAAd, e& BotrAE, weve eri TO oTpatedpati, eyo YF eOéro mopeverOar: ei S& ypntes, mopevouv emi TO dpos, éym pwev@ avrod.” “’ArArAA Siwpi co,” bn 6 Xeipicofos, “omdtepov BovrAa éréeoOa.” eirav 6 Revopav Str vewtepds eat aipeitat twropeverOat, Kedever ol cupTéupat amd TOU oTduaTos avopas. paKpov yap Hv ard THs ovpas AaBetv.—IIl, 4, 41-42.

a) Explain the use of the participle odcav (line 2). Account for the case of iv (line 3); the mood and tense of AdS@yer (line 4). What form

is ot (line 8)?

b) Which part of the army did Xenophon command on the retreat ?

128

3. Translate into English:

o Om AN Lh W ND H

‘O 8 rdrros obtos "Appevia éxadeiTo F dps lols ess trrapyos & Hv aurns TipiBalos, 6 cal Baciret piros yevdpevos, Kal ordre trapein ovdels adrros Bacitéa éri tov immov avéBaddev. obtos mpoojracey imméas éyov, Kal mpotréurypas épunvéa elev Ste BovrorTo SiarexOfvat ois dpxyover. ois otpatnyois eofev axodoar: Kat tmpocedOdvtes eis émjkoov ypwtwav ti Oddo. oO elev 8tt oTrelcacAa BovrorTo 颒 G@ pHte avTds Tos “EAAnvas aduxeivy pyre éxelvous Kxalew Tas oikias, AapBavev te TaTiTHdea Bowv SduvtTo. eéoke tadta toils otpatnyois Kat éoreicavto él tovrous.—IV, 4, 4-6.

a) Account for the mood of maped (line 2); the tense of avéBadrev (line 3); the case of avrds (line 7). 6) Give the derivation of ém7xoor (line 6).

c) What was the title given to the governor of a Persian province? What can you say of the authority vested in him ?

129

: 1914 GREEK B—ANABASIS, I-IV

Friday 4.15-6 p. m.

The translation should be exact, but clear and idiomatic English is required. Number

your answers. -

1. Translate into English:

0O cons AN HRW bd H

we xe O

Kal évrat@’ jv Ky\éapyxov xatapabeiy ws émeotate. év per TH apiotepG yeipt TO ddpu eixev, év TH Se~iG Baxtnplav: cal ei Tis aiT@ Sokoin Tay mpos Tov’TO TeTaypEvwv BraxKeveLv, Exreydpevos TOV émLTH- Sevov éraev av, kal Gua avTos mpocerawBaver eis Tov ryndov éuBaivov @oTe Taw aioxvvnv eivat jy) OV GuvoTrovddtev. Kal érdyOnoav péev Tpos avTo ot TpidKovTa ety yeyovdtes: eel Kal Kréapyov édpav omovodlovta, mpoceAduBavov Kal ot mpecBirepor, ord padXAov 6 Knréapyos éxrevdev, trrortetwy yun) del orm TWAIpELS Elva Tas Tadpous bdaTos od yap Hv pa ola rd mediov dpdeav: AN tva Sn TOAAA TrpO- gaivoito toils “EAAnoe Seva eis THY wopelav, TovTov evexa Baciréa ure@mrreve ert To mediov Td Vdwp &derkévar.—tlI, 3, 11-13.

a) Account for the case of éry (line 6); of wAyjpes (line 8). Comment on the case of the word KAeapyoyr (line 1), and illustrate by a similar idiom in English.

b) Where is yeyovéres (line 6) made? Give the first singular present indica- tive of éweordre (line 1); of ddetxévas (line 11).

c) Account for the mood of Soxofy (line 3); of zpodpaivorro (lines 9, 10).

2. Translate into English:

I

cons’ AM PW bd

Tovdto Aéyovtos avTod, wrapvutai Tis: axovoavtes & oi oTpaTL@TaL TavTES ua Sppt Mpocexvynoav Tov Oedv, kal 6 Hevopear eime: “Aoxel fol, ® avdpes, érrel mrepl cwrnplas juav ey6vTOV oiwvds ToD Aros TOU carhpos ébdvn, edEac0a TH Oe TOUTH OUcewW corhpia, brov dv TpaTov eis diriav yopav &pikdpeBa, cvvetrevEacGar kai Tois arrows Oeois Gicew kata Stvapiw. Kal btw Soxei tadta,” edn, “dvatevdtw TV xeipa.” Kal avérevav Gmravtes. €x TovTov 8 nv—avTo Kal érradvcar. érrel TA TOV Oedv Kaas clyev, NpXETO Tad GSe.—III, 2, g.

a) Account for the use of the participle Aeyévrwv (line 3); the mood of

adudpeba (line 5); the case of dpuy (line 2).

b) What is Xenophon’s general attitude toward the gods and signs from heaven? Cite another instance from the Anabasis. | c) What is the force of the prepositions in cvverevfac6au (line 5) ?

130

3. Translate into English:

I

00 ON AN FW DN

Lal

’Ezred © 4 Bor wrel@v Te éyiyveTo Kal éyyttepov, Kal of del érudyTes cov Spop@ érl Tors Bowvtas, Kal TOAAD pelCwv éyiyveto 7 Bon dom dH mrelous éyiyvovto, éddxer 5% peifdv Te civar TO Revodartt, cal avaBas éd’ timmov kal Avkiov kal Tors imméas avadkaBov trapeBonba. Kal raya 5% axovovor BoovTav Tov oTpatiwtGv “@dratta,” “Oaratra,” Kab Tapeyyvovtav. év0a by cov mdvres Kal ot omicOodiraxes, kal Ta e 7 > 7 \ oe > \ % 2 / / 3 b feay 4 brroliyia HAatveTo Kal ot trot. Enel 8 adixovto travtes érl Td dxpor, évtav0a. 5 mepréBadrov addndovS Kal Tos oTpaTHYoUs Kal oyayors Saxpvovtes. Kal éEamivys, Grou o) mapeyyuicavtos, of otpaTiata

/ /, \ nA bs / pépovow AlGovs Kal TroLovaLY KOAwWVOV peyayv.—lV, 7, 23-25.

a) Account for the case of 7oAA@ (line 2); of otparwréy (line 5); explain the frequent use of the imperfect tense in the passage.

b) What was the reason for the great joy of the Greeks upon this occasion ?

131

I9I5 GREEK B—ANABASIS, I-IV

Friday 4.15-6.15 p. m.

The translation should be exact, but clear and idiomatic English is required. Number

your answers.

1. Translate into English:

o ons Amn ff WH HD

‘O 6& Kipos brodaBav rods detryovras avAdé~as orparevua éro\LdpKe Midnrov kal xara yqv kal Kara Oddarray Kal éreipGro Karayeu Tovs éxreT~- TwKoras. Kal arn ad &AAn Tpddacts Hv alr@ Tod dOpoitew orpdrevpa. mpos 6€ Baoitéa tréurwv jélov ddeXpds Sv adbrod dobfvar of rabras ras modes UaAov } Ticcadépyynv apxew airav xal h untrnp cuvérparrev aitS Taira’ wore Baoidels THY pev pds éavrdov éxiBovdrAHy ovik Habavero, Ticca- dépver évoute ToXeuodvTa airov dudl Ta oTparebuara baTmavav’ adore ovdév HxOETO aira&v TodeuolvTwr. Kal yap 6 Kipos arémreure tovs yuyvo- pévous dacpovs Bacidet é€x Tv 7oAewv.—lI, 1, 7-8.

a) Comment on the use of the imperfect tense in this passage.

b) Account for the case of air@ (line 3), a’rGp (line 5), Tusoadépver (line 6), ovdéy (line 8).

c) Account for the mood of daravay (line 7), nobavero (line 6).

d) Where is éxerrwxéras (line 2) made? Give the principal parts of the verb; also of d@poifey (line 3).

e) What was the motive of Parysatis in helping Cyrus? What may be inferred from this passage about the conditions of government of the Persian Empire ?

2. Translate into English:

I

onsr Am fF W ND

Nov yap tows kal tyuets aicbavecbe ws &Oiuws pev HAOov emi Ta Sra. GObuws 5& rpds Tas hudraKkas’ Gore, oltrw y’ éxovTwv, ovK olda STi ay Tis xpnoato avrots elite vuxtos déot ere kal juépas. jv b€é Tis aitav tpévy TAS YVOPAS, WS M1) TOUTO pOvoy évvowvTar Ti TEitgovTat GAG Kal TL ToLN- covet, TOAD evOupdrEpor EgovTa. émicracbe yap bn Stu ore TAHOds EoTiv ore icxds ) &v TH ToAEuwW TAS Vikas ToLovoa, aA’ SrdTEpoL ay aby Tots Geots Tats Puxats éppwyevéorepor twow él Tos Todeutous, ToUTOUS ws emi TO mond ot dvriou ov déxovrar.—III, 1, 40-42.

a) Explain the construction of éxévrwy (line 2). Account for the case of

avrots (line 3), vuxrds (line 3), uxais (line 7).

b) Account for the mood of 6éou (line 3), twouy (line 7).

c) What calamities have befallen the Greeks that they are now so dis-

couraged ? 132

~

3. Translate into English:

TAM ff W ND HF

“Oi ev rod€uior, ws Spare, Karéxovar Tas brepBoras Tod Spous’ wpa BovreveoOar Srws ws KaANOTA aywriobueba. Euol wey ody boxe? Tapay- yethar pev apiororoetobar Tots orparwwrais, Huds BovdrevecOar ere Thuepov eire ailpiov doxel brepBadrev 76 pos.” “’Eyol ye,” edn 6 Kyedvwp, “doxe?, €rav raxioTa apiornowper, EEoTmALCApévous Ws TAaXLOTA iévar él Tovs &vipas. el yap diarpiWouev THY ThuEpoY Huépay, ot VOY Huds Op@vres toéutoe Oapparewrepor Eocovra.”’—IV, 6, 7-0.

a) Account for the mood and tense of dywvotpeba (line 2); dtarpivouev

(line 6); the mood of édpiorhowper (line 5); the tense and case of é£o7oapevous (line 5).

b) What is the force of the rapa in rapayyetAau (line 2) ?

c) Indicate, preferably by drawing a rough map, the route of the Greeks from the gathering of the expedition to its arrival at the Black Sea, and locate several of the more important districts, cities, and rivers on the route.

133

‘a

be UNe Fah Sere SRE

4 —t &

<5 < +; 2 ay ye eee | ern

bie sya > > j 4 * ¢ .. - 4 * a8 ie . zu “te i oar a ee weir 3, : Ray . ra * a es See bea

~ zorr GREEK C—HOMER’S ILIAD, F-III Friday 3-45-5.45 p. m.

The translation should be exact, but clear and idiomatic English is required. Number your answers.

1. Translate into English: @éris & ov AfPer’ Edherucwv 495 ma.dos 0%, GAN’ H y’ avedicero Kdua Pardoons, nepin & avéBn péyav otpavdv OvtAvproyr Te. etpev 8 evpvora Kpovidny arep juevov GAAwv axpotdatn Kopypy moAvderpddos Odtriproro. kai pa mapoil’ abroio KabeLero, kal Ad Be yotvav 500 oxain, Sefvrepy & ap’ iz’ dvOepedvos EXodoa Avocomevyn mpocéere Ata, Kpoviwva dvaxra: Zed warep, et wore bn oe per’ APavdroow dvyca 7 ere. 1) Epyw, TOE por Kpynvov €ASwp.—I, 495-504.

a) Give the Attic equivalents for 496er’ (line 495); €0t (line 496); OvAvpzrov0 (line 499); yovvwr (line 500); Segvrepy (line sor).

b) Explain the formation of dvedvcero (line 496), naming the tense and giving the tense-suffix. What is the first person singular present indica- tive active of dvyca (line 503) ? ;

c) Account for the.case of otpavdv (line 497) and give the prose construction for this expression. What is the prose equivalent for per’ dOavdroow (line 503) ?

2. Translate into English: "H piv Kal rovos éoriv avinbevra veer Oar. Kat yap tis @ &va pyva pévwv ard Hs aAdxo10 doxardg odv vyt roduliyw, dvrep deAXat

, > / > / ,

xerpmepiat eiA€wowy dpivopevyn Te Oddacoa" npiv 8 eivards éote mepitpoTréwv eviavTos 295 evOade pipvdvrerou TH ov veweciLop’ "Axauods doyaAdav rapa vyvot Kopwviow: GAAG Kal éur

x pa vy p v a pays aicxpov Tor Sypdv Te pevev, Kevedv Te veerOat. TART, Piro, Kai peivar’ eri ypovov, dppa Sadpev A > \ , , 2X ed n é€reov Kadyas pavteverar, n€ Kal ovxi.—II, 291-300.

a) Give the Attic equivalent for piyvovreoor (line 296) and explain the Homeric suffix.

b) What part of speech is Sypdv (line 298)? What form is da@mev (line 299) and from what verb?

c) Who speaks these lines? Show that they are appropriate to the character of the speaker.

136

. Translate into English:

"AAN Gre 5) woddpytis avaikeev ‘Odvacers,

otdoxev, tral de derKe KaTa XOovds Gppata mas.

oxyrtpov & ovr’ éricw ovre pompyves Evwpa,

GAN doreudes Exeokev, aldpei uti éouxws:

gains Ke takorév Tw’ Eupevor Gdpovd 7 aitws. 220 GAN’ bre by) Gra Te peyaAnv éx or7nGeos ein

Kai rea vupaderowy €orKdTa XELpEpinow,

ovk ay éreit’ “Odvoni y’ épiccwere Bpotds aAXos-

ov Tore y’ BS’ ‘Odvotos a&yacoduch? cidos iddvres.—III, 216-224.

a) Scan lines 221 and 223, marking quantities, division into feet, ictus, and chief metrical pauses.

b) Explain the apparent hiatus in line 221 and the quantity of re in this line.

c) What are the component parts of roAvuyris (line 216) and {dkorov (line 220)?

137

Igr2 GREEK C—HOMER’S ILIAD, F-IIl Friday 3-45-5-45 p.m. The translation should be exact, but clear and idiomatic English is required. Number your answers.

1. Translate into English: ot & ézet odv Hyepbev Sunyepées 7” éyevovro, toto. 8 dvorduevos perépy mddas dks "A xAdeds" ““Arpeidn, viv Gppe radu rAayxGevtas diw ay drovorrycev, et nev Odvardv ye pbyoupev, 60 ei 51) 60d woreuds Te SapG Kal Aowwds “Ayxatovs. GAN’ aye 84 Twa pavriv épelopev, 7) tepfa, } kai évetporéAov—xai ydp 7’ Svap éx Aids éorw— 6s K’ €izror, 6 TL TéoTOV exdoaTo BoiBos “ArddAwy, ei 7’ dp 6 vy’ ebxwdfis emipenerat, et & ExatouBys* 65 al Kev Tws apvaev kvlens aiyOv Te TeAciwv Botrerar dvridcas yuiv dad Aovyov dpiva.”—I, 57-67.

a) Give the Attic equivalents for jyepOev (line 57); dupe (line 59); Sap@ (line 61); tepja (line 62); tdéccov (line 64).

b) What mood is épefouev (line 62) and ovderas (line 67)? Explain the construction of ei xev diyoer (line 60).

c) Explain the case of edywAjs (line 65); of xvicns (line 66). -What is the present indicative of dvridcas (line 67) ?

2. Translate into English; | 6 8 dp’ &ero rapBycev re dAynoas 8’, axpeiov idwv, dmopopéaro Saxpv. of 88 Kai axvipevot wep ex’ aita 4Od yeAaocay’ 270 de O€ tus elrerkev idwv és wAyoLov GdAov" “°) admou, 4 8) ppt’ “Odvaceds Ec Ord Eopyev BovAds 7’ eEdpxwv ayabas moAeudy Te Koptrrav’ viv d& Tdde py’ dpirrov év “Apyeiovow epegev, ds Tov AwByTipa érerBddrov eoy’ dyopdwv. 275 ov Onv puv mdduy adris avira Ovuds aynvwp _vekeiew Baoirjas dvedelous éréeoow.”—II, 268-277. a) Give the first person singular present indicative of éopyev (line 272); epegev line 274); dvyoe (line 276). b) What is the derivation of xopiccwv (line 273)? The composition and the meaning of the component parts of éreaBoAov (line 275) ? c) Explain the formation and the meaning of «iweoxev (line 271) and the use of tis (line 271). d) Who is the subject of ero (line 268) ?

138

3. Translate into English:

ce 2% , , > , 7] TY pe TPOTEpw TroALwv ed varomevawv 400 4 x , xX , > a ages Ppvyins 7 Myovins épareivys, Lee + a 4 > , el Tis To Kal Ket hiros pepdrwv avOpadruv ; ovvexa dy viv Siov “AX<dEavdpov MevéAaos vikynoas €berer orvyepyy pe oixad ayerOar, | towvexa 5% viv Setpo Sorodppovéovera, rapEerrys ; 405 00 map’ avtov iodea, Oedv & drdeuxe KedevOov" pnd’ ere coior wédecow brootpapeas "OAvpror, GAN’ ail wepi Keivov difve, Kai PiAacce,

eis 5 0” H GAoxXov TanceTaL, } 5 ye SovAnv.”—ITII, 400-400. a) Scan lines 400 and 404, marking-quantities, division into feet, ictus, and chief metrical pauses.

b) Account for the metrical quantity of pe (line 400); the retention of the final vowel in éyé (line 404).

c) State briefly the circumstances under which these words were uttered, naming the actors.

139

GREEK C—HOMER’S ILIAD, Ell Oe

Saturday Q-II a. m.

The translation should be exact, but clear and idiomatic English is required. Number your answers.

1. Translate into English: “Myre od Tév8’, ayabds trep éwv, atroaipeo Kovpnr, 275 adr’ &a, & of mpata Sdcav yépas vies *Ayadv: pnte od, Tinreldn, EX’ épiSdwevar Bacirije avriBinv: érel ovo” omoins Eupope Tihs oKyTTodKxos Bacirets, 6 Te Zeds Kddos edwxev. ei O€ od Kdptepds éoor, Ged ce yelvato unTnp, 280 arn’ Be héprepds oti, eel mrEdverow AvacceL. ’"Atpeldn, od mrave Tedv pévos: avtap éywrye Alcoow’ "AyirArHe weOduev yoror, bs wéya Tacw &pxos “Ayatoiow médeTat TohELOLO Kaxoio.”’—I, 275-284.

a) Account for the case of rdvé (line 275); timis (line 278); mroAdyovo (line 284). What other case might have been used with avdoce: (line 281)?

b) Give the derivation of oxnmrobyos (line 279).

c) Who speaks these words and what does he say of himself in the part of his speech preceding this selection ?

2. Translate into English: | "Evéa xev ’Apyelorow brréppopa veotos éTvy On, 155 ef uy "AOnvainv “Hpn pos pdOov ertrev> “*C) ardzrot, airytdyoto Aids téxos, atpuTovn, oT Si oixdvde PiAnv és tratpida yaiav ’"Apyeios hevEovtat er’ evpéa vata Oaracons ; Kab ev evywrny Tpidu cal Tpwor dAérrovev 160 ’"Apyelnv ‘EXévnpv, fs etveca troddol ’"Ayarav év Tpotn améXovto, hirns aro tratpioos ays. arn’ tOt viv Kata ANaov ’Ayardv yarKoyitTovOr, cols ayavois éréecow épntve SOTA ExacTor, pndé a vijas Grad EXkévev Audieriooas.”—II, 155-165. a) Give the Attic prose equivalents for devzrev (line 156); «dé (line 160); ains (line 162); vijas (line 165); édxepev (line 16 5). b) Explain the metrical quantity of 6% (line 158); of -as in vias (line 165); the formation of «aé (line 160). c) What is meant by tmesis?

140

3. Translate into English:

Acvtepov adr’ ’Odvcja idmv épdew’ 6 yepaios:

“elt? dye wot Kal Tovde, hirov Tékos, bats 58’ éoriv:

petov ev Kcepary ’Ayauéeuvovos ’Atpe(dao, > tA > © > , 2

evputepos 8 w@poow idé orépvoow idéoBar.

Tevxea ev of Keitar ert yOovt rovAvfoteilpy, 195 | P>) / f e > a f > 5 a

QUTOS O€, KTINOS WS, ETIT@AELTAL OTiYaS AvOpwr:

apve@ piv eywye eioxw Tyyer“arro,

és 7” diwv péya trav Siépyerar apyevvawv.”—III, 191-198,

a) Scan lines 195 and 108, marking quantities, division into feet, ictus, and chief metrical pauses. What is a line like 198 called?

b) Mention the things (giving Greek names) which make up the tevyea (line 195).

c) Explain the accent of e/7’ (line 192). Is the middle form sdéc0a (line 194) used in Attic prose?

141

-

GREEK C—HOMER’S ILIAD, I-III

Saturday Q-II a. mM.

1914

_ The translation should be exact, but clear and idiomatic English is required. Number your answers.

1. Translate into English: . ““Xalpete, enpuxes, Avos dyyedou 7dé cal avdpar. dooov i7’+ ov Ti por Tppes eralrio, AA ’Ayapeuvor, 3an d opax pole Bpionidos etvexa Kovpns. 3 aX’ aye, Sioyevées ILarpéxras, éEaye kovpnv kai char dos Gyew. THO adTo pwaptupo éotov Tpos Te Oedv waxdpwv pds Te OvnTtav avOpoTrav kal mpos Tov Baoihfjos amnvéos, ef mote 6) avdTe 340 xper@ enelo yevyntar devxda Aouyov apdvar Tos adAos. 7 yap 6 y SdoLToL dpeot Over, ovdd Tt olde vojoa dua Tpdccw Kal dricca, OrTas of Tapa vyVvol cdo paxéovTat "Ayarol.”—T, 334-344.

a) Give the Attic prose equivalents for types (line 335); Pacrdmos (line 340); éueto (line 341); dAoujor (line 342); vyvot (line 344).

b) Explain the syntax of pot (line 335); of dyev (line 338); of yéevyra (line 341); of paxéovras (line 344).

c) What part has Achilles played in preceding events that Agamemnon now takes away his prize? Who was the prophet concerned ?

2. Translate into English: ‘CH pav adr’ ayopy wuKds, yépov, vias ’Axyaav. 370 - at yap, Zed re matep cal ’AOnvain cal” ArrodXop, TovovTou Séxa por ovupppadpoves elev “A yaar: TO Ke TAY’ Hpvoere words I pudporo dvaxTos, © Xepolv ip Hpetépyow adotod re wepGopevy Te. adrrAd por aiyloyos Kpovidns Zeds ddye’ axev, 375 ds pe per’ GtrpyKTovs epidas Kal veixea Barret. Kal yap éyov "Ayireds Te waynoduel” eivera Koupns avTiBios éréeciv, eyo & Apyov Yahetraivav- el O€ trot’ & ye piav Bovrevcoper, ovKéer’ errerta Tpwcly avaBryors Kaxod éocerar, 008’ 7Batdv.”—II, 370-380. a) State the composition and derivation of cvpyppddpores (line 372); dxpyxrovs (line 376); davdéBAnors (line 380). b) Explain the mood and tense of jpvcee (line 373); the use of the participle xareraivwv (line 378). c) Write the Attic prose construction for xepolv id’ jyerépyow (line 374). Give first singular present indicative of dAotea (line 374).

142

3: Translate into Engi: Tov & as ovv évdncev ’AréEavdpos Oeoerd7)s “a 30 év mpoudyouor pavevta, KateTAHYyn hidov HrTop, aap & érdpwv eis vos éyaleto Kip’ adecivor. as & ote tis te Spdxovta idwv tarivopoos amréoTn ovpeos ev Byoons, vid Te TPdmos EXAaBe yvia, day & dveywpnoer, dxypds wy fre Traperds, 35 @s avtis Kal’ duirov edu Tpowy ayepdyov deicas ’Atpéos viov ’AX€Eavdpos Oeoerdys. tov © “Extwp veixercer idwv aioxpois émréeoi: “Avotapt, eidos dpioTe, yuvatpmaves, nTrEpotrevTa, ai®? Shehes Gyovds 7’ Epevat dyapuds 7’ arroddoGat.”—TIT, 30-40. a) Explain the tense used in the main verbs of the simile. b) Account for the case of yrop (line 31); of wapeds (line 35); of dyovos (line 40). : c) Write line 40 in Attic prose, expressing the meaning without the use of Oderes. d) Scan lines 34 and 40, marking quantities, division into feet, ictus and chief metrical pauses. Account for the quantity of re (line 34); the quantity of the final syllable of €uevax (line 40).

143

GREEK C—HOMER’S ILIAD, I-III Saturday

1915

Q-II a. m.

The translation should be exact, but clear and idiomatic English is required.

Number your answers.

1. Translate into English: “Tov viv wiv wyjoaca trapéleo Kal AaBE yowvwr, ai xév Tas €Oédnow ért Tpwecow apnea, Tous O€ KaTa Tptpvas Te Kal aud’ Gra doa ’Ayauods KTELVOMEVOUS, tva Travres érravpwvTar Bacid#os, yv@ Kal ’Atpeidns edtpd xpetov ’Ayapyéuvov a + 7 - aa > n Oe nv atny, 67 adpicrov Ayatov ovdéev éticer. Tov © npelBer’ Erecta Oris Kata Saxpuv yéovca: éé + / > / 4 / > had a @ mol, TéKVoV eudv, TL Vd ao” etpedhov aiva Texovoa; aif’ dheres Tapa vnvoly adaxpuTos Kab amrnpwov

410

415

noOa, éret vb To aioa wivevOd trep, od Tt wdra Syv.”—I, 407-416.

a) Give the Attic prose equivalents for rapéfeo (line 407); -yobvwr (line 407); édnow (line 408); Tpweoo (line 408); Bacidfjos (line 410); vyvolp

(line 415).

b) Account for the use of the subjunctive in line 408; the mood of ddedes

(line 415); the case of BacvAjjos (line 410).

c) What cause does Homer assign for the expedition against Troy? What

- is the reason for Achilles’ wrath ?

2. Translate into English: Tap &, ds 7 dpviOwv mrerenvav eOvea trodXdd, a 4 f

XNVaV i) yepdvev 7 Kixvov SovrALyodelpwr,

"Aci év Netwou, Kavotpiov audi péeOpa,

4 a > , 7

évOa kal év0a TroTavTat ayaddoueva TTEpUyEecouy, \ / a / A

Krayyndov mpoxabifdvtwv, cwaparyel Od Te AEcpar,

as Tov EOvea TrOAAA Vedv Aro Kal KALOLAwY

3 f / Ul > \ e \ \

_ és mediov mpoydovto ZKapavdpiov avtap vd yOov opepdardov KovaBile mrodav avtav te Kal trmev. éotav & év ANexpou VKapavodpio avOeuderts

460

405

Kupto, dooa Te PUAN Kai dvOea yiryverat Gpyn.—Il, 459-468.

a) Narrate briefly the events, related in Book II, which lead up to this marshal-

ing for battle.

b) Scan lines 461, 462, 463, marking quantities, division into feet, ictus, and chief metrical pauses. Explain the quantity of the last syllable of ’Aciw - (line 461). Comment on the effect of the dactylic and spondaic feet in

lines 462, 463.

c) Give the derivation of rerenvar (line 459); of kAvovdwy (line 464).

144

3. Translate into English: *Ipis & at@?’ ‘Enrévyn NevKwrev@ ayyerdos 7rOer, eldouevn yardw, "Avtnvopidao dapaprt, tiv “Avrnvopidns eiye xpeiov “EXixdor, Aaodixny, Ipuduoto Ovyatpav eidos apiortnv. Tiny & ebp’ év peyapw: 7 S€ wéyav iotov bpawvev, 125 Sirraxa trophupény, tor€as 8 évérraccev adOdous Tpowv & immoddpov cal ’Ayadv yadKoxiTo@ver, ods &Oev ever’ Eracyov vr’ “Apnos Tadapawr. ayxod & ictapéevn tpocépn Tddas axéa-"Ipis- “Sedp’ 101, vida dirn, iva Oéonera Epya idnat 130 Tpdwv & immodduov kat Ayadv yadKoyiTover.” III, 121-131.

a) Give, in English and in Greek, familiar epithets of Achilles, Agamemnon, and Odysseus. |

b) What is the real purpose, in the poem, of calling Helen to the wall? c) Write line 128 in Attic prose.

_d) Give the Attic prose equivalents for Tprdpovo (line 124); modéas (line 126).

145

5 La wy 2 Legs tae es

PROSE COMPOSITION

IQII GREEK F—PROSE COMPOSITION Saturday 3-45-5.15 p. Mm.

Translate into Greek:

When the heralds from the king had said what they were ordered to say, the generals were very angry, and Clearchus, after telling the rest to give what- ever reply seemed best to them, went away to do something which he had begun before these men came. So the other generals told the heralds that they would not give up their arms, for if they did that, they were afraid that they would be deprived even of their liberty; and besides,’ they would be worth? much more to the king if they had their arms than if they gave them up. But no one would declare’ whether they intended to remain where they were or go away.

tapos Tovrots. 2 cos. 3 dnddw.

148

IQI2

GREEK F—PROSE COMPOSITION Saturday 3.45-5 .I5 p.m.

Translate into Greek:

It seemed best to me to come to a conference with you that I might, if possible, remove your distrust of us before war should arise. May the gods prevent us from being enemies of each other! If we should slay you, who are our friend and benefactor, we should deprive ourselves of the hope to return in safety to our native land. We know that you are able to help us and we believe that we can be of service? to you by conquering for you nations which are now hostile to you. Who, therefore, would be so senseless? as not to wish us to be friendly to each other ?

t dpedelv= to be of service. 2 walvesSat=to be senseless.

149

| 3 1913 GREEK F—PROSE COMPOSITION

Saturday 4.15-6 p. m.

Translate into Greek: .

“Why do we remain so long a time in this place? We know that if it were possible the king would attack us now, but he cannot for his army is scattered." Let us not wait until it is collected again. The king will never willingly permit us to report to the Greeks that though so few in number we defeated him.” To those who said this Clearchus replied: ‘If we try to march away it will be neces- sary to fight at once. But we have no cavalry, so that even if victorious, we could not accomplish anything; if defeated, no one of us would be saved.”

Sracre(pw= scatter.

150

| 1914 GREEK F—PROSE COMPOSITION Saturday 4.1 5-6 p. m.

Translate into Greek:

While the Greeks were encamped here, Tissaphernes came to them and said: “T was the first to report to the king that Cyrus was marching against him, and we plotted together to kill Cyrus. Although I did this, I am friendly to you and I asked him to allow me to lead you safely back to Greece. He bade me ask you why you marched against him. I advise you to answer moderately,’ so that I may be able to save you.”’ Clearchus replied that they would not have attacked the king had they not been persuaded by Cyrus, and that they wished to return home without injuring anyone.

t ueT plus,

ISI

IQI5 GREEK F—PROSE COMPOSITION

Friday Il.15 a.mM.-I p.m.

Translate into Greek:

The Lacedaemonians had in mind to march against the Persians, since they believed most of the countries through which they must pass were friendly. At this time, however, those at home perceived that the largest cities in Greece were preparing to attack them, and they sent a messenger to their king bidding him to come to their aid with all speed. He was troubled* when he considered? the honors and hopes of which he was now deprived. But he called the allies together and told them that Sparta had summoned him. He promised to return to accomplish what they now desired, as soon as conditions‘ should be satis- factory in Greece. On hearing him, many wept and all voted’ to go with him to save Sparta.

txarerGs pépw. 2évOuudowat. 3dmoorepéw. ‘things. 5 yydlgoua.

152

TION OF PROSE

2

IQII GREEK G—SIGHT TRANSLATION OF ATTIC PROSE Saturday I.30-3.30 p. m.

Read the passage through several times before beginning to write. The trans- lation should be exact, but clear and idiomatic English is required.

Translate into English:

XENOPHON IS PROPOSED FOR THE GENERALSHIP, BUT SHOWS Wuy His ELEc- TION IS INADVISABLE

fol \ /, A nm A ‘H 8 orparia cvvndGe, Kat mdvtes Eeyov Eva aipeicOar Kal érel TovTo édoke, mpov- ~ > 9 BdddXovro! airdv. émei eddxer SpAov civar Ste aipyoovra airdv, «i tis erulypilor,? dvéorn Kat éreke rade, Ss X a , »” ‘Kyo, & avdpes, ndomar pev bh’ tuov Tyrwpevos, eirep avOpwrds eipt, Kal yapw »” A 5 lal , \ g \ » 4 c wa > a cal / 6 A , } sae. éxw kal ebxouat Sodvat por Tods Deods aitiov Tivos byiv ayaBod yeverOai: Td pevTor eye 6n c 7. “a + A da / > 5 -. »” c a 5 - F mpoxpiOnvar vp’ tpwaov dpxovta Aaxedatpoviov avdpds mapdvtos ovTe tiv pou Soxel oup- x la » / cal épov civat, GAN’ HrTov Gy dua TodTo Tvyxavew, et TL SéoieHe wrap’ aitdv: euol TE ad ov , / > x : “a c “~ ‘\ 9 \ a / 3 /, > 3 ravu Te vopilw dopadés eivat TOUTO, dp@ yap OTe Kal TH waTpids pov ov mpdabev éxav- lal ‘\ 4 a / Cal al GavTo ToAEuovVTEs Tplv eroinoay Tacav THY TOA Spodoyev Aakedapovious Kal aditdv .3 4 > > \ de ~ ¢ Ao en} > 4, “a \ r Byes Hyewovas elvat. éei S& Toto Hmodrdynoav, edOis ExatcavTo wodemovvTes Kal ovKETL , 2 4 ‘\ aN > > a ee ee! 8 , LA 8 , > a 6 mépas éroAdpkynsav THY TOA. «i OdV TadTa dpGv éyw SoKoinv drov Suvaiunv évradOa a , , a a , N a dxvpov4 zrouiv Td éxeivov akiwpa,S éxelvo évvo® py Alay dv Taxd cwdpoviabeinv.® 06 8 a a h x * A al > » bpels évvocire Ste NTTOv Gv aracis? Ely EvOS GpxXovTOs 7 TOAAGY, ev LoTE OTL GAAOV pev EX 4 > Teed 2 8 , ra a 14 64 > X al EXdpevor ovK evpHoeTe Ene oTacidlovTa’ vouilw yap doTis év Tokeuw dv oracidler A » a“ \ e a / 4, + ae de : eX. 6 3 Xx mpos GpxovTa, ToUTov mpds THY EavTOD TwTypiav oTacidlew’ eav SE Eue EAnoHe, ovdK ay Be + 9 a Se \ 3 aw 66 Oavpdoatpe et Twa evporre Kal tiv Kal éwot axOdpevor. ~ / / , / "Exel S& tadra ele, woAd mAcloves avicravTo A€yovTes ws Séou adTov dpyxew.—Xeno-

phon, Anabasis, VI, 1, 25-30.

rpoBdddowat =propose for election; = 7émivndlf{w~=put to vote; 3ovKére mépa = no further; 4 &xvpos = without authority; 5 dtiwua =dignity, position; 6 cwohpovitw = bring one to his senses; 7 ordo.s = discord. :

154

IQI2 GREEK G—SIGHT TRANSLATION OF ATTIC PROSE

Saturday | I.30-3.30 p. m. Read the passage through several times before beginning to write. The translation should be exact, but clear and idiomatic English is required.

Translate into English:

PHARNABAZUS AND CONON EXPEL THE SPARTAN HARMOSTS FROM THE CITIES ON THE COAST, BUT DERCYLIDAS KEEPS ABYDUS

mpatov pev Tovey PapvaBaos kai Kovwv, éret évixnoav tovs Aaxedaipoviovs tH , , \ \ , \ \ \ > , , , vavpaxia, mepimdéovres Kal Tas viTous Kal mpds Tas émOadrarrTidias TédeLs Tovs TE Aaxwvixods dppooras é€nAavvov Kal mapeuvOotvro® tas modes ws otTE dKporddets $ , 22% , > , = 3 * » > , on 5 , er ee . évretxioouev? edoouev Te adrovopous.3 ot 8 dxovovres Taira HdovTd Te Kal éxyvovv Kal Etna. rpoOipws exeurov TH DapvaBdly. Kat yap 6 Kovwv riv PapvdBolov edidackev ds ovTw pev rowdvTe Tao adT~ at wodes hiro EvowTo, ci SovrAotcbar BovdrAdpevos \ 4 e , | ae 4 , e , A , avepos Ecorro, deyev ds pia Exdory TOAAG mpdypara ixavy ein wapéxewv Kai Kivdvvos cin ph Kat ot "EdXnves, ci tatta aicGowro, ovoraiev.4 tadra pev ov éreifeto 6 DapvdBalos. droBas & «is "Edecov 7G pév Kovon Sods terrapdxovra rpijpes eis A \ a , > a Synoriv ciwev aravray, aitos S& rely mapye eri rHv abrod dpxyv. Kal yap 6 , 3 A > > Aepxvridas, domep Kal mada rodeptos Hv aito, Ervxev év “ABidw wy, Ste Y vavpaxia éyévero, kal ovx domep of GAAOL dpposrai eSeAimev, GAAG KaTéecxe THY "ABvodov Kal _ , \ A , 4 > Suécwle Pidrnv rots Aaxedaupovios. Kal yap cvyKahécas Tovs “ABudnvods edeke Toidde * ro nw , a a A “0, dvOpes, vov &eortw tpiv kat rpdcbev pirors odor TH oA Hudv edepyéras havjvar tov Aaxedatpoviov, Kat yap 7d piv éy sais eimposiasS motois aiverOae ovdéev / an davpaorov’ drav twes év cvpopats® yevouevwv dirwv BéBaior7? havaor, Todr’ eis Tov dmavTa xpovoy pynumoveverar. ore OF ovx oOvTws Exov Hs, ei TH vavpayxia expatnOnuev, ovdev dpa ere éopev’ GAA Kal TO tpdcbev Syrov, ’AOyvalwy apxdvtwv THs Oararrys, ixavy hv y Hperepa ToAs Kai ed pidrovs Kal KaK@s €xOpovs woreiv.”’ —Xenophon, Hellenica, IV, 8, 1-4. t rapeuv0odvro=consoled by saying; 2 évraxliw=fortify; 3 abrévouos = independent;

4ovoTfivar=get together, combine; ‘Sebrpatia=prosperity; 6 cuudpopd = misfortune, ad- versity; 7féB8acos=steadfast.

155

1913 GREEK G—SIGHT TRANSLATION OF ATTIC PROSE

Saturday 2-4 p. mM.

Read the passage through several times before beginning to write. The translation should be exact, but clear and idiomatic English is required.

Translate into English:

SEUTHES, THE THRACIAN KING, LEADS THE GREEKS TO PRO- VISIONS AND OFFERS THEM INDUCEMENTS TO SERVE UNDER HIM

"Exel 5€ LevOns mpoohrOev, cie Hevodar, “Hyeis tropevdueba Srrov péerre ev TO oTpdtTevpa Tpopyv: Av odv Huiv Hynon Srov mreioTa éoTW emiTnoera, Ud cov vomodpev Eeviler Oa.” Kal 6 LevOns én, “’AAAA oidat Kopas ToANaS aOpdas Kal Tavta éyovcas Ta émiTHdea ATEYovcaS Huav doov? SuerOdvres Av Hdéws apiote@nte.” “‘Hyod toivur,” &bn 6 Bevodar. érel & adixovto eis avtas ths SeiAns, cuvArAOov of otpaTi@Tat, Kad eEize TevOns tordde. “’Eyo, @ avdpes, Sdowar tuav otpateverOar crv éepol, Kar imicyvodpat tpiv doce Tots oTpatiatas Kuliknvdr,s Noxyaryois Kal oTpa- Tnyois TA voptduevas4 Kal mpdos TovTas Tov aELoV Tiunow. outia Kal Tota aoTrep Kad vov éx THS yopas Nau Bavortes ete. omdca 8 Av adiocknta akidocw adres exe, iva tadra SvatiOduevoss div Tov piobdv tropive. Kal Ta pev hevryovta Kal arrodidpdcKovta Hels ixavol éodpeba Si@Kev Kal pacrevery 6 av Tis avOtoTHTaL, crv bpiv TepaccpeOa YeipodoOat.? peta TadTa édidoTO Aéyeww TH Bovropevm. Kal édeyov TodXOl KaTa TavTa, btt TavTOs dkia Aéyor Levys yewov yap ein kal ote oixade arroTheiv TH ToOVTO BovrAopev@ Suva- tov ein, SvayevérOar Te év diria® ovy oidv te, eb Séor wvovpéevous Chv, év be tn Torenia® SiatpiBew Kal tpépecOa acharéotepov peta LevGou 7 pdvovs- évrev & ayabav TocovTwr ef pic Odv TmpocrApowTo, evpnua? eddxer eivat. ert TovTols elev 6 Revopar, “Ei tis avtireyet, reyéerw: ef wy, errupndifo” éya tara.” érel ovdels avtéreyer, ereyrydice, kal édoke tadta. evOds YevOy ele tadta, bt cvotparevoowTo avr@.

—Xenophon, Anabasis, VII, 3, 3-14. tievifw=entertain; 28cov=so far that; %xuginvds=a Cyzicene stater, a gold coin;

‘7a vou.foueva=the customary presents; 5 Scar (Geuar=sell; 6 waoredw=search for; Txepbouat=subdue; supply: x@p¢g; Y%elpnua=agodsend; ”érunplg{w=put to vote.

156

1914 GREEK G—SIGHT TRANSLATION OF ATTIC PROSE

Saturday ‘2-4 p. m.

Read the passage through several times before beginning to write. The translation should be exact, but clear and idiomatic English is required.

Translate into English:

[Xenophon calms the soldiers at Byzantium]

‘O 8 Bevodadr as cide ta yiryvdpeva, Selcas pn ed’ aprraynv tpatroito 76

otpdtevpa Kal avnKxeota! Kaka yévorTo TH TédEL Kal EavT@ Kal Tois oTPATLO- Ba / v lal al \ aw \ ¢

ras, Ge Kal cvveotrinte elow THY TUAOY ody TOE OxAw* Of BUavTLOL ws elSov TO otpdtevya Bia eiorrimtov, hevyovow ék THs ayopas, ol pév els TA mroia, of olxade, door Evdov étiyyavor dvtes, &Ew, of KabetdKov? Tas

Js ce >] al , 4 , > / e e Tpinpes, ws év Tais Tpinpeot o@loWWTO, TaYTES d€ @ovTo aTroA@XEévat, ws Earo- kvias THS TdAEws. 0 ’EteduiKos eis THY dxpav amogpevyea. 0 5é’AvaE(Bv0s katadpapov érl Oadatrav év dduevTiK@s Twroiw TepiéTrEL els THY AxpdTrohuD, Kal evOds petarréurreras x Karynddvos dpoupovs: ov yap ixavol éddxour eivat

a 2 a > / al \ + } e be n ¢ 75 = A of év TH axpoTroAe oxelv Tors avdpas: of O€ oTpaTL@TaL ws eidov RevodarTa, mpoomimrovot Toddol avTG Kal Aéyovor- “Niv oo eeotw, @ Fevodar, > \ / 4 i / 4 7 64 A avodpi yevéecOm. eyes modu, Exes Tpinpes, exes Yphnwata, eyes avdpas TocovTous: viv av, et BovrAoL0, ov TE HuaS OvnoaIs4 Kal Hels TE WeyaV TroLN- cauev.” 6 8 amexpivato: “’AXX’ ed ye AéyeTe Kal Troijow TadTa: ei SE TouTwy émibupeite, Oéobe Ta Ora ev TaEE WS TdyLTTA*’’—BovACpEVOS AUTOS Katnpeploa’s Kal avtds Te Tapnyyva® TadTa Kal TOUS GAXous exédevE Trapey- yuav kai Tiber Oa Ta Ora. of adTol bd’ éEavToY TaTTOmEvo. of TE OTAiTAL 3 > / , > > \ > / e \ > \ / e / év odiy@ ypovm eis OKT@ éyevovTo Kal of TeATAaCTal érl TO Képas ExaTEpOV ,

mapededpaynxecav.—Xenophon, Anabasis, VII, 1, 18-23.

‘I dvfxecros=irremediable. 7 xa0e7\xov=launched. 3 ddevrixds=fishing (adj.). 4évivnuc= help. Sxarnpeul{w=calm. 6 rapeyyvdw=pass the word along.

157

IQIS GREEK G—SIGHT TRANSLATION OF ATTIC PROSE

Saturday. 2-4 p. Mm.

Read the passage through several times before beginning to write. The translation should be exact, but clear and idiomatic English is required.

Translate into English:

[The envoy from Acanthus warns Sparta of the growing power of Olynthus in Thrace]

"EE ’AxadvOov kal "Ato\Nwvias, alrep péyrorar Tv Tepl “OdvvPov rodEwr, mpéo Bers adixovro eis Aaxedaivova. axovocavres 5’ of Epopor Gy Eveka HKOV, Tpod- nyayov avrovs mpos Te THY ExKAnoLay Kal Tovs guupaxous. EvOa by Krevyévns "AxavOtos EdeZev? “QD &vdpes Aaxedarporrol re Kal cbupayxot, oldueba NavOdverv duds mpayyuat péya dvdyuevov? év rn ‘EAAGSL. Sr yey yap Trav érl Opaxns peylorn mods “OdvvOos cxedov mavres érioracbe. ovo? T&V TOAEwWY* TpoT- nyayovro éf’ wre vouous Tots abrots xphobat, erevra Kal Tov perCovwv mpoc- é\aBov Twas. ék d€ ToUTOU érexXEipnoary Kal Tas THs Maxedovias modeus EAevOEpody® ard ’Aubvrov Tod Maxedévwv Baciiéws. rel elofxovoay’ of éyybrara abrar, Taxv Kal érlt Tas Toppw® Kal peifous éropebovTo’ Kal KaTeAlropuey Huets ExovTas 75n &ddXas TE TWoAAGS Kal Tlé\Xav, Hrep weyiorn Tov év Maxedovia rod\ewv’ Kal "Aubvrav 5€ nobavoueba aroxwpodyTa TE ex TaV TOAEWY Kal Soov OvK? éxrEeTTTW- Kora On éxk waons Maxedovias. méuparres 6& Kal mpds Huds Kal mpds ’Azro\- Awviaras of ‘ONbvOLor mpoetrov Hutv Sri el pu Tapecducba ovoTparevoopEvoL, éxetvor eh’ uds lovey. uets 5€, © Avdpes Aakedarudrior, BovAdueba pév Tors marplois vouows xpjobar* ei pévror yr) BonOnoe TLs, avayKn Kal Hutv per’ Exelvwv viyveoOa.”’—Xenophon, Hellenica, V, 2, 11-14.

1 rpayua=power. 7 pvduevov= growing up. %ofro.=the Olynthians. 4767 rédewr,

supply tas. 5 颒 ¢re=on condition that. ° édevOepdw, compare éAeVOepos, 7 elajxovcay = yielded. *réppw=ata distance. 9% dcov odx = all but. , |

158

LATION OF HOMER

IQII

GREEK H—SIGHT TRANSLATION OF HOMER

Friday 3-45-5-45 p. m.

Read the passage through several times before beginning to write. The trans- lation should be exact, but clear and idiomatic English is required.

Translate into English:

Zeus HESITATES WHETHER TO SAVE SARPEDON AND Is PERSUADED By HERA TO Attow Him To BE SLAIN

"ED por éyov, Ste por Sapryddva, pirtratov avdpav, poip’ td IlatpdéxAovo Mevatiadao dapjvan. dix Oat wor kpadin pewove! hpeciv dppaivoyte, 7 piv Cwov édvta paxns aro daxpvoécons Geiw avapragas Avkins év mriovi? dypw,3 } non bd xepot Mevoiriddao Sapdaoow.”

Tov 8 jpeiBer’ erara Bodmis rorvia “Hpy “aivdrate Kpovidn, roiov tov pov eeu7res. dvdpa Ovyrov édvra, mdédor rempwpévov aicy, dy €G€dre1s Oavarouo Svanxéos4 eLavarAdoa; €p0’S arap ov Tou mavres eratveowev Oeol aArAot, dAXo d€ rou épéw, od 8 evi pect Bddrco oHow ai ke Cav’ réuyys Sapmydova dvde ddmovee, paleo, wy Tis erevta Gedy €9éAnor Kat GAXos wéurev Ov pirov vidv ard Kparepns topuivys. ToAAol yap mept dorv péya Ipidpoo paxovrat vices GOavarwv, Totow KOrov aivov évncets.7 GAN’ ei Trou hiros éori, tedv 8 dAopvperat® Hrop, ] TOL Mev pv Eacov evi Kpatepy vopivy xépo” to IlatpdéxAo1o Mevoitiddao dayjvas: aitap ériv bn Tév ye Airy Wyn TE Kal aidv, Téprew piv Odvarov te pepe Kat vydvpov *Yrvov, eis 6 Ke Oy) Avxins eipeins Snmov tkwvta.”’

“Os par’, ob8’ axiOnoe rarip avdpav te Gedy Te. aipotoecoas b€ Yuddas? KaTéxevev Epa e™ maida pirov Tinav, Tov ot IldrpoxAos EweAXev pbicav™ év Tpoin éprBwrAakt, THACH warpys.

—Homer, Iliad, XVI, 433-455, 458-461.

™6:x0a pwéuove=is in doubt. 2 arlovi=rich. 3 djuq here=land. 4dvonxéos = ill-sounding. 5 pd’ = do it. 6 téy = fGrra, Tévjces from évinuc. 8 dopper at = pities. 9 ¥.ddas = drops. t0%oage=on the earth. I d0icew =slay.

160

1912

GREEK H—SIGHT TRANSLATION OF HOMER

Friday 3-45-5.45 p.m. Read the passage through several times before beginning to write. The translation should be exact, but clear and idiomatic English is required,

Translate into English:

ZEUS Makes KNowN TO HERA His PLANS FOR THE DEATH OF HEROES AND THE CAPTURE OF ILIUM “Bi pev 6) ot y’ rata, Bodms rota “Hpy, : tov éuol ppoveovoa per aOavarocr xabiors, to xe Lloveddwy ye, kai ei pada Bovreror GAXy,

> , s eee, + ay a alfa peraotpepere voov peta adv Kal Euov Kip. GAN’ ei Oy fp’ éredv ye Kal atpexéws ayopevess, épxeo viv peta ida Pedy, kai Sedpo KaAcooov "Ipiv 7’ €AOguevar kat “AmdAXAwva KAvTOTOE OV, bhp’ f pev pera Aadv “Axodv xadAKoxiTOvOV €Oy, kai eiryor Toceddwvt dvaxte

, , \ & \ 5 4 Vr, ee mavodpevov 7oACuowo TA & pds Sdual’ ixéoBa, °Exropa 8’ étpivyct paynv és Poi Bos ’ArdAXov, atris 8’ éurvevonot pcvos, AehdOy 8 ddvvdwv ?

A , \ , San au viv pty Telpover3 Kata dpcvas, avTaa Axatovs avris arootpapyow avarKida pilav4 évdpoas,

, 8’ é \ fos , evyovres 0 €v vyvat rodvKAnion TETwoL TIyAcidew “AxtdAjos* 6 8 dvornoe dv éraipov IldtpoxXov* ov d@ Krevel éyxei haidipos “Extrwp *IXiov rpomdpobe, moras 6AEoavT’ aiyodss Tovs GAXous, peta 0’ vidv éuov Saprydova Siov. Tov xoAwodsevos KTevel “Extopa dios ‘AxidXeds. > “A + , 6 lal éx tod 8’ adv rou éreita taXiwgww ® rapa vydv

28 hay , 8 ; 1 6 eS aiey éyw Tedxount Svapmepés,’ cis 6 x’ “Axouot "IAuov aizd® éXouev “AOnvains da BovAds.

To mplv 8 ovr’ dp’ éya mavw xoXov ovTe Tw’ GAAov

3 / ~ > e > 4D? 37 davarwv Aavaciow duuveuev évOdd’ édow, mpiv ye To [IyAcidao TeXcuTynP vat eeASwp,

) ec ¢ a 2 A O92 7 , ws of tréoTnv? mpOTov, éud 8 erevevoa Kapytt, npate TO OT’ éuelo Ged. O€ris Haro yowvur,

Acoopnevy Tysjoae"AxiAAQa wrodiopGov.”—Iliad, XV, 49-77.

t éumvéw=breathe into, inspire; 2 d0vn=pain; 3 relpw = distress; 4 pvfa=flight; Saifnol = vigorous men; 6 rariwéis =pursuit; 7dvaumepés=continually; *%alrés=high;

, 9iméorny =I promised.

161

GREEK H—SIGHT TRANSLATION OF HOMER vee

Saturday II a.m.

Read the passage through several times before beginning to write. The translation should be exact, but clear and idiomatic English is required.

Translate into English: THE TROJANS ARE TERRIFIED BECAUSE ACHILLES HAS APPEARED AFTER A Lonc ABSENCE FROM BATTLE. PoLyDAMAS URGES IMMEDIATE RETREAT TO THE CITY. !

Tpaes & ail?” éEvépwber dro Kpatephs touivns

xopnoavtes EXvaav vd’ dppaciw wKéas larmovs:

és 8 dyopiyv ayépovto tapos Sdptroto® pédeo Oar.

ép0av & éctadtwv ayoph yéver’, ovdd Tis ETAH

&erbar mavras yap éye tpdpmos, obver’ ’Ayirreds

éEehavn, Snpov wayns érémavt’ areyevijs.?

roto TlovAvddwas rremvupéevos Hp’ ayopeverv

TlavOoldns: 6 yap oios Spa tpdcow Kal drriccw:

"Extops & jev éraipos, ins & év vue yévovto-

GAN’ 6 pev ap pvOorow, 6 & &yyer modAXdor évixa.

& cdi ed dpovéwy ayopncato Kal perder:

“’Audl udra ppaverbe, pido: KédNomat yap eywrye

aoatuoe viv tévat, pH) pipverv n@ Siav

év medi trapa vnvoiv: éxas & amo Telyeds eiper.

ddpa pév odtos avnp "Ayapéuvou pjue Sie,

toppa pniteport Trodepivery Hoav ’“Axatol.

yalpecxov yap &ywrye Oons emt vnuolv iadwr,s

éXmrduevos Vas aipnoduev audierioaas.

vov & aivas SelSorxa trodmxea Inrelova:

olos éxeivou Oupos brrépBr0s,° ove eOedyjoet

pipvew év medio, 60. wep Tpwes kat ’"Axatol

év péow aphdtepos pévos "Apnos datéovtat,?

GANA wepl wrodds TE paxynoeTaL HOE YyUvALKOD.

GAN’ lower mpotl’ dotu: riBecOd por: de yap éorat

viv pev vv0é arréravoe Trodw@Kea IInrelwva

auBpocin: ei 8 dupe xiynoetas evOad édvtas

avpiov opunbels adv Tevyeow, Ed VU TIS aUTOV

yvecetat: aotaciws? yap apiterat "Idov iphy

ds xe huryyn, ToAXAods KUvES Kal yires® EdovTat

Tpowv.”—Homer, Iliad, XVIII, 243-272.

t §dprov=supper; 2 ddeyervds = grievous; 3 iés=the same; 4 bnlrepos =easier;

latw=pass the night; 6 iépBios = wanton; 7 dar éowar= share; 8 wporl =mpés : domaciws=gladly; 10 yoy = vulture. Es

GREEK CH

HOMER /Z/AD, BOOKS I-III, AND SIGHT TRANSLATION OF HOMER

163

IQII

_ GREEK CH—HOMER’S ILIAD, I-III, AND SIGHT TRANSLATION .

Friday 3-45-5-45 p. m.

The translation should be exact, but clear and idiomatic English is required. Read the sight passage through several times before beginning to write the transla-

tion.

1. Translate into English:

2.

@dris 8 od Afr’ Eherucwv 495 maidds €08, GAN’ 7 y’ AveBioero Kiya Oadrdoons, nepin 8 avéBn wéyav obpavdv OtAvprdv Te, etpev 8 etpvora Kpovidny arep jpevov GAXwv dxpotaty Kopupy modvdepados Odriproro. Kai pa mapo.l’ avroio kabéLero, Kai AdBe yotvev 500 oxain, Sefitepy 5’ dp’ bz’ avOepedvos EXodoa Awwoopevyn tpocéerrre Aia Kpoviwva dvaxra: Zed warep, et rote by oe per’ AOavdrorrw dvyca

i) rer) Epyw, Tdd€ wow Kpynvov €éASwp.—I, 495-504

a) Give the Attic equivalents for Aer’ (line 495); €0d (line 496); OvrAdp- ovo (line 499); yovvwr (line 500); Segirepy (line 50r).

b) Explain the formation of dvedvcero (line 496), naming the tense and giving the tense-suffix. What is the first person singular present indicative active of ovyoa (line 503) ?

c) Account for the case of odpavov (line 497) and give the prose construction for this expression. What is the prose equivalent for per’ d0avdéroow (line 503) ? |

Translate into English: “AXN’ Gre 6H woddpytis avaiteev ’'Odvaceds, / c \ \ »” Ld otdokev, imal ideoke Kata XOovds Gupata mygas. oKynmtpov © ov’ éricw ovre mporpyves evopa, GAN’ dareppes Exeokev, dldpei putt éorxds: rd cd , ne gains Ke Laxorév Te Ti Eupevat adppova T avTws, 220 GAN’ ore OF) Ora Te peydAnv ex aTHOeos ein kai xen vipdderow €ouKdTa xXEmepinow, ovr’ av eet’ ‘Odvoni y’ épicaee Bpords aAAos* ov Tore y’ Gd ‘Odvonos dydooaped’ cidos idovres.—III, 216-224

a) Scan lines 221 and 223, marking quantities, division into feet, ictus, and chief metrical pauses.

b) Explain the apparent hiatus in line 221 and the quantity of re in this line.

c) What are the component parts of wodvuyris (line 216) and dxorov (line 220) ?

d) Where, in the books which you have read, does Ulysses show his ability as an orator?

164

3. Translate into English: Ajax Despairs oF BEING ABLE TO GET THE BoDy OF PATROCLUS O18’ Zab? Aiavra peyadrntopa kat MevéAaov

Zeds, dre 8) Tpweoor Sidov Erepadxéa! vixny. rotor pvOwv Apxe peyas TeAapavos Alas: “*O mdzou, non MeV KE Kal Os pdda VyimLds éoTW , 9 , A k ye > , yvoin ore Tpweoor ratnp Zevs avtros apyyet. 630 a Xx ‘\ , / 7 bd > , Tov pev yap mavtwv Bede’ arretat, Goris adyy, H xaxos % Gyabds: Leds & Eumrns avr’ diver? Cn Se a > 2 , quiv 8 attws3 raow erwo.as wimre epale.s GAN’ ayer’ airoi wep ppaldpeba pytw apioryy, > ‘\ 9 \ \ > 4 cEND! \ 3 A Huev Orws Tov vexpov Epvocopuer, HOE Kat avTOL 635 xdppa pirous Erdporor yevoueba voornoarres.””—X VII, 626-636

t érepadxéa =in turn; 2 (@bver = guides; 3adrws =just so; 4érwo.a =in vain;g 5€pafe=to the earth.

165

1912 GREEK CH—HOMER’S ILIAD, I-III, AND SIGHT TRANSLATION Friday 3.45-5.45 p. m. The translation should be exact, but clear and idiomatic English is required. Read the sight passage through several times before beginning to write the translation. 1. Translate into English: ot 3 eel ody HyepOev dunyepees 7’ éyevovro, trois. © dvuotdwevos peréyn mddas dks ’AyiAXevs* ‘’Arpeidy, viv Gppe maAcv TAayxOevras diw au arrovoorycey, el kev Oavardv ye piyoupev, 60 ci Ot) G6pwod woAEuds Te SapG Kai Aowwds “Axauors. GAN’ aye 8H Tiva pave epelopev, 7 tepfa, - } Kat dvepordAov—xai yap 7’ dvap éx Aids éorw— ds k’ €l7rol, 6 TL TéCGOV éxdoato Poi Bos “ArdAAwv, ei 7’ dp’ 6 y’ edxwdfis éxipeuerar, el O ExardouBys° 65 al Kev ws apvev kvlons aiyav Te TeX€ELwv Botrerar dvridcas yuiv amd Aovydv dydvar.”—I, 57-67. - a) Give the Attic equivalents for jyepOev (line 57); dume (line 59); dad (line 61); ieppa (line 62); tdéocoyv (line 64).

b) What mood is épecomev (line 62) and BovAerau (line 67) ? as the construction of ei kev piyouser (line 60).

c) Explain the case of cbxohijs (line 65); of «xvioys (line 66). What is the present indicative of dvridcas (line 67) ?

2. Translate into English: “<7 Wy PE TPOTEpw TOALWY EV vaLOoMEVawV ge afes 7 Ppvyins 7 Myovins éparewwis, ei Tis Tou Kal KeiOe piros pepdrwv avOpHrwv; ovvexa 59 viv Siov “AXcavdpov MevéAaos viknoas €Bédea. orvyepiv eye oixad’ d&yerOar, tovvexa Oi) viv dedpo Sorodpdvéovera zapéeorys ; 405 joo map’ avtov iotoa, Oedv 8 dmdeuxe KedevOov' py® ere coior wodecow trootpepaas “OAvuTov, GAN’ aici epi Keivov dilve, Kai E dvAaoce, cis 6 KE 0” 7 GAoxov rowjoerar, } 3 ye SovAv.””—III, 400-409. a) Scan lines 400 and 404, marking quantities, division into feet, ictus, and chief metrical pauses. b) Account for the metrical quantity of pe (line 400); the retention of the final vowel in éué (line 404). c) What is the composition, and the meaning of the component parts, of SoAodpoveovea (line 405) ?

d) State briefly the circumstances under which these words were uttered, naming the actors?

166

3. Translate into English:

ACHILLES PRAYS TO ZEUS FOR THE SUCCESS AND SAFETY OF PATROCLUS

“*A)X’ érapov réumrw mwoddow peta Muppiddverow , » A cal 9 , 3 , an papvacbur’ To KdOos Gua mpdes, edpvora Zed. O@dpovvov 8€ of Hrop évi hpeviv, dpa Kat “Exrwp »” ye \ > / eloeTau 7 pa Kal olos éxiotytas moAcuilew Hyerepos Oepdrrwv, 7 of TéTE Xeipes GarrToL , ra ee Ser A » x a » paivovd’, érmdr’ eyo wep iw peta wOAov “Apyos. avTap émei x’ ad vaddu paxnv evornv Te Sinrat, > , * \ ~ Fee." fol 97 doKnOns? por ererta Gods éxi vas iKowTo , , \ a \ 9 , (els revxeot Te Sty aot Kal ayxipdxois3 Erdporow. “Os par’ edydpevos, Tod 8 ExAve pytieta Leds. To 8 erepov pev Cdwxe TaTyp, Erepov & avevevoe’4 vyov pev ot dradcacbat rorEuov TE waXHV TE Saxe, adov 8 dvévevore pans && aroveerOau.—Iliad, XVI, 240-252. 6{nrac=drive off; 2 doxnO4s = uninjured; 3 dyxmudxos=that fight hand to hand; 4 dvévevoe=he denied.

167

1913 GREEK CH—HOMER’S ILIAD, I-III, AND SIGHT

TRANSLATION Saturday Q-II a. m.

The translation should be exact, but clear and idiomatic English is required. Read the sight passage through several times before beginning to write the translation. 1. Translate into English: "EvOa xev ’Apyelovow tréppopa vdotos érvyOn, 155 et 2) “AOnvainv “Hpn pos pidov éertrev> “°C adios, aiyidyoro Atos tTéxos, atpuT@vn, ovt@ Sh oixdvde Pidnv és matpida yaiav ’"Apyetou hevEovrar én’ evpda vata Oardoons ; Ka Kev evywrry Ipidu@ Kal Tpwol Aérrovev 160 ’"Apyetnv ‘Erévnp, fs elveca trodndol ’Axarov év Tpotn amerdovto, didns ad twatpisos aiys. GAN i viv Kata Nady ’Ayatov yadKoyiToVOD, cols ayavols éréecow épntve HOTA ExacTor, pndé a vijas ddad’ Eképev audueriooas.”’—II, 155-165.

a) Give the Attic equivalents for éerev (line 156); «dé (line 160); ans (line 162); vias (line 165); éAxéyer (line 165).

b) Explain the quantity of 67 (line 158); of -as in vas (line 165); the formation of «dé (line 160).

c) What is meant by tmesis? Give an example in this passage.

2. Translate into English: Actrepov atr’.’Odvoja idav épéew’ 6 yepaids: “elim aye por Kal Tdvde, Hirov Téxos, dots 68 éorivs pelov pev Keparn ’Ayapuéuvovos ’Atpeidao, evpvtepos 8 amorow ide orépvoiow idécbar. Tevyea wey ot KeiTaL él YOovi TovuPoTeipyH, 195 avtos 5é, xTiNos Os, émriT@rcitat otlyas avdpav: apve@ pv éyoye dono TnyerimdrrA®, és 7 div péya Trev Sidpyetas apyevvawv.”—ITI,. 191-198. a) Scan lines 195 and 198, marking quantities, division into feet, ictus, chief metrical pauses. b) Give the derivation of ovduBorelpy (line 195). Why does the poet prefer this form to 7oAvPorelpy ? ¢) Who speaks the words of this passage? Who is addressed?

168

3. Translate into English:

ACHILLES’ PREMONITION OF THE DEATH OF PATROCLUS IS CONFIRMED BY THE NEWS OF HIS FALL “?H udra 62 téOvnxe Mevortiou adximos vids, oxyéeThuos* 9 7 éxédevoy aTrwcapevoyv® Sytov Tip dap eri vias inev2 und “Exropt ids udyerOa.” Eios 6 tad’ ppawe cata dpéva cal cata Oupdr, Toppa ot éyyiOev AAGev ayavod Néoropos vids Sdxpva Oepya yéwv, ato & ayyerinv areyevyv's “6”, wot, Linrdos vie daippovos, 7) waddra Avypis TEVTEAL AYYEAins, 7 eh WheArXrE yeveo Oat. keiral4 ILdtpoxdos, véxvos 2) audimayovrar yupvov: atap Ta ye Tevye’ 2yer KopvOaionros “Extwp.” “Os dato, tov & dyeos veheérn exaddruwe pératva. : | —XVIII, 12-22. x admwiéouat=beat back; 2 tuev = iévat; 3 ddeyervds = grievous; 4 xetrat=lies dead.

169

1914 GREEK CH—HOMER’S ILIAD, I-III, AND SIGHT TRANSLATION

Saturday : Q-II a. m.

The translation should be exact, but clear and idiomatic English is required. Read the sight passage through several times before beginning to write the translation.

1. Translate into English: “Xalpere, enpuces, Aros dryyeror HOE Kal avdpar. dooov it’: ob Ti por Tppes érrattiot, AA *Ayapéeuvor, 335 é ape mpote. Bpionidos eivexa Kovpns. Grn’ aye, Scoyeves Tlatpdéxres, éEaye Kovpnr, kal odo Sos &yew. Too adta wdptupa error mpos Te Oe@v waxdpwv mpds Te Ovntav avOpaTrav ‘Kal mpos Tod Baoidfjos arnvéos, el tote 6) atte 340 Kperm éweto yévyntar aerxda Aovyov apdvar Tots GdAos. yap hy SroiTjor hpeol Ove, ovo TL olde vonoa dua Tpdcow Kal oTricca, drrmTws of Tapa vyvol cdo paxéovTat ’Ayavol.”—T, 334-344. a) Give the Attic prose equivalents for tyes (line 335); PacvAjos (line 340); éueio (line 341); dAoujor (line 342); vyvod (line 344). b) Explain the syntax of por (line 335); of dyev si 338); of yevyra: (line 341); Of paxéovras (line 344). c) What part has Achilles played in preceding events that Agamemnon now takes away his prize? Who was the prophet concerned ?

2. Translate into English: Tov 8 as ody évdnoev ’AXéEavdpos Oeoerd7s 30 év mpoudyouor havévta, kateTAIHyn hirov Hrop, avy & érapwv eis EOvos éyalero Kipp’ adecivor. e 8’ ¢ /, Ul > 5 \ / > / ws 0’ Ste Tis Te Spdxovta id@v Tadivopoos atréoTn ovpeos ev Bnoons, Ure TE Tpdmos EXAXaPeE yuIa, avy & aveyopnoev, pds py etre TapELas, 35 &s adtis Kal’ Susrov &dv Tpowyv ayepoyov defoas "Atpéos vidv ’AdXéEavdpos Oeoedys. \ 5) 7 / 39 \ > a > /

Tov © "Extop velxeroev dav aicypois éméecow:

“Avorrapt, €id0s apioTe, PORES ad aralaebdes aid’ bpedes Byovds 7’ Epevar dyapds 7 amrodécOar.””—ITI, 30-40.

r€

a) Explain the tense used in the main verbs of the simile.

6) Account for the case of jrop (line 31); of wapeids (line 35); of dyovos (line 40).

c) Write line 40 in Attic prose, expressing the meaning without the use of OdeXres, d) Scan lines 34 and 40, marking quantities, division into feet, ictus and chief metrical pauses. Account for the quantity of re (line 34); the quantity of the final syllable of €uwevar (line 40).

3. Translate into English: [Night has come on and Hector asks Ajax to postpone their combat to another day.| Aiav, émet tor Ske Oeds péyeOds te Binv Te kal mivuTnv, wept & éyyer Axardv héptatds éoor, vov pev Tavo@pecOa wayns Kal SnroTHTos onpepov'? UoTepoy avTe waynodped’, eis 6 xe Saiuov dupe Staxpivyn, don & érdporot ye vinnv: vue & dn TerOer4 ayabov Kal vuKrl mibécbar- as ov T’ éeudpnvyss TavTas Tapa vnvow ’Ayxa.ovs, cous Te waduota étas® Kal éraipous, of Tor éacuv: avTap éy@ Kata aotu wéya Ipidmoro avaxrtos Tpaas évdpavéw cai Tpwadas édxeovrrérXovs.7 Sapa 8 ay’ adAjdovot TepiKrAvTA S@opev Gudw.”—VII, 288-299.

tmivuT#=wisdom. oypyepov=today. 3diaxplyw=decide between. ‘47TeAéGer=is here. sedppalvw=delight. S@ras=kinsmen. 7 €AKeow.rém)dous=with trailing robes.

IQI5 GREEK CH—HOMER’S ILIAD, I-III, AND SIGHT TRANSLATION

Saturday Q-II a. m.

The translation should be exact, but clear and idiomatic English is required. Read the sight passage through several times before beginning to write the translation.

1. Translate into English: “Tav viv uv pyjncaca tapéleo Kal AaBE yowvor, ai Kev Tras €0édnow éml Tpdecow aphéat, Tovs Kata Tpvuvas Te Kal aud’ dra roa "Axatods KTeLvoevous, tva TravtTes érravpwvTat BacirHos, 410 a be \ .” /d > \ / > / yv@ o€ kal ’Atpeidns evipd xpetov ’Ayapéuvor a 54 ee Se > fal >Q\ 54 nv arn, 6 7 adpiotov ’"Axardv ovdev éticev. tov & npelBer’ éreata Oétus kata Saxpu yéovoa: “ce , s. Y { ole aay > \ A S @ mol, TéeKVOV Eudv, TL VU o EtpEehor aiva TEeKodca; aif’ dderes trapa vnvoly addKpuTos Kal aTnwov : 415 hoOa, érel vb Tor aica pivuvOa trep, ob Te wdda Synv.”—I, 407-416.

a) Give the Attic prose equivalents for rapéfeo (line 407); ‘yobvwr (line 407); €édAnow (line 408); Tpweoow (line 408); Baoidjos (line 410); vyvoiv (line 415).

b) Account for the use of the subjunctive in line 408; the mood of ddeXes (line 415); the case of Baow7jjos (line 410).

c) What cause does Homer assign for the expedition against Troy ? What © is the reason for Achilles’ wrath ?

2. Translate into English:

Tar 0’, ds 7 dpvi0my Terenvar EOvea Troddda,

xnVvav } yepavev 7) Kikvwv SovrALyodeipwr, 460

’"Acio év Netwau, Kavorpiov audi péeOpa,

4 » n > / s

év0a kab év0a TrotavTat ayadroueva TTEpUYyErow, \ / o / /

KNayynoov mpoxabiforvtwv, cuapayel Te Netpmov,

a a » \ a \ /

&> TOV €Ovea TOAAA VEdY ATO Kal KLCLAwY

és trediov TpoyéovTo LKapavdptov> avtap vo yOwv 465 / U lal > n \ &

opepdardov Kova. today avtav Te Kal imrov.

éotav © év Aexepan XKapavdpio avOeudevte

puptor, dooa Te PUAXA Kal avOea ylyverar Bpy.—II, 459-468.

a) Narrate briefly the events, related in Book II, which lead up to this mar- shaling for battle.

172

b) Scan lines 461, 462, 463, marking quantities, division into feet, ictus, and chief metrical pauses. Explain the quantity of the last syllable of ’Aciw (line 461). Comment on the effect of the dactylic and spondaic feet in lines 462, 463.

c) Give the derivation of rerenvév (line 459); of kdvordwy (line 464).

3. Translate into English: [Hector comes to aid Sarpedon|

Kal wt «’ étt mréovas Avkiwv xtdve dios "Odvaceds, 3 \ ek: ae \ / / / A ef 7) ap’ 0&D vonoe péyas Kopv@aionros “Exrap. 680 nq \ / / y” A BA Se 8a trpopayev KexopvOuevos aiPor.e yark@ Seiuat dépwv Aavaoior- yapn & apa of mpocidvte Laprndav Ards vids, Eros & dropudvov? Eevtrev: “TI pvawldn, wn On we EX@p3 Aavaoiow éaons Keio Oar, AA’ érrapuvoy: éreta we Kal Nitro ai@y4 685 , G , inn ah 5) yo 0 eee EV TTONEL UMETEPN, ETTEL OVK AP EMeAXOV EYW YE vootnaas oikovee hirny és watpida yaiav evdpavéevs aroxdv Te diAnv Kal vyTLOV vidv.” “Os dato, tov & ov Tt trpocépy KopvOaionros “Extap, arra trapnEev® Aedinpmevos, Oppa Tayiota 690 @oat’? *“Apyetous, todkdwv & amd Ovupuov €dorTo. e \ tA a /, PS) / yi vad of pev ap’ avTiGeov Laprrnddva Siow éraipor eloav® wr’ aiyidxoro Avds Tmrepixarres dn y@2—V, 679-603. t Sefua, compare deldw. 2 d6dopvdvdv=plaintive. 3&\wp=prey. ‘4alév=life. 5 eddpalyw= delight. 6 wapyEev Aehinuévos=rushed eagerly by. 7#6éw=thrust back. 4 eoav=placed. 9 pyybs=oak tree.

173

y+ Sb Dy

_ a

co

,) * , 4 = road

'

ay

=

cob

ii--

§ cae Pon kn wae mae hat

arty Wed

for m5

THIS BOOK IS DUE ON THE LAST DATE STAMPED BELOW

AN INITIAL FINE OF 25 CENTS

WILL BE ASSESSED FOR FAILURE TO RETURN THIS BOOK ON THE DATE DUE. THE PENALTY WILL INCREASE TO 50 CENTS ON THE FOURTH DAY AND TO $1.00 ON THE SEVENTH DAY OVERDUE.

AUG 16 1937

OCT 26 1939

e

REC'D LD ye loys) a,

LD 21-—5m-6,'37

NY

- . —=——, : e | . © ; : =i . | x a ‘a . A 4 ; “te 4 4 ig ee : ts : , > ; \ a *y : ‘4 4 » a al . 7 . - prey # . ; - & 2 = L ~ r > 4 7 * ae) Mgaa)t F a 4 : am bie a P - ? f « ae 7